TORTS I /II /III - MCQ

Lakukan tugas rumah & ujian kamu dengan baik sekarang menggunakan Quizwiz!

Question 4 2 / 2 pts Acme Gravel Company was blasting for new gravel. Acme spent the afternoon blasting rock from the side of a mountain. A few miles away, Peter maintained a mink farm. Unfortunately, the female minks are very nervous animals. After about an hour of blasting, several of the mother minks began to kill their young babies. Although Peter discovered what was happening and managed to save some of the animals, he did, however, suffer a substantial loss. If Peter sues the Acme Gravel Company for the loss, which of the following is correct? Peter will lose, since the harm was caused by the nervousness of the minks and not the blasting of Acme. Peter will lose, since blasting is not an abnormally dangerous activity that gives rise to strict liability. Peter will win, since blasting is an abnormally dangerous activity giving rise to strict liability. Peter will win, since the harm was caused by the blasting.

A is the correct answer. Restatement Second states: "There is no strict liability for harm caused by an abnormally dangerous activity if the harm would not have resulted but for the abnormally sensitive character of the plaintiff's activity." This question contains facts similar to the case of Foster v. Preston Mill Co., 44 Wash. 2d 440, 268 P.2d 645 (1954). It appears from the facts that the blasting was not the cause of the harm. The minks are nervous and could have begun to kill their young under many different circumstances. It did not take the abnormally dangerous activity to create the harm. B is incorrect since blasting is generally considered to be an abnormally dangerous activity, giving rise to strict liability. C is wrong since it appears from the facts that the blasting was not the cause of the harm. The minks are nervous and could have begun to kill their young under many different circumstances. It did not take the abnormally dangerous activity to create the harm. D is wrong since the harm appears to have caused by the mink's nervousness, not from the blasting.

Question 12 2 / 2 pts Late one night, Cindy hears someone outside her window. She throws open the door and finds Pete, the neighborhood troublemaker, swimming in her pool. Cindy yells at Pete, who immediately jumps over the fence into the adjoining property, owned by Dustin. Unfortunately for Pete, Dustin had just purchased a deadly pet python to protect his property. When Pete landed on the other side of the fence, the python started strangling him. Pete is soon rescued, but not before suffering serious injury. If Pete sues Dustin for his injuries, he will likely: Lose, as long as Dustin has posted signs stating, "Beware of Attack Python." Lose, since Pete was a trespasser on Dustin's property. Win, because Dustin is liable for any damages to a trespasser on his property. Win, since Dustin cannot use a dangerous animal like a python to protect only his property.

D is the correct answer. When dealing with the intentional use of a dangerous animal to protect property, Dustin loses and Pete wins, as such use is not permissible. Only reasonable force can be used in defense of property, not force that would cause death or serious injury. Here the python is deemed excessive deadly force. Also recall that indirect deadly force (like spring guns) is not permissible if this force cannot be lawfully used appropriately. In other words, deadly force is not permitted as a deterrent against mere trespassers. B is wrong. Generally the court will not apply strict liability in cases involving trespass onto land. Strict liability typically does not attach to injured trespassers as used against a landowner, but rather courts look to principles of negligence to determine if the landowner acted unreasonably by harboring an unreasonably dangerous animal etc. C is wrong. A landowner, who keeps a dangerous animal on his property that does damage to a trespasser, may be held liable for these damages under intentional tort principles. Since a landowner cannot use deadly force to protect his property, he cannot use deadly force indirectly to protect this same property by using a dangerous animal. A is wrong regardless of how many signs Dustin posts. Under intentional tort principles, Dustin's use of a python to protect his property knowing that the snake is likely to cause serious injury, may make him liable.

Question 9 15 / 15 pts A twelve-year old boy suddenly ran out into the street chasing a basketball, in the path of a car being driven by the Dan. Since Dan was speeding in violation of a city vehicle code, he was unable to stop in time to avoid hitting the boy. The boy suffered an injured leg in the accident. The jurisdiction applies the all-or-nothing rule of contributory negligence. If the boy asserts a claim against the Dan for the injuries to his leg, which of the following would be Dan's most effective argument in defense? Dan had a clean driving record, with no infractions. Dan's violation of the city's vehicle code was not the proximate cause of the boy's injury. Dan was only driving 5 miles above the speed limit. The boy was negligent for running out into the street to chase a basketball.

In a jurisdiction applying the contributory negligence doctrine, any negligence on the part of the plaintiff would bar his action. Consequently, choice D is the only answer that would provide a successful defense.

Question 15 2 / 2 pts Alan, a football player, got onto the school bus to travel back to the school after a high school football game. Alan saw Evan, a team member, and started a fight with him on the bus over a girl. After Alan threw a punch at Evan, Evan kicked Alan, injuring Alan's leg badly. When the bus arrived back at the school, the bus driver locked the doors of the bus and refused to let Alan off until he apologized to Evan for starting the fight. Does Alan have a claim against the bus driver for false imprisonment? Yes, because Alan was detained in a confined area. Yes, if the confinement was for a substantial period of time. No, because the bus driver had a privilege to detain Alan. No, if Alan violated the transit rules.

A is the best answer because false imprisonment is the --intentional conduct causing a person to be --confined in a closed space --with no reasonable means of escape, --where the plaintiff is aware of the confinement. B is incorrect because confinement for a brief period of time will suffice. C is incorrect because there is no shopkeeper's privilege here and no other privilege is apparent. D is irrelevant to the analysis of the problem.

Question 8 2 / 2 pts Barb is a babysitter for Parents. Each day Parents provide her a detailed list of instructions including picking up the children from school, taking them to events, feeding them dinner and putting them to bed. When picking up the children from school one day, Barb got into an automobile accident with Pam, injuring Pam, after Barb failed to check for traffic prior to merging into traffic and did not see Pam coming along beside her. If Pam sues Barb and Parents, which of the following are correct? Barb and Parents are liable because Barb is an employee of Family. Barb is liable but Parents are not, because Barb is an independent contractor. Neither Barb nor Parents are liable, because Barb's conduct was not intentional. Parents are liable to Pam based on vicarious liability, but Barb is not liable to Pam.

A is the best answer here because --Barb is liable based on negligence given her failure to look before merging and --Family is liable because based on the detailed instructions used to control Barb's activity Barb will most likely be regarded as an employee. --B is incorrect because Barb will not be determined to be an independent contractor. C is incorrect because --Barb is liable based on negligence, and Family is vicariously liable because she was acting in the course and scope of her employment. D is incorrect because --Barb will be liable for negligence.

Question 15 2 / 2 pts Pam and Denise graduated from the same university and both interview for the same job. Pam had more extra-curricular activities and authored a widely published article. Denise however, had higher grades, although she did no such extra-curricular work. Denise is interviewed by Michael. They were talking in a casual manner and Michael asked Denise what she thought about Pam's article. Denise stated, "Oh, the article is brilliant. But I heard some grumblings from professors that Pam actually got help from Taylor for that article. Taylor practically wrote the whole thing." At the time she made the statement, Denise knew that Pam had written the article on her own. Based in part on this information, Denise got the job. One year later, Pam learned that Denise had made this comment during the interview and sues her for slander. Assume that there was never any such rumors from professors, and that Pam did not receive help from Taylor. Who will likely prevail in the slander suit? Pam, since Denise's statement to Michael was false. Pam, since in making the statement Denise showed reckless disregard for the truth of her statement. Denise, because Michael directly asked for Denise's opinion in this matter. Denise, unless special damages are proved by Pam.

A is the best answer. Defamation requires a showing of: 1) defamatory language on the part of the defendant; 2) where the defamatory language is "of or concerning" the plaintiff, 3) publication of the defamatory language by the defendant to a third person; and 4) damages to the reputation of the plaintiff. Here, Denise made a defamatory statement that concerns Pam. Denise published the communication to Michael who understood it. (B) is wrong for misstating the applicable standard. Answer (B) recites the needs test for cases dealing with public people (i.e., a politician or celebrity). In such a situation malice must be shown in making a false statement with reckless disregard for the truth in order to prove slander. Here, Pam is a private person, negating the need to show malice. (C) is wrong. Simply being asked her opinion does not justify Denise's defamatory response. Damages are dependent upon the type of defamation. If it is slander per se, a court does presume injury to a person's reputation, making it necessary to show damages supported by proof. Denise's statement relates to Pam's abilities in her work or profession, making her statement fall into the slander per se category and negating the need for Pam to prove damages. Therefore, (D) is wrong. Quiz Score: 30 out of 30

Question 2 2 / 2 pts Randy knew that Justin, a bully in Randy's 8th grade class walked through the woods every day on his way to school, as a shortcut, along a certain trail. After Justin beat up Fred, one of Randy's friends, Randy created a trap for Justin by digging a 10 foot deep hole and covering it with a thin layer of burlap concealed by fallen leaves. As expected, Justin fell into the trap the following day on the way to school. Justin injured himself and was stuck in the trap, and never made it to school that day. After school, Randy and Fred went and checked and sure enough, there was Justin, stuck in the trap. Randy and Fred left Justin in the trap, and gave an anonymous voicemail to Justin's mother, who finally found Justin, with the help of police, and got him out, later that evening. Justin broke his leg in the fall and had to go to the hospital. Randy is liable to Justin for: Battery, but not false imprisonment. Battery and false imprisonment. False imprisonment but not battery. Neither battery nor false imprisonment.

B is the best answer because Randy acted intending to confine Justin to the hole, based on the depth of the hole, so he was confined, and also experienced harmful contact as a result of Randy's actions, which suffices for both battery and false imprisonment. For these reasons, answers a, C and D are incorrect.

Score for this quiz: 28 out of 30 Submitted Dec 21 at 1:10am This attempt took 27 minutes. Question 1 2 / 2 pts Cars Go manufactures hybrid cars. Phyllis purchased a hybrid car from Cars Go. The first time that she drove the car, the brakes gave out and Phyllis crashed into another car, causing her to suffer serious injuries. Accident investigators determined that a critical component of the braking system was missing and that the accident resulted from this defect. Which of the following statements, if true, would constitute Cars Go's best defense to a strict products liability action by Phyllis? Cars are inherently dangerous, so Phyllis assumed the risk when she drove the car. Phyllis tampered with the brake system before she drove the car. Phyllis had never driven a hybrid car before. Cars Go has been rated as the safest car in the world three years in a row.

B is the correct answer. A plaintiff may not recover in strict liability if the manufacturer can show that the product had been altered after it left the manufacturer's hands. A is wrong since cars are not considered to be 'inherently dangerous products'. Even if it was inherently dangerous, the manufacturer would not be liable if the injury was caused by a substantial alteration in the condition of the product. C is wrong since it is irrelevant. Even if Phyllis drove negligently, her claim would still be viable, unless she substantially altered the car's condition. D is irrelevant.

Question 6 15 / 15 pts Alex is peddling his bike along a bike path when he reaches a pedestrian bridge. The bridge has a sign stating: "No bikes, rollerblades or skateboards under penalty of law." Alex rode this area every day and used the bridge regardless of the city ordinance, because the alternative requires that he ride along a busy intersection. After crossing the bridge and getting back onto a continuation of the bike path, Alex travelled another 20 feet when he was run over by Big Bob who is driving a station wagon. Big Bob's failure to stop at the stop sign is the reason he plowed into Alex. This action was in violation of an applicable traffic statute. Had Alex avoided crossing the bridge in violation of the statute he would not have been in the intersection where Big Bob hit him. Alex's violation of the bridge crossing ordinance can, therefore, be viewed as: An intervening superseding cause of Alex's harm. A contributing but for cause of Alex's harm A natural cause of Alex's harm. An artificial condition of Alex's harm.

B is the correct answer. Alex's violation of the bridge statute regarding bicyclists can be viewed as a contributing actual cause of his injury. Note, though, that it would be a stretch to call it the proximate cause of Alex's injury. In light of this, both Answers A and D are incorrect since Alex's violation was neither the actual cause (not the cause of Big Bob running the stop sign) nor an unrelated cause, as absent the bridge crossing Alex would not have been here. C is wrong because Alex's act of crossing the bridge with his bike is not a natural condition.

Question 10 2 / 2 pts Peter and his friend Frank were water skiing on the lake when their boat stalled out. Frank decided to swim for shore but Peter was not the best swimmer, so he decided to wait for another boater to come by to pick him up and take him to shore. Daniel was out boating on the lake and saw Peter. Peter signaled to Daniel by waiving his arms and yelling. Daniel was in a hurry to get back to shore, and ignored Peter. Peter suffered severe illness as a result of being left on the lake and unable to get his medications. If Peter asserts a claim against Daniel, for failing to provide assistance to Peter, will Peter prevail? Yes, because Daniel's failure to rescue Peter increased the harm to Peter. Yes, if the risk of harm to Peter outweighed the inconvenience to Daniel. No, unless Peter and Daniel had a special relationship. No, if Daniel was truly in a hurry to get back to the shore

C is correct. There is no duty to rescue absent a special relationship. A, B and D are wrong and not relevant to the analysis. There is no duty to rescue absent a special relationship. Quiz Score: 20 out of 20

Question 18 15 / 15 pts Dalton owns a minor league baseball team that plays at the local park. Patrick has attended games at the park many times, and loves to sit on the third base line near the foul pole, although he knew that he had to pay close attention to the game, because the players sometimes hit balls into the stands and he had nearly been hit before. Dalton has posted signs at the entrance warning patrons of the possible of danger from baseballs flying out of the field and into the stands. Dalton also has the announcer warn fans at the start of each game. Patrick was drinking a soda when a foul ball came into the stands knocking him on the head. Can Patrick recover on a claim for negligence against Dalton for failing to have a screen in place in order to protect against the possibility of harm from a foul ball? Yes, because the expense of taking further precautions was modest. Yes, because the possible harm to fans was significant. No, because Patrick voluntary assumed the risk. No, because the baseball player who hit the ball into the stands was a superseding cause of the harm.

C is the best answer based on the doctrine of assumption of risk, given that Patrick knew of the risk and attended the game. A is incorrect because it does not account for the defense of assumption of risk which would prevail here. B is incorrect for the same reason. D is incorrect because the conduct of the baseball player was foreseeable and therefore would not cutoff liability for the harm.

Question 13 2 / 2 pts Mother was on an airplane, flying from New York to her home in Hawaii. The airline negligently failed to completely fuel the plane, which was forced to land in a remote area of Hawaii very near a volcano for emergency refueling. The volcano unexpectedly erupted. As Mother ran from the flowing lava of the volcano, she tripped and fell and was injured. Another passenger, a medical doctor, tried to help Mother, but actually caused additional serious damages to Mother's leg, which was broken in the fall. Assume the jurisdiction has a typical Good Samaritan statute. If Mother sues the passenger, who will prevail? Mother, because a doctor has a duty to rescue others. Mother, because the other passenger is liable for any injury to mother while rescuing her. The other passenger, unless he acted with gross negligence in assisting mother. The other passenger, because he had no duty to rescue mother, he was not liable for causing her injuries.

C is the best answer because a Good Samaritan statute typically limit liability to gross negligence or reckless conduct. A is incorrect because there is generally no duty to rescue, even for a medical provider. B is incorrect because it is too broad, since under a Good Samaritan statute, there is not liability for negligent rescue, and no strict liability. D is incorrect because once the passenger intervened he had a duty to rescue in a non-negligent manner.

Question 11 2 / 2 pts Matt, a famous movie actor, enjoys Bad Bull Beer. Bad Bull Beer took a photo of Matt drinking a Bad Bull Beer on the beach and decided to use the photo in an ad in a sports magazine and on the related website. The Bad Bull Beer label was visible in the photo. If Matt did not authorize the advertisement, he can sue Bad Bull Beer for: Defamation, if the advertisement harmed Matt's reputation. False light, if Matt is not really a beer drinker. Misappropriation, even if the photo was an accurate depiction. Intrusion upon seclusion, if Matt was with his family at the beach.

C is the best answer because misappropriation of likeness for financial benefit is an invasion of privacy. A is incorrect, because there no indication that the advertisement was false, which is required for a claim of defamation of a public figure. B is incorrect because the image is not false. D is incorrect because Matt was on a public state beach so his private space was not invaded as required to establish intrusion upon seclusion.

Question 7 2 / 2 pts Patty bought a new food processor made by Sun Light Corp from Retail Mart. The device came fully packaged and shrink wrapped, so that Retail Mart did not have any chance to inspect it. Patty carefully read the instructions before using the device and followed the manual step by step. Unfortunately, the blade assembly became dislodged unexpectedly during cleaning, and Patty severed her finger. Patty sued both Retail Mart and Sun Light Corp. for strict product liability. Retail Mart settled the case before trial for $100,000. Patty went to trial against Sun Light Corp. The jury awarded Patty $1 million in damages, including pain and suffering. Sun Light sued Retail Mart for contribution. Under the modern approach, what is the likely result? Retail Mart is liable for contribution of $1 million to Sun Light. Retail Mart will not be liable for contribution, and Patty will recover $1 million from Sun Light. Retail Mart will not be liable for contribution, and Patty can recover $900,000 from Sun Light. Retail Mart is liable for contribution to Sun Light based on Retail Mart's share of proportional fault.

C is the best answer here under the modern view, most courts will reduce the plaintiff's claim by the dollar amount of the settlement. Sun Light, however, as the manufacturer, would not be entitled to contribution from the retailer, Retail Mart. A is incorrect because the retailer is not subject to contribution. B is incorrect because under the modern view, there would be a reduction to the plaintiff's claim that is reflective of the earlier settlement. D would be true generally had there been no settlement; but under the modern view the settlement amount limits Retail Mart's liability.

Question 7 2 / 2 pts Dave runs an Animal Park located in a remote area where vacationers often go to enjoy peaceful camping trips on summer vacation. Dave's Animal Park has an excellent reputation for providing high quality care to all of the wild animals housed on its property. The park had a habitat with Bengal Tigers, some of which became particularly ornery in the summer heat. Pete and his family were camping on a campground located several hundred feet outside of the fencing that surrounded the Animal Park. One day during the camping trip, the temperature soared to 100 degrees. At night, one of the tigers became very agitated and managed to break through the security fencing and roamed into the campground. The tiger approached Pete who managed to cry out for help, alerting Dave that the tiger had escaped. Dave captured the tiger, but not before Pete received severe cuts and bruises from wrestling with the tiger. If Pete asserts a claim against Dave and the Animal Park for his injuries, he will likely: Not recover, because Dave took great care in maintaining the Animal Park. Not recover, because Dave is not liable for injuries caused by his animals when they escape. Recover, but only if Dave did not properly maintain the security fencing. Recover for all of the harm done by the tiger.

D is the correct answer because the possessor of a wild animal is subject to strict liability for harm done by the animal to another person. This is true whether or not the animal owner exercised the utmost care to confine the animal or otherwise prevent it from escaping and causing harm. The possessor of a wild animal is subject to strict liability for harm done by the animal to another person. This is true whether or not the animal owner exercised the utmost care to confine the animal or otherwise prevent it from escaping and causing harm. For that reason, A and B are incorrect. C is wrong because the owner is strictly liable whether or not the owner is at fault for allowing the animal to escape.

Question 12 0 / 15 pts Ryan often went to a local supermarket to buy groceries. Ryan had some old books that he wanted to donate to the public library and he needed boxes to transport them. One day, he went to the supermarket and asked the manager if he had any empty boxes. The manager said that he did, but that they were located in the back of the store. Since many customers were present, the manager could not retrieve the boxes, but he stated that Ryan could go into the back and take as many boxes as he wanted. The manager told Ryan to "be careful back there because it's pretty messy. Be sure to turn on the light." Ryan failed to hit the light switch when he went into the back. Ryan tripped over a box, fell, and injured his back. Ryan sued the supermarket for negligence. At trial, the manager testified he had given Ryan a warning. The likely outcome of Ryan's lawsuit will be: Ryan will recover because the manager's warning was not adequate. Ryan will recover because the manager directed Ryan to a messy area. Ryan will not recover because the manager warned him to be careful and use the light. Ryan will not recover because the manager should have gone with him into the back.

C is correct since Ryan assumed the risk by walking into a dark area without turning on the light, in view of the manager's warning. A is wrong because the manager's warning is adequate in view of the risk posed. B is wrong because although the manager directed Ryan to a messy area, he warned him to use the light. D is incorrect because the warning is adequate.

Question 15 15 / 15 pts After being terminated by Dr. Dan from her employment as a nurse, Priscilla applied for a position with Spring Hospital. In her application, Priscilla listed her former employment with Dr. Dan, and authorized Spring Hospital to contact Dr. Dan. In response to a telephone inquiry from Spring Hospital, Dr. Dan stated that "Priscilla mishandled patient care in several situations by failing to give timely medication." Dr. Dan believed that to be a fair assessment of Priscilla, and his statement was based on several episodes in which he had observed that Priscilla's care had resulted in harm to patients who did not get their medication in a timely manner. Because of Dr. Dan's criticisms of Priscilla's performance as a nurse, she was not hired by Spring Hospital. If Priscilla asserts a claim based on defamation against Dr. Dan, will she likely prevail? Yes, because Dr. Dan's statement was slanderous per se. Yes, because Dr. Dan made materially false statements about Priscilla. No, because Dr. Dan's statements were privileged. No, because Dr. Dan's statements were opinions.

C is the best answer because Dr. Dan's statement was protected by a qualified privilege as long as they were not made with malice, which appears to be the case here, since his statements were based on personal observations. A is incorrect because although the statement did concern her work, it was privileged. B is incorrect because it does not appear that the statements were false, again since they were based on his personal observations. D is incorrect because his statement about her mishandling patient care is likely to be regarded as one of fact rather than opinion.

Question 6 2 / 2 pts Paul is a sales representative for a pharmaceutical company who calls on doctors by visiting their medical offices. Paul called on Dr. Davis to speak with him about a new clinical drug trial in hopes of getting Dr. Davis to participate. While in Dr. Davis' office, Paul tripped when his shoe became hooked on a frayed carpet edge and fell, hitting his head on a counter in front of the receptionist console. Dr. Davis leases the property from the building owner. What was Dr. Davis' duty to Paul? Paul was a licensee and Dr. Davis had a duty to warn him of the problem with the carpet. Paul was a licensee and Dr. Davis had a duty to inspect the carpet for issues. Paul was an invitee and Dr. Davis has a duty to inspect and repair the property. Paul was an invitee, but Dr. Davis had no duty to Paul because Dr. Davis did not own the property.

C is the best answer because Dr. Davis had a duty to Paul, an invitee who was in Dr. Davis' office for business purposes. D is incorrect because tenants are treated as owners for purposes of liability to third parties. A is incorrect because Paul is not an invitee??? or social guest because he is on the property for business reasons. B is incorrect because Paul is not an invitee??? or social guest because he is on the property for business reasons.

Question 8 2 / 2 pts Diane, an executive assistant for a large advertising agency, was driving in heavy traffic on her way to work when a bus swerved in her direction. Diane swerved in reaction to avoid hitting the bus, and hit a motorcyclist who was driving between lanes who she said "came out of nowhere." What standard will apply to evaluate whether Diane met the standard of care? Reasonable person of like age, intelligence and experience. Reasonable person of Diane's mental characteristics. Reasonable person under emergency circumstances. Reasonable person with Diane's physical characteristics.

C is the correct answer because Diane is operating in an emergency traffic situation. A is incorrect because the reasonable person of like age, intelligence and experience is the standard applied to children. B is incorrect because mental characteristics are not taken into consideration. D is incorrect because physical characteristics are taken into consideration only for persons with disabilities.

Question 13 15 / 15 pts On a commercial airplane flight, Pat becomes ill after eating the packaged snack offered by the airline and is vomiting uncontrollably. Dave, a physician also on the flight, recognizes Pat's illness, but does not take any steps to assist her. If Pat sues Dave for negligence, which of the following statements is correct? Dave had the ability as a physician to assist Pat, and his failure to do so is legally actionable. Dave had a special relationship with Pat, as co-passengers on the flight, and had a duty to assist her. Dave had an obligation to assist Pat given his expertise as a physician and the emergency nature of the situation. Dave had no obligation to assist Pat, and will not be liable for negligence.

D is the correct answer. Generally speaking, there is no duty to warn another of an impending danger, unless there is a special relationship with that person. Since the defendant was not acting as the plaintiff's physician, there was no special relationship here.

Question 9 15 / 15 pts Paul owes money to Devin, a loan shark. Later one evening, Devin stood on the sidewalk outside Paul's home and shouted, "Paul, you are a deadbeat. If you don't pay up your loan by tomorrow morning, I'm going to cut off your fingers". Paul is so terrified by Devin's threat that he has a nervous breakdown, requiring hospitalization. If Paul asserts a claim against Devin for assault, Paul will: prevail, since Devin intended to frighten him prevail, since Paul was in apprehension of a battery not prevail, because neither a harmful or offensive contact occurred not prevail, because Paul was not in apprehension of an imminent harmful or offensive contact

D is correct since assault requires that the apprehension be of an imminent battery. For this reason, B and C are incorrect. A is wrong since intent to frighten is not an element of assault.

Question 5 3 / 3 pts During an awards dinner, the judges announce that Paul is runner up and Don has come in first. The crowd slowly quiets in anticipation of Don's acceptance speech as he makes his way to the stage, at which point Paul yells out: "You only won because your wife, Rose, slept with all the judges!" Rose rushes onto the stage and punches Paul in the face. Paul sues for battery. Who wins? A. Rose, since Paul provoked her with an inflammatory derogatory comment. A. Rose, since any reasonable person would have punched Paul under the situation. A. Paul, unless Rose's punch was totally spontaneous. A. Paul, since Rose's actions caused an offensive touching.

D is the correct answer. A prima facia case of battery requires the following elements: an act by the defendant that brings about harmful or offensive contact, intent by the defendant to bring about the contact, and causation. Each of these is clearly present here, and there is no valid defense that Rose can advance. (Provocation is not a defense.) Rose punching Paul in the face clearly is an offensive touching. A and B are wrong. Again, actual provocation or reasonable provocation is never a defense to a battery claim. C is wrong since even spontaneous actions are deemed volitional and the "act" requirement of the battery elements is met by these spontaneous actions. Note: Any action that is triggered when a person is conscious is deemed volitional movement and qualifies as an "act" under battery (e.g., sleep-punching may not be deemed an "act" under battery).

Question 9 3 / 3 pts Polly was relaxing in her living room when she heard the screech of the brakes on David's car. A young boy chasing a ball had run out into the road and David slammed on the brakes to try to avoid hitting him. Unfortunately David hit the boy, causing the boy's body to fly through the air, landing on Polly's front lawn. When Polly saw the boy lying on her lawn, Polly suffered a heart attack and was hospitalized. If Polly asserts an action for intentional infliction of emotional distress against David, who is likely to prevail? Polly, since she suffered a heart attack. Polly, since David's action in hitting the boy caused her to suffer severe emotional distress. David, only if Polly had a pre-existing heart condition. David, since his conduct was not reckless or intentional.

D is the correct answer. IIED is the intentional or reckless infliction of severe emotional or mental distress by extreme and outrageous conduct. Here, the facts do not indicate that David acted intentionally or recklessly when he hit the boy or that his conduct in hitting the boy would be considered extreme and outrageous. A is wrong since David did not intentionally or recklessly cause Polly to suffer severe emotional distress (a heart attack). B is wrong since there is no intent on these facts. C is wrong since whether or not Polly had a pre-existing condition is irrelevant to IIED, if the other elements of the tort are present.

Question 9 0 / 2 pts Alex drives a Ferrari. One night, when Alex parked his car on his street, Dex broke the window of the Ferrari, unlocked the door of the car, hotwired it, and drove off. While speeding down the road, enjoying his stolen car, the engine exploded and Dex was severely injured. Upon investigation, it was discovered that during the manufacturing process, a small part of the engine was installed improperly. If Dex asserts a strict products liability action against Ferrari, he will most likely: Prevail, if the vehicle was sold in a defective condition. Prevail, if Alex failed to inspect the vehicle as a reasonably prudent owner would. Not prevail, because Dex was speeding. Not prevail, because Dex was not a foreseeable user or consumer of this vehicle.

(D) is correct. Under Section 402A of the Restatement of Torts 2d, any "foreseeable user" or "ordinary user" of a defectively dangerous product may recover for strict liability. Prosser states that "the user may be a member of the family of the final purchaser, or his employee, or a guest at his table, or a mere donee from the purchaser." In this regard, a thief is not regarded as a reasonably foreseeable user. A is incorrect since as an improper plaintiff, Dex cannot succeed on a strict liability claim, even if the product is defective. B is wrong since Alex's actions are irrelevant since Dex is not a proper plaintiff. C is wrong since as a thief, Dex is not a proper plaintiff, making his actions after he stole the vehicle irrelevant.

Question 10 15 / 15 pts Carnival Company runs an amusement park. Debbie decided to take her children to the park. She asked her neighbor if the neighbor's three-year-old daughter, Monica, could go with them. The neighbor agreed, so Debbie, her children and Monica went to the park. After they arrived, Debbie went to buy cotton candy and temporarily lost sight of Monica. Monica was attracted by a ride that had colorful cars spinning at a high rate of speed. The ride was surrounded by a fence, but the ride attendant had left the gate open. Monica walked through the gate, approached the ride, and was struck by one of the cars. She suffered serious injuries. Simon, who was 18-years-old and a visitor to the park, watched Monica walk through the gate but did not try to stop her or alert the attendant. If Monica sues Carnival Company, Debbie and Simon for negligence, she will prevail against Carnival Company and Simon only. Carnival Company and Debbie only. Carnival Company only. Carnival Company, Cynthia and Simon.

B is correct since Debbie is negligent for failing to supervise Monica and the Carnival Company is negligent for leaving the gate open. A is wrong since Simon had no duty to rescue Monica. C and D are wrong for the reasons stated above.

Question 5 0 pts El Mansion is a huge estate that belongs to Adam, the sheriff of Blackhawk, a town of 100,000 people. El Mansion is bought by Patrick, the CEO of the largest company in Blackhawk. A reporter notices that the "for sale" sign has been removed from the lawn of El Mansion. He looks through public records and finds the sale price. The local newspaper runs a story that says, "El Mansion was recently purchased for $1 million by Patrick, CEO of one of the largest companies in Blackhawk." Additionally, the reporter includes a brief bio about Patrick, as well as a photo of the mansion taken from across the street on the sidewalk. Patrick is livid. Assume that Patrick sues the newspaper for invasion of privacy. Will Patrick likely win? Group of answer choices Yes, he never consented to publication of info about himself or his new house. Yes, if a reasonable person of ordinary sensibilities would have been upset by publication of the story. No, because the reporter printed publicly available facts. No, because the story was true.

C.) Publicly disclosed facts cannot be the basis for which an invasion of privacy claim is filed. Invasion of privacy falls into four categories: 1) appropriation of the plaintiff's picture or name for the defendant's commercial advantage; 2) intrusion upon the plaintiffs affairs or seclusion; 3) publication of facts that place the plaintiff in a false light; and 4) public disclosure of private facts about the plaintiff by the defendant. Category 1) is inapplicable as use of a photo or name does not qualify as a commercial appropriation, something generally viewed as the use of a product name or image (e.g., the Nike swoosh or the Ralph Lauren polo player on his horse). Category 2) is also inapplicable as info gathered from public sources and photos snapped from outside the place on the sidewalk is allowed. The publication does not place Patrick in a false light, making category 3) inapplicable. Category 4) is also inapplicable as none of the published facts can be considered private in nature. (A) is wrong. The fact that Patrick never consented to the publication only means that the reporter and newspaper cannot use the defense of consent. (B) is incorrect for only stating one element of the prima facia case of public disclosure (i.e., an ordinary person would have been upset by the publication). Note, that the second element requires that the disclosure must be of private facts about the plaintiff. That was not the case here. (D) is wrong as truthfulness of the story is meaningless. When dealing with a true story concerning private facts, you still may be on the hook for violating the privacy statute.

Question 9 0 / 3 pts Due to an oil rig explosion set off by a disgruntled employee off the coast of Santa Barbara, California, oil spilled into the area and washed up on the local beaches. Frank, a local fishermen has lost his livelihood, because the pollution has ruined the catch. Bill, a local businessman with a hamburger restaurant right on the beachfront, lost his customers, after oil washed ashore onto his property. The best claim(s) are: I. Public nuisance because the harm extends to the public's use of the beach. II. Private nuisance for Frank. III. Private nuisance for Bill. All of the above. I and II. II and III.r I and III.

D is the best answer. There is a claim for public nuisance because the oil has interfered with a right common to the general public in terms of their use of the beach. There is no private nuisance claim for Frank because there is no interference with an interest in his own land that is substantial and unreasonable, since he does not own the beach or the ocean. Bill does have a private nuisance claim, since his business on the beachfront has been adversely impacted by the oil, and the impact is a result of wrongful conduct, creating a substantial and unreasonable interference with his business.

Question 9 2 / 2 pts Donald was a commercial pilot flying commuter flights for Standard Air Lines. Donald's plane crashed due to pilot error and while Donald walked away from the crash, several of the passengers were very seriously injured. What is the standard of care applicable to Donald? The reasonable pilot with Donald's training. The reasonable pilot with Donald's experience. The reasonable pilot of Donald's physical characteristics. The reasonable prudent professional pilot.

D is the best answer because a pilot is a professional held to the standard of care of one who engages in a profession. A is incorrect because the standard of care is not measured by the specific training of Donald. B is incorrect because the standard of care is not based on Donald's experience. C is incorrect because the standard of care is not based on Donald's physical characteristics, which is the standard for a person with a disability.

Question 3 2 / 2 pts Pam bought a new Pear computer. She brought the computer home, turned it on, and then went to sleep. The computer caught fire and destroyed her home office. Pam hired some computer experts, who all agreed that the fire was a result of faulty manufacturing of the computer. Apparently, the metal used in the exterior of that computer to make it look aesthetically pleasing could react with the wires used in the circuit boards, causing a fire. Upon further investigation, it appeared that only Pam's computer was constructed this way. If Pam sues Pear for her emotional distress and to recover the cost of the repair of her home office, the most appropriate cause of action would be: A suit for strict liability in tort, because the computer had a design defect and an economically feasible reasonable alternative design was available. A suit for strict liability in tort, because the computer had a design defect and a reasonable consumer would not expect that the computer would catch on fire. A suit for strict liability in tort, because the computer had a manufacturing defect. A negligence action, provided Pam can show that she had a contract with Pear.

(C) is the correct answer. A manufacturer is strictly liable for a manufacturing defect, where the product is different from and more dangerous than intended design. Clearly the computer was not designed to start a fire and Pam's computer apparently did not comply with the manufacturer's specifications for the computer. (A) and (B) are incorrect since the computer had a manufacturing, not a design defect. (D) is wrong since assuming there is a product defect, a strict liability claim is easier for a plaintiff to prevail than proving negligence against a manufacturer.

Question 12 0 / 2 pts Mother was on an airplane, flying from New York to her home in Hawaii. The airline negligently failed to completely fuel the plane, which was forced to land in a remote area of Hawaii very near a volcano for emergency refueling. The volcano suddenly erupted, injuring Mother and several other passengers. If Mother sued the airlines for her injuries, who will prevail? Mother, because the airline's failure to refuel properly was negligent. Mother, because the airline should not have landed near the volcano. The airline, because Mother's injuries were not foreseeable. The airline, because it satisfied its duty of reasonable care to mother.

A is correct because since the airline negligently failed to fuel the plane properly, that conduct actually and proximately caused Mother's injuries from the volcano. B is incorrect because the landing was necessitated in order to avoid a crash. C is incorrect because the airline's conduct was a proximate cause of Mother's injuries, since injury from an erupting volcano, although sudden, is a foreseeable cause of harm from the failure to fuel a plane. D is incorrect because the airline did not satisfy its duty of care.

Question 7 2 / 2 pts Donna owns a small boutique in an outdoor shopping mall. Patty stepped into the boutique one afternoon to shop for a gift for her daughter's wedding. Patty slipped on some water that had spilled when Donna had arranged a display that included fresh flowers earlier that morning. Patty broke her wrist in the fall. Which of the following are true? Patty is an invitee and Donna owes her a duty to exercise reasonable care in the maintenance of the boutique. Patty is a licensee and Donna owes her duty to inspect the premises, make them safe, and warn about any dangerous conditions. Patty is a licensee, but Donna does not owe any duty to Patty, because the condition of the premises did not pose any risk to Patty. Patty is an anticipated trespasser and so, Donna owes her no duty.

A is correct since Patty is an invitee. An invitee is a person who comes onto property with consent of the landowner; here, Patty is a business invitee, as she is in the boutique shopping for a gift. A landowner has a duty to a business invitee to act as a reasonable person, and this means she must --inspect and discover dangers, --remedy the danger, and --warn if the danger is not obvious. B is incorrect since here, Patty is a business invitee, as she is in the boutique shopping for a gift. A landowner has a duty to a business invitee to act as a reasonable person, and this means she must inspect and discover dangers, remedy the danger, and warn if the danger is not obvious. C is wrong since it states that Donna owed no duty. D is wrong since Patty is not a trespasser.

Question 8 15 / 15 pts Art and Bill were hunting ducks with Clem. Clem went ahead of Art and Bill, and when a flock of ducks flew over Clem, both Art and Bill fired on them. Clem was hit in the eye causing him serious injury. There was no evidence to show whether Art or Bill had fired the particular shot that hit Clem. Clem sued both Art and Bill, claiming one or the other had injured him. The most likely result is: Both Art and Bill are jointly and severally liable to Clem because they both were negligent. Clem will prevail but he must provide evidence regarding apportionment of wrongdoing. Both Art and Bill are jointly liable to Clem only if they encouraged each other to shoot. Neither Art nor Bill will be liable to Clem because Clem cannot prove whose shot harmed him.

A is the best answer based on Summers v. Tice. B is incorrect because the defendants were required to provide evidence to apportion the injury, not the plaintiff. C is incorrect because encouragement would create potential liability, but is not required for joint liability. D is incorrect because both Art and Bill are jointly and severally liable under Summers v. Tice.

Question 14 2 / 2 pts Carnival Company runs an amusement park in the State of Adams. Cynthia decided to take her children to the park. She asked her neighbor if her six-year-old daughter, Monica, could go with them. The neighbor agreed, so Cynthia, her children and Monica went to the park. After they arrived, Cynthia went to buy cotton candy and temporarily lost sight of Monica. Monica was attracted by a ride that had colorful cars spinning at a high rate of speed. The ride was surrounded by a fence, but the ride attendant had left the gate open. Monica walked through the gate, approached the ride, and was struck by one of the cars. She suffered serious injuries. If Monica's representative sues Cynthia, who will prevail? Monica, because Cynthia breached her duty of care. Monica, because Cynthia had a duty to warn Monica. Cynthia, because she had no duty to rescue Monica. Cynthia, because she was not Monica's mother.

A is the best answer because Cynthia assumed a duty of reasonable care when she offered to take Monica to the park. B is incorrect because a duty to warn is generally imposed on possessors of land, and Cynthia was not in possession of the park. C is incorrect because Cynthia will not prevail and also because Cynthia did assume a duty of care here. D is incorrect because Cynthia will not prevail and has a duty even though she is not Monica's mother.

Question 11 2 / 2 pts Sal's friend Fran took Sal to the emergency room for treatment. Fran, in a hurry, left her car with a valet service operated by Dan. When Fran returned several hours later, Dan could not produce the car, which had been stolen. If Fran asserts a claim against Dan based on conversion, will Fran prevail? Yes, because Dan could not produce Fran's car. Yes, unless the car is eventually recovered. No, unless Dan gave the car to the thieves. No, as long as Dan took reasonable precautions.

A is the best answer because a claim for conversion is an intentional interference with possession of property that causes substantial deprivation of the property; the intent required is the intent to take possession of the property. Here, Dan took possession of Fran's car, and the theft interfered with her possession. B is incorrect because Fran is still deprived of the use and possession of the car, even if the car is eventually recovered. D is incorrect because it would be relevant to negligence, but the claim is for conversion which does not involve the reasonableness of Dan's conduct.

Score for this quiz: 14 out of 20 Submitted Nov 1 at 10:28pm This attempt took 19 minutes. Question 1 2 / 2 pts Sal's friend Fran took Sal to Hospital for treatment after his illness. Fran, in a hurry, left her car with a valet service operated by Dan. When Fran returned several hours later, Dan could not produce the car, which had been stolen. If Fran asserts a claim against Dan based on conversion, will Fran prevail? Yes, because Dan could not produce Fran's car. Yes, unless the car is eventually recovered undamaged. No, unless Dan gave the car to them. No, as long as Dan took reasonable precautions

A is the best answer because a claim for conversion is an intentional interference with possession of property that causes substantial deprivation of the property; the intent required is the intent to take possession of the property. Here, Dan took possession of Fran's car, and the theft interfered with her possession. B is incorrect because the deprivation may be substantial even if the car is eventually recovered. D is incorrect because it would be relevant to negligence, but the claim is for conversion which does not involve the reasonableness of Dan's conduct.

Question 9 2 / 2 pts Otto owns a home with a pool in the back yard, and knows that children often come to swim in the pool when Otto is at work. Ted, who is barely two years old, lives next door, and Otto is aware Ted does not know how to swim, so he tries to remember to keep the gate to his back yard closed. Otto thought Ted was probably too young to join the others. Otto thought about locking the gate but kept forgetting to buy a lock. One afternoon when other children were swimming in Otto's pool, Ted wandered in and somehow ended up in the pool and almost drowned, suffering brain damage. Did Otto owe a duty to Ted? Yes, because the pool was an attractive nuisance. Yes, because Otto owed a duty to have a lifeguard. No, because Ted was a trespasser. No, because Ted was not an invitee.

A is the best answer because an attractive nuisance requires that the owner know the area is used by children, that the condition poses an unreasonable risk of serious injury or death, that a child not realize the danger due to his youth, and that the benefit to the owner of maintaining the condition is light weighed against the risk to children, and that the owner fails to use reasonable care to eliminate the danger. B is incorrect because there is generally no duty to have a lifeguard, though Otto was obliged to take measures to make the property safe, such as a lock on the gate. C is incorrect because Otto owed Ted a duty under the attractive nuisance doctrine. D is incorrect because Otto owed Ted a duty even though Ted was not an invitee.

Question 12 2 / 2 pts Alan, a football player, got onto the school bus to travel back to the school after a high school football game. Alan saw Evan, a team member, and started a fight with him on the bus over a girl. After Alan threw a punch at Evan, Evan kicked Alan, injuring Alan's leg badly. When the bus arrived back at the school, the bus driver locked the doors of the bus and refused to let Alan off until he apologized to Evan for starting the fight. Does Alan have a claim against the bus driver for false imprisonment? Yes, because Alan was detained in a confined area. Yes, if the confinement was for a substantial period of time. No, because the bus driver had a privilege to detain Alan. No, if Alan violated the transit rules.

A is the best answer because false imprisonment is the intentional conduct causing a person to be confined in a closed space with no reasonable means of escape, where the plaintiff is aware of the confinement. B is incorrect because confinement for a brief period of time will suffice. C is incorrect because there is no shopkeeper's privilege here and no other privilege is apparent. D is irrelevant to the analysis of the problem.

Question 8 0 / 2 pts Sparks from a train engine start a fire, which merges with a wildfire already burning, started by campers. Together both fires cause damages to the property of Patsy destroying her rural home and killing her horses. At trial, expert testimony indicates that either fire alone would have been sufficient to cause the harm. As to causation, the court should find: Both fires were causes in fact of the harm to Patsy's property. Both fires were but for causes of the harm to Patsy's property. The fire sparked by the train is not a cause in fact of the harm, because the wildfire would have destroyed Patsy's property anyway. The wildfire is not a cause in fact of the harm, because the fire sparked by the train would have caused the harm alone.

A is the best answer because where several causes concur to cause harm, any defendant's cause is not the but for cause of the harm, but each is a cause in fact of the harm. B is incorrect because where several causes concur to cause harm, any defendant's cause is not the but for cause of the harm. C and D are incorrect because each fire is a cause in fact of the harm.

Question 5 0 / 2 pts Jackie West, a big time wrestler turned actor, later turned governor, alleged that he was defamed in a book that indicated he punched out a Navy Seal in a barroom brawl. Jackie sued the book's publisher for defamation. Jackie testified that it never happened, and there was no evidence to the contrary. To prevail, Jackie must show that: I. The book publisher acted with malice. II. The book publisher acted with knowledge of falsity. III. The statement harmed Jackie's reputation. All of the above. I and II only. I and III only. II and III only.

A is the best answer. A prima facie case of defamation for a public figure requires a showing of malice, falsity and harm to reputation, under NY Times v. Sullivan. Therefore, B, C and D are incorrect

Question 4 2 / 2 pts Joe was running through the parking lot with his friend Frank. The two teenaged boys both played football and constantly punched at each other, on and off the field. Frank punched at Joe, and Joe told Frank to "knock it off, now!" Frank punched Joe again, causing him to trip and fall. Joe fell to the ground. He hit his head on a parking curb and cracked his skull. Is Frank liable for battery? No, because the two boys often punched around. No, because this was a freak accident, which Frank did not intend. Yes, because his punch caused harm to Joe. Yes, but only for the harm to Joe's arm, not for the skull injury.

C is the best answer based on the intentional harmful contact by Frank, which was not consented to by Joe. A is incorrect because the two boys punching around doesn't give license, and Joe expressly told Frank to stop hitting him, right before Frank threw the offensive punch. B is incorrect, because Frank intended to make contact with Joe, which is all that is required. D is incorrect, because the harm to Joe's skull was a direct result of the contact initiated by Frank.

Question 12 2 / 2 pts Art and Bill were involved in an argument over Gemma, a woman in their college class that both Art and Bill liked. Art said, "If you don't stay away from her, I'm going to kick you in the behind." Bill picked up a baseball near where he was standing and threw it at Art, shouting, "No one tells me what to do! I'll show you!" Art ducked and the baseball did not hit him, and instead hit Carla who happened to be walking by. Carla fell to the ground and screamed and suffered a bruise on the side of her head. What intentional torts have been committed? Both Art and Bill committed assault. Bill committed assault only. Bill committed assault and battery. None.

C is the best answer because Bill committed assault when he threw the ball at Art while threatening Art, but Art's words alone will not constitute assault without any further action on his part; Bill committed battery as well although against Carla, who was hit by the ball, based on transferred intent, which provides that the intent to hit Art with the baseball transfers to Carla. A is incorrect because Art did not commit assault. B is incorrect because Bill also committed battery. D is incorrect because Bill committed both assault and battery.

Question 10 2 / 2 pts Mart Corp. manufactures display racks used in retail stores, including Smart Mart. Paula was shopping at a Smart Mart facility when she cut her hand on a sharp metal part of a display rack that did not get properly coated in the manufacturing process. If Paula sues Mart Corp. and Smart Mart, seeking recovery of damages for pain and suffering and medical expenses, Paula can recover: Both damages for pain and suffering and medical expenses as general damages. Damages for pain and suffering as special damages and damages for medical expenses as general damages. Damages for pain and suffering as general damages and damages for medical expenses as special damages.

C is the best answer because pain and suffering is treated as a general damage and medical expense is treated as a special damage. For that reason, A, B and D are incorrect.

Question 10 2 / 2 pts As a result of a childhood disease, Patty had very thin and fragile bones. When Don accidentally rear-ended her car, both of Patty's arms and legs broke. The medical testimony showed that this would not have happened to a person without Patty's particular condition. Don argued at trial that he should not be liable for the expenses incurred as a result of Patty's medical condition, because they were not foreseeable. Patty argued that Donald should be liable for all of her injuries. How should the court rule? For Donald, because he was not the cause in fact of Patty's harm. For Donald, because the degree of Patty's injuries was not foreseeable. For Patty, because Donald is responsible even for unforeseeable injuries. For Patty, unless Donald proves that her injuries were uncommon and unforeseeable.

C is the best answer because the "eggshell skull plaintiff" rule provides that a tort defendant takes his plaintiff as he finds her and is responsible for the full extent of the injuries caused by the defendant's negligence, even if the injuries are unforeseeable or uncommon and would have not been suffered by others, so Donald is liable for all of her injuries. A is incorrect because Donald was the cause in fact of her injuries. B is incorrect because Donald is liable even if her injuries were unforeseeable. D is incorrect because even if Donald does prove that her injuries were uncommon and unforeseeable, and would not have been suffered by others, he remains liable under the eggshell skull plaintiff rule. Quiz Score: 14 out of 20

Question 14 2 / 2 pts Walt published a book about the employees of Diamond's Department Store, a high-end, exclusive boutique with twenty world-wide locations, known for in-store modeling of designer evening wear. In his book, Walt stated that all of Diamond's models are "high end call girls," and some of the saleswomen are "cheap whores". Walt repeated these statements on his book tour on radio and television. Diamond employed nine models, and 300 saleswomen, all of whom filed suit against Walt for defamation. In fact none of the models or sales women were prostitutes. Which of the following is true? The statements are statements of opinion. All of the statements are actionable. The statements regarding the saleswomen are not actionable. The statements regarding the models are not actionable.

C is the best answer because the statements about the saleswomen refer to a group and the group is large so that the defamatory implication would not arise as to every member of the large group. A is incorrect because the statements are statements of fact, to wit, that the women are prostitutes. B is incorrect because the statements regarding the saleswomen are not actionable. D is incorrect because the statements regarding the relatively small group of models that all of the models are call girls would be actionable.

Question 13 2 / 2 pts Fritz owned a 30-acre parcel of undeveloped land in a suburb. The land was located between a new high school and a housing subdivision. Many students took a short cut through Fritz's property to reach the school. One Saturday morning, Fritz had a construction crew dig a 10-foot-deep hole for the foundation of a new building. Because Fritz knew that students sometimes walked across his land, he posted a warning sign near the hole. Elmer, a high school senior, walked across Fritz's property to get to school the next day. He almost ran into the sign, which was posted directly in front of the hole. Still, after walking past the sign, Elmer was not paying attention and fell into the hole, breaking both of his legs. If Elmer sues Fritz for negligence, the likely result will be: Elmer will prevail because Fritz failed to satisfy his duty to warn. Elmer will prevail because Fritz did not own him a duty to warn. Fritz will prevail because he posted a warning. Fritz will prevail because he owes no duty to Elmer.

C is the best answer. A landowner owes a duty to known trespassers to make safe or warn regarding artificial hidden highly dangerous conditions, such as the hole. Fritz performed this duty by posting the sign. A is incorrect because Fritz satisfied the duty to warn by posting the sign. B is incorrect because Fritz owed a duty to warn of the particular hazard. D is incorrect because Fritz owed a duty to Elmer.

Question 7 0 / 15 pts Six year old Claire was playing outside in the cul-de-sac when Ted, a teenage neighbor, came driving toward her, swerving a bit to try to scare her. As Claire ran to get out of the way, Ted's car struck her, injuring her badly. After the accident, Claire's mother Mary came outside, looking for Claire and saw what happened. Mary instantly became hysterical. Claire was hospitalized and Mary required treatment for mental distress. Claire sued Ted for negligence. Mary sued for negligent infliction of emotional distress. Who is likely to prevail under the majority view? Both Mary and Claire will prevail. Mary will prevail, but Claire assumed the risk of harm. Claire will prevail, but Mary cannot recover. Ted will prevail against both Claire and Mary.

C is the correct answer since on a bystander theory of liability, Mary cannot prevail unless she contemporaneously observed the negligence directed towards her daughter, B is incorrect because Mary cannot prevail since she did not observe the accident and Claire a five year old, did not expressly assume the risk of playing in the cul-de-sac with an understanding of the risks. D is incorrect because Claire will be able to prevail against Ted, whose conduct is negligent in causing the harm to Claire.

Question 3 2 / 2 pts Alex and Peter were both 7 years of age. One day when they were playing together, Alex showed Peter his new air rifle. Peter had purchased the rifle from Store with money he recently received for his birthday. While demonstrating the air rifle to Peter, Alex accidently shot Peter with it, severely injuring Peter's eye. If Peter asserts a negligence claim against Store and he is successful in his claim, it will be because a jury finds that: The air rifle is defective. Peter suffered a severe eye injury. It was unreasonable for Store to sell the air rifle to Alex. Alex did not know how to use an air rifle.

C is the correct answer. It is unreasonable (negligent) for Store to sell a product as dangerous as an air rifle to a 7-year old child since the risk that he would use it to shoot another person is foreseeable. A is incorrect since there are no facts to indicate that the rifle had a defect. B is wrong since a personal injury alone is not sufficient to establish the elements of negligence. D is wrong because the fact that Alex, a seven year old child, did not know how to responsibly use an air rifle, while foreseeable, was not the cause of Peter's eye injury.

Question 4 2 / 2 pts Art was seriously injured when a metal object fell from the sky into his backyard where he was playing with his dog. It was determined that the object was a part of an AeroFlight airplane that had not been properly installed by the manufacturer, causing it to come loose in flight. Art was hospitalized and Doctor performed surgery negligently, resulting in Art going into a coma for several weeks. Art sues AeroFlight and Doctor. Which of the following are correct under the modern view? Art can recover only from AeroFlight based on the defective product. Art can recover only from the Doctor based on the Doctor's fault. AeroFlight and Doctor are jointly and severally liable for all harm caused to Art. Doctor and AeroFlight are each liable based on comparative fault.

D is the best answer based on modern law which is trending away from traditional joint and several liability (answer C) and toward comparative liability in most jurisdictions. Both Doctor and AeroFlight will be liable for the harm to Art, and their liability will be proportionate to the harm caused, with Doctor liable for the amount of harm caused by the worsening of the condition, and AeroFlight liable for all harm. A is incorrect because Art is not limited to recovery from AeroFlight. B is incorrect because Art is not limited to recovery only from Doctor. C is incorrect because it does not represent the modern view.

Question 9 0 / 2 pts Six months after Dr. Dan performed surgery on her, Peg had x-rays taken by another doctor. The x-ray showed a surgical instrument inside Peg's abdomen. Dr. Dan was the only person who had ever performed surgery on Peg. Peg subsequently brought a negligence claim against Dr. Dan, alleging that Dr. Dan had negligently left the surgical instrument inside her while operating on her. If an expert testifies that surgeons do not usually leave instruments inside a patient's body unless they are acting unreasonably, can Peg rely on res ipsa loquitur in her claim against Dr. Dan? No, because the doctrine of res ipsa loquitur is not applicable to a claim for negligence against doctors. No, because a physician is not negligent if they accidently leave a surgical instrument inside a patient's body. Yes, because a surgeon has a duty not to leave surgical instruments inside a patient's body. Yes, because Dr. Dan was the only person who had ever performed surgery on Peg.

D is correct. Under the doctrine of res ipsa loquitur, an inference that the defendant acted unreasonably can be drawn from the facts that the injury involved was one which does not usually occur without unreasonable conduct and that the defendant was the only person whose conduct could have caused the injury (i.e., the defendant was in exclusive control of the circumstances). If an expert witness testifies that surgeons do not usually leave instruments inside a patient unless they are acting unreasonably, Peg can rely on the inference established by res ipsa loquitur if she can show that Dr. Dan was the only person who could have left the instrument inside her. Since Dr. Dan was the only person who had ever performed surgery on Peg, he is the only person who could have left the instrument inside her. A is incorrect because it is based on an inaccurate statement of law; there are many negligence cases against doctors in which the plaintiff is permitted to rely on res ipsa loquitur. Because of its lack of specialized knowledge, a jury is not competent to decide whether the particular result of a professional's conduct is one which would not usually occur without negligence. Once a jury has heard testimony to that effect from an expert witness, however, the jury may base an inference of negligence on that expert's testimony. For this reason, B is incorrect. C is incorrect because res ipsa loquitur is not dependent on the existence of any "duty," but rather on evidence which justifies the circumstantial inference that a particular defendant acted unreasonably.

Question 4 15 / 15 pts David was in the act of siphoning gas from Neighbor's car in Neighbor's garage and without Neighbor's consent, when the gasoline exploded and a fire followed. Rescuer, seeing the fire, grabbed a fire extinguisher from his car and put out the fire, saving David's life and the neighbor's car and garage. In doing so, Rescuer was badly burned. If Rescuer asserts a claim against David for personal injuries, Rescuer will: Prevail, because he saved David's life. Prevail, because David was at fault in causing the fire. Not prevail, because David did not breach a duty of care to Rescuer. Not prevail, because Rescuer was not a foreseeable plaintiff.

The correct answer is B. It is foreseeable that where a defendant's wrongful conduct places him in a position of peril, a rescuer may suffer injuries while reasonably attempting to aid him, i.e., "danger invites rescue." (D) is incorrect because the rescuer was a foreseeable plaintiff, given that danger invites rescue. (C) is wrong because David had a duty of care to act as a reasonably prudent person and a rescuer is a foreseeable plaintiff. (A) is not a reason for imposing liability on David because it is not relevant to David's negligence.

Question 7 3 / 3 pts Sal's friend Fran took Sal to Hospital for treatment after his illness. Fran, in a hurry, left her car with a valet service operated by Dan. When Fran returned several hours later, Dan could not produce the car, which had been stolen. If Fran asserts a claim against Dan based on conversion, will Fran prevail? Yes, because Dan could not produce Fran's car. Yes, unless the car is eventually recovered undamaged. No, unless Dan gave the car to them. No, as long as Dan took reasonable precautions.

Answer: A is the best answer because a claim for conversion is an intentional interference with possession of property that causes substantial deprivation of the property; the intent required is the intent to take possession of the property. Here, Dan took possession of Fran's car, and the theft interfered with her possession. B is incorrect because the deprivation may be substantial even if the car is eventually recovered. D is incorrect because it would be relevant to negligence, but the claim is for conversion which does not involve the reasonableness of Dan's conduct.

Question 6 0 pts Patricia lived next door to Cindy. Cindy had a pool in her yard. Cindy told Patricia that Patricia could come over any time, even if Cindy was not home, to use the pool, but asked her to bring her own towel, if Patricia didn't mind. One day when Cindy was not home, Patricia went into Cindy's back yard to use the pool. It was a hot day and Patricia jumped into the pool before she realized she had forgotten to bring a towel. Patricia swam for a while, then got out of the pool. She saw a basket filled with fresh towels on Cindy's patio, and took one of them, thinking Cindy would not miss it. Patricia threw the towel into Patricia's laundry when she got back to her own house. She washed the towel and brought it back to Cindy's house the next time she went swimming and replaced it in the basket. Cindy never noticed the towel had been missing. The next time Patricia went swimming she remembered to bring her own towel. Patricia is liable for: I. Trespass to land. II. Trespass to chattel. III. Conversion. Group of answer choices All of the above. I and II only. II only. II and III only.

C is the best answer. There is no trespass to land because Cindy gave consent for Patricia to use the pool. Therefore (I) is not applicable. There is a trespass to chattel, because Patricia intentionally deprived Cindy of the use of the towel for a period of time. Therefore (II) is applicable. There is no conversion, because the deprivation was not permanent and there was no harm to the towel. Therefore III is not applicable.

Question 4 15 / 15 pts One morning, Peter brought his new car to the dealership for servicing. When he came to pick his car up at the end of the day, it was no longer at the dealership. Peter contacted the police and it was later discovered that the dealer had sold Peter's car to Abe. Abe did not know that the car sold to him actually belonged to someone other than the dealership. If Peter asserts a claim based on conversion against the dealership and Abe, Peter likely will prevail against: The dealership, but not Abe, since Abe was a good faith purchaser Abe, but not the dealership, because the dealership no longer has possession of the car Abe, but not the dealership, because the dealership had lawful possession of the car Both the dealership and Abe, since they both exercised dominion over the car

D is correct. A is wrong since a bona fide purchaser of the (stolen) car is a converter, even if there is no way that he could have known that the car was stolen; B and C are wrong since the dealership interfered with Peter's rights of ownership of the car.

Question 4 3 / 3 pts Tim, an art thief, took a small painting from a gallery run by Anna in a popular shopping center. As Tim left the gallery with the painting, an alarm sounded. Tim ran down the street, and Anna followed him, tackling him after he slowed down about 50 yards down the street. Thankfully Anna was able to recover the painting which was not damaged. Which of the following are correct? Anna is liable for battery of Tim. Tim is liable for conversion of the painting. Anna used excessive force against Tim. Anna's use of force to recover the painting was reasonable.

D is the best answer. Under the circumstances, Anna has a privilege to recapture her chattel in fresh pursuit if she acts without unreasonable delay and uses only reasonable force. A is incorrect because Anna is permitted to use reasonable force, which it appears she did here, as Tim was running away with her gallery's property. C is incorrect because the force she used was excessive under the circumstances. B is incorrect because there is deprivation of only short duration and no harm to the painting.

Question 7 15 / 15 pts Dave owned an electronics store in a seedy part of the city. To discourage thefts, Dave had a high barbed wire fence installed around the store. Dave had the fence electrified, so that a lethal voltage of electricity passed through the fence during the hours that the store was closed for business. Late one evening, Percy attempted to climb over the fence, planning to steal some of the computer equipment. Percy suffered a severe injury when he was electrocuted by the current passing through the fence around Dave's store. If Percy asserts a claim against Dave to recover damages for his injuries, Percy will:! prevail, because Dave used unreasonable force to protect his property not prevail, because Dave was not present on the property when Percy's injury occurred not prevail, because Percy was trespassing on Dave's property. not prevail, because Percy intended to commit a crime.

A is correct since a landowner does not have the right to use deadly force to protect his property. B is wrong because whether or not the landowner is present is irrelevant (analogous to the spring gun case); C and D are wrong since Percy's status as a trespasser or would-be criminal does not give Dave the right to use lethal force solely in defense of property.

Question 19 0 / 15 pts Dillon had recently been released from a mental institution, and was living in a halfway house. He had a long history of mental illness and had been in and out of institutions since he was an adolescent. Although Dillon would improve for a while, he was subject to recurrent hallucinations and delusions involving a belief that people, animals, and alien beings from another planet were about to attack him. Pam was walking along a quiet residential street when she noticed that a person was walking up the street on the sidewalk towards her, but she did not know Dillon. As Pam approached, she smiled politely at Dillon. Dillon pulled a soft drink bottle from his shopping bag and hurled it at Pam, striking her in the head and causing her injury. Pam filed a battery action against Dillon. Which of the following is correct? Dillon acted in self-defense after Pam threatened him Pam impliedly consented to contact with Dillon by smiling at him Dillon is liable if he had the required intent Dillon's conduct was privileged due to necessity

C is the best answer, because battery requires intentional action causing harmful or offensive contact. A is incorrect because Pam's smile was not threatening, and a person would not expect the use of force exhibited by Dillon under the circumstances. B is incorrect because a smile does not communicate implied consent to being hit with a bottle. D is incorrect because there is no privilege covering this scenario, as there was no necessity for his conduct.

Question 11 0 pts Homer was thinking of hiring Ed as a professor for his new law school. Ed had published several scholarly articles on the subject of the death penalty, so Homer contacted Dean who had also published articles on the subject of the death penalty. Homer asked if Dean was familiar with Ed's work and Dean said that one of Ed's law review articles contained plagiarized content. This in fact was not true, and Dean was aware that it was not true. Dean's statement to Homer is: Group of answer choices Libel per se. Libel per quod. Slander per se. Slander per quod.

C is the best answer, because the statement is oral, and therefore, slander rather than libel, and per se, because it concerns Ed's professional conduct. A is incorrect because the statement is not written or recorded in a manner that provides longevity so therefore it is not libel. B and D are both incorrect because a per quod statement requires external proof because the statement's meaning is not apparent on its face.

Question 10 2 / 2 pts Patricia is a blind person who makes her living selling sodas and snacks from a pushcart outside of the federal courthouse. She did not realize that her cart was leaking causing slimy water to pool on the sidewalk. Arthur, an attorney, came out of the courthouse in a big hurry to get to a meeting and slipped in the water, causing Arthur to fall and fracture his arm. What is the standard of care applicable to Patricia? A reasonable woman of like physical disability. A reasonable person under emergency circumstances. A reasonably prudent professional person. A reasonable person in the same locality.

A is the best answer because physical disabilities are taken into consideration when evaluating the reasonable person standard of care. B is incorrect because Patricia was not involved in an emergency situation here when her conduct resulted in harm to Arthur. C is incorrect because Patricia is not in a professional position. D is incorrect as the locality rule measures the negligence of medical professionals in connection with medical malpractice by comparing them to others in the same locality or community and is not applicable in this context. Quiz Score: 16 out of 20

Question 5 15 / 15 pts Dever drove his car into an intersection and collided with a fire engine that had entered the intersection from Dever's right. The accident was caused by negligence on Dever's part. As a result of the accident, the fire engine was delayed in reaching Peter's house, which was entirely consumed by fire. Peter's house was located about 10 blocks from the scene of the accident. If Peter asserts a claim against Dever, what will Peter recover: That part of his loss that would have been prevented if the collision with the firetruck had not occurred. The value of his house before the fire. Nothing, because Dever was not the actual cause of Peter's harm. Nothing, because Dever's conduct did not breach a duty of care to Peter.

(A) is correct because it recognizes that Dever breached a duty of due care owed to Peter, and that Dever will be liable for that part of Peter's damages that was actually and proximately caused by Dever's breach of duty. In negligently causing the collision with the fire engine, it was foreseeable that Dever's actions could cause injury to someone else (e.g., a local resident who the fire engine would be unable to reach because of the accident). Peter, whose house was located only 10 blocks from the scene of the accident, falls within the foreseeable zone of danger. It follows that (D) is incorrect because Dever has a duty of care to drive as a reasonably prudent driver and Dever breached that duty. (C) is incorrect because, Dever was a substantial factor causing Peter's harm and therefore was an actual cause of all or part of the damage suffered by Peter. (B) is incorrect because it assumes, without a factual basis, that Dever's negligence caused the entire loss.

Score for this quiz: 26 out of 30 Submitted Dec 6 at 10:58pm This attempt took 26 minutes. Question 1 2 / 2 pts RadBikes is a manufacturer of dirt bikes. Bikeco, an independent retailer, sells a number of dirt bikes from RadBikes' line. Patty purchased a RadBikes dirt bike from Bikeco. She took her bike out for a spin, and as soon as she started riding, the wheels fell off. Patty was thrown from the bike and she landed on the pavement, suffering serious injuries. Accident investigators determined two bolts were inadvertently missing from the bike, which led to the accident. What will be the likely outcome if Patty sues Radbikes? Patty will recover because the accident was the result of a manufacturing defect. Patty will not recover, unless she can prove RadBikes was negligent. Patty will recover because the accident was the result of design defect. Patty will not recover because she had no contract with RadBikes.

(A) is the correct answer. A manufacturer is strictly liable for a manufacturing defect, where the product is different from and more dangerous than intended design. We can assume that the intended design for the dirt bike was for it to contain the two bolts which would have prevented the wheels from falling off. (B) is incorrect because a purchaser need not prove negligence to recover for a personal injury caused by a dangerously defective product. (C) is incorrect because a manufacturing defect, not a design defect, caused the injury. (D) is incorrect because privity of contract between a manufacturer and a purchaser is no longer required.

Question 6 2 / 2 pts While shopping at Store, Peter grabs an electronic toothbrush and puts it into his shopping basket. As he does so, the toothbrush malfunctions and explodes causing him serious injury. Pater sues Store and Oral-F, the manufacturer of the toothbrush. Who will Peter likely prevail against? Oral-F and Store, but store may seek indemnity from Oral-F. Oral-F is liable but Store is not liable to Peter. Store is liable, but Oral-F is not liable to Peter. Neither Oral-F nor Store are liable to Peter.

(A) is the correct answer. Oral-F is liable since it appears that the toothbrush was defectively manufactured. It does not matter that it did not sell the toothbrush to Peter - see Restatement Second § 402A(2)(b). --In addition, Store, the retailer, may seek indemnity from Oral-F because a retailer selling a defective injury-causing product will get indemnity from those further up the distribution chain. --Store is liable even though there was probably not yet any contract of sale between Peter and Store. --One who sells any product in a defective condition unreasonably dangerous to the user or consumer or to his property is subject to liability for physical harm thereby caused to the ultimate user or consumer, or to his property, --if the seller is engaged in the business of selling such a product, and it is expected to and does reach the user or consumer without substantial change in the condition in which it is sold. Since both defendants are liable, answers B, C and D are incorrect.

Question 19 15 / 15 pts Donna was in her car, speeding down the street since she was late to work. In her haste, Donna failed to see a pedestrian, Paul, who stepped into the crosswalk when the signal changed in his favor. To avoid being hit, Paul jumped out of the way of Donna's car. Still in the crosswalk, Paul was hit by Hank, who was speeding down the street, and sending a text message to his girlfriend. If Paul chooses to sue both Hank and Donna, what is the likely outcome? Both Hank and Donna are liable to Paul. Only Hank is liable to Paul because Hank's negligence was a superseding cause of Paul's harm. Only Donna is liable to Paul, because only she is an actual cause of Paul's harm. Neither Donna nor Hank will be liable to Paul, because Paul was negligent.

A is correct because Donna was negligent in moving her car toward Paul while he was in the crosswalk with the light in his favor and Hank was negligent in speeding while sending a text message. B is incorrect because Hank is not a superseding cause of Paul's harm, since the negligence of others is foreseeable and generally is not considered a superseding cause of harm. C is incorrect because Hank is also an actual cause of Paul's harm. D is incorrect because there is no indication Paul was negligent, since he used the crosswalk when the light was in his favor and was still in the crosswalk when he was hit by Hank.

Question 10 2 / 2 pts Auto Company operated a service department. Peter wanted to ask the service manager, whether Auto Company would check the muffler on his car. Peter parked on the street near the service department and he walked through the entrance, but he immediately was struck on the back of the head and neck by the large overhead door which was descending. The blow knocked Peter unconscious and caused permanent damage. Peter did not know how the door was raised and lowered. However, the overhead door was operated by the use of either of two switches in the building. One switch was located in the office of the service manager and the other was located near the door in the service work area. On this occasion, no one was in the service work area except one Auto Company mechanic. The service manager, who had been in his office, and the mechanic denied having touched a switch that would have lowered the door. Subsequent investigation showed, however, that the switches were working properly and that all of the mechanisms for moving the door were in good working order. If Peter asserts a claim based on negligence against Auto Company, Peter probably will: Recover, because an employee of the Auto Company was negligent. Recover, because he did not act negligently. Not recover, because he did not know which of the two employees was negligent. Not recover, because Peter could not show that the accident would not have happened unless someone was negligent.

A is correct because res ipsa loquitur applies. When the facts strongly indicate that the plaintiff's injuries resulted from the defendant's negligence, the trier of fact may be permitted to infer the defendant's liability. Res ipsa loquitur requires the plaintiff to show: (i) the accident causing his injury is of the type that would not normally occur unless someone was negligent; (ii) the negligence was attributable to the defendant; and (iii) the plaintiff was free from negligence. All of these elements are satisfied from the facts of this case. Because the switches and the doors were found to be in good working order, the injury can reasonably be inferred to be attributable to one of Auto Company's employees, because they had access to the switches. B is wrong because the lack of negligence of the plaintiff is but one factor to proceed on a res ipsa loquitur theory. C is wrong because Peter can proceed on res ipsa loquitur because he can identify which persons had control over the instrumentality. D is wrong because on these facts, he can prove that the accident would only happen if someone was negligent. Quiz Score: 14 out of 20

Question 8 2 / 2 pts As Pam is walking down the street she is suddenly struck in the head by a bucket and injured. At trial, Pam is able to show that the bucket fell from the window of a shop in the building. The shop was owned by Darla. Pam demonstrated that buckets do not normally fall from shop windows without negligence, but had no direct evidence of what happened to cause the bucket to fall, other than the fact that Darla's gardening shop sold similar buckets. Which of the following is correct? I. The evidence creates an inference that Darla was negligent. II. Darla may attempt to rebut the evidence by showing the bucket did not come from her shop because it did not have a label she always uses on the bottom of the bucket Both I and II. Only I. Only II. Neither I nor II.

A is correct because res ipsa loquitur requires that there is no direct evidence of defendant's conduct; the harm ordinarily does not occur in the absence of negligence; exclusive control of the instrumentality' by defendant, and that the plaintiff was not negligent, therefore an inference is raised that Darla was negligent. In addition, Darla may rebut the inference by showing she was not negligent or the bucket did not come from her shop, assuming such evidence is available. Accordingly, B, C and D are incorrect.

Question 17 0 / 15 pts A businessman was staying in a suite in a luxury hotel in Las Vegas. The hotel had installed defective deadbolts on the doors of their suites which made the deadbolts easy to open when they were in the locked position. Several patrons had complained about the deadbolts for several weeks but the Hotel had not scheduled a repairman to replace the locks by the time of the businessman's stay. Three patrons had alerted the hotel that they had been robbed during the night, and suspected the thief gained access due to the defective deadbolt. One night, while the businessman was sleeping, a thief was able to unlock the deadbolt in the businessman's room and steal his wallet. If the businessman brings an action against the hotel for damages for the loss of his wallet, the businessman will: Prevail, because the hotel did not exercise reasonable care comparable to other hotels in the industry in timely replacing the defective deadbolt. Prevail, because the hotel has a duty to insure the absolute safety of all its guests. Not prevail, because the thief's action was a superseding cause of the harm. Not prevail, because the hotel did not manufacture the deadbolt.

A is correct since the hotel has a special relationship with its guests and a duty of care comparable to other similar hotels in the industry. B is incorrect because the hotel is not the insurer of its guests. C is wrong because it is foreseeable that a crime of theft can occur if a hotel deadbolt is defective. D is wrong since it is irrelevant since the hotel was negligent and responsible for the harm.

Question 4 2 / 2 pts After Dennis' wife left him, he was distraught. His friend Paul invited Dennis to stay at Paul's house for a while. While Dennis was staying with his best friend Paul, he became more and more depressed. Dennis decided to kill himself. Paul came home from work to find Dennis lying on the floor dead, in a pool of blood. Paul immediately fainted. For months he required psychiatric treatment and medication to sleep, as he suffered from anxiety and nightmares. Paul eventually felt no choice but to sell the house. Paul sued Dennis' estate for intentional infliction of emotional distress. Which of the following is most accurate? Paul should prevail because Dennis' conduct was extreme and outrageous. Paul should prevail only if Dennis' conduct was intended to cause distress to Paul. Dennis' estate should prevail because he acted reasonably under the circumstances. Dennis' estate should prevail because Paul consented to have Dennis stay in his house.

A is the best answer because Dennis acted at least recklessly as to causing emotional distress to Paul, the conduct was extreme and the distress caused was severe,. B is incorrect because Dennis' conduct can create liability even if it was reckless, which is likely the case here. C is incorrect because this is not the test for IIED. D is incorrect because Paul's consent was only to let Dennis stay at his house, not to kill himself in the house.

Score for this quiz: 20 out of 20 Submitted Nov 8 at 9:44pm This attempt took 19 minutes. Question 1 2 / 2 pts Walt published a book about the employees of Diamond's Department Store, a high-end, exclusive boutique with twenty world-wide locations, known for in-store modeling of designer evening wear. In his book, Walt stated that all of Diamond's models are "high end call girls," and some of the saleswomen are "cheap whores". Walt repeated these statements on his book tour on radio and television. Diamond employed nine models, and 300 saleswomen, all of whom filed suit against Walt for defamation. In fact none of the models or sales women were prostitutes. Which of the following are true? Plaintiffs also have a claim for false light against Walt. The statements made in the book are slander per se. The plaintiffs must prove that Walt knew the statements were false. Walt's statements fall within the absolute privilege.

A is the best answer because false light is an invasion of privacy based on plaintiff being placed before the public in a false light that would be highly offensive to a reasonable person. B is incorrect because the statements in the book are written and would be libel. C is incorrect because plaintiffs are not public figures so they will not have to prove knowledge of falsity. D is incorrect because the absolute privilege applies only in judicial or legislative proceedings, to husband and wife, with consent, and to government officials.

Question 6 2 / 2 pts Plaintiff Patrick purchased a computer from Digital Retail, a retailer of all things electronic. The computer was manufactured by Campus Computers. Patrick told Sam, the sales representative at Digital Retail, that he intended to use the computer to take an important examination, so it had to be highly reliable. Sam assured Patrick that the model he purchased should be ideal. Sam had forgotten a consumer recall on the model Patrick was purchasing. Unfortunately, during the exam, the computer overheated, started smoking and then caught fire during the exam as a result of insufficient air ventilation within the laptop, which was a problem well known by Campus which Sam had failed to remember when selling the computer to Patrick. Patrick tried to move the computer away from another student taking the exam and was badly burned. What claims can Patrick viably assert against Digital Retail? I. Negligence II. Breach of the implied warranty of fitness for a particular purpose III. Strict products liability All of the above I and II only. II and III only. I and III only.

A is the best answer because there is a claim for negligence based on Sam "forgetting about the recall". There is a claim for breach of warranty of fitness for the particular use Patrick advised Sam of at the time of purchase. There is a viable claim for strict product liability against a retailer based on defect, here a design defect, given that the product would not meet the consumer expectation test. B, C and D are incorrect for the reasons indicated.

Question 8 2 / 2 pts Donald left his key in the ignition of his car while he ran into a drugstore to pick up his prescription. Todd, a 17 year old delinquent teenager who happened to walk by and see the keys in the ignition, stole the car and went on a joy ride. However, Todd was not a very good driver, and he sideswiped another car that belonged to Paul. A criminal statute imposes a fine on drivers who leave their keys in their cars. Paul sued Donald, alleging negligence based on the violation of the criminal statute. The court has determined that the purpose of the statute was to protect the public from property damage caused by the operation of stolen cars. Which of the following are true? Donald will be liable under the doctrine of negligence per se assuming Paul is in the class of persons intended to be protected. Donald will be liable assuming the court applies the doctrine of res ipsa loquitur to determine that the harm would not occur without negligence. Donald will not be liable under the doctrine of negligence per se assuming that Todd had a valid driver's license. Donald will not be liable because Paul was not in the class of persons intended to be protected by the statute.

A is the best answer. Negligence per se requires that the statute was designed to protect against the type of harm suffered by the plaintiff and that the plaintiff was in the class of persons intended to be protected by the statute, and the violation of the statute is unexcused. B is incorrect because the doctrine of res ipsa loquitur does not apply here. Res ipsa loquitur applies where the precise cause of harm is unknown requires 1) the harm causing event was probably due to negligence; 2) exclusive control of the instrumentality; and 3) absence of voluntary action by the plaintiff. C is incorrect. Whether Todd has a valid license is not relevant to a determination of whether Donald is liable for negligence based on the violation of the statute; that turns on the elements of negligence per se. Negligence per se requires that the statute was designed to protect against the type of harm suffered by the plaintiff and that the plaintiff was in the class of persons intended to be protected by the statute, and the violation of the statute is unexcused. D is incorrect factually. Negligence per se requires that the statute was designed to protect against the type of harm suffered by the plaintiff and that the plaintiff was in the class of persons intended to be protected by the statute, and the violation of the statute is unexcused. Based on the court's finding regarding the purpose of the statute, Paul was in the class of persons intended to be protected.

Score for this quiz: 30 out of 30 Submitted Dec 13 at 11:40pm This attempt took 16 minutes. Question 1 2 / 2 pts Alice purchased a new leaf blower manufactured by Gardening Inc. On a fall afternoon when there was an abundance of leaves over her expansive front lawn, Alice decided to use the leaf blower for the first time. As she was using the leaf blower near her neighbor, Patrick's driveway, it ignited from getting over-heated, and sparks flew out from the leaf blower, injuring Patrick. It was determined that the sparking occurred because Gardening Inc. failed to carefully inspect a compressor belt before marketing the leaf blower. If Patrick asserts a products liability claim based on negligence against Gardening, Inc., will he prevail? Yes, if the defect could have been discovered through the exercise of reasonable care by Gardening Inc. Yes, because Gardening Inc. knew that there was a risk that the leaf blower could spark. No, because Alice and not Patrick, purchased the leaf blower. No, assuming that the risk of sparking was small.

A is the best answer. One who negligently manufactures a product is liable for any personal injuries caused by its negligence. Here, the sparking was caused by Gardening Inc.'s negligent failure to carefully inspect a compressor belt before marketing the leaf blower, making it liable for Patrick's harm. B is wrong since Gardening Inc.'s liability stems from its failure to inspect the product, which would have revealed the danger. C is wrong because privity is no longer required. D is wrong because the risk of harm here was preventable had Gardening Inc. exercised reasonable care.

Question 8 2 / 2 pts Peter buys a used car from Donald, a car salesman, for Pamela, his wife. Peter inspects the car and asks Donald if the car is safe. Donald replies "Oh yeah, it even has a shatterproof windshield." Peter buys the car. One day when Pamela was driving the car on her way home, a rock in the roadway shattered the windshield of the car. If Pamela sues Donald for her injuries based on a theory of breach of express warranty, will she likely prevail? Yes, since Donald expressly represented that the windshield was shatterproof. Yes, since the car was not fit for an ordinary purpose. No, because a car dealer is not an insurer of the safety of the cars that it sells. No, because Donald made the representation about the windshield to Peter, not his wife.

A is the correct answer since Donald expressly represented that the car had certain qualities, a shatterproof windshield. Since the car did not apparently have a shatterproof windshield, Pamela has a claim for breach of express warranty. Courts routinely extend the warranty to members of the purchaser's family who are harmed by the product. B is incorrect since there are not facts to indicate that the car did not function as a car should. Further, products carry implied warranties of fitness for an ordinary purpose and Pamela is bringing her claim under an express warranty theory. C, while more or less describing a manufacturer's duty under strict liability, does not answer the question. D is wrong since courts routinely extend the warranty to members of the purchaser's family who are harmed by the product.

Score for this quiz: 30 out of 30 Submitted Nov 22 at 8:47pm This attempt took 29 minutes. Question 1 2 / 2 pts Farmer Fred had a large farm and several large pieces of farm equipment. In order to run that farm equipment, he needed a supply of gasoline. Fred installed an underground gasoline storage tank and filled it with gasoline. The tank worked well for several years, but then it developed a small leak. Because of local geological conditions, no one else used underground storage tanks in this area. Before anyone realized the problem, the gasoline had leaked out and contaminated the water well of his closest farming neighbor, Organic Jane. If Jane sues Fred, which of the following is true? Strict liability will apply since the use of underground tanks is not of common usage and inappropriate to the area. Strict liability will not apply since the use of underground tanks is common. Farmer Fred will not be liable since his underground storage tank worked well and did not contaminate Jane's water well for several years. Farmer Fred will not be liable since he did not intend to contaminate Jane's water well.

A is the correct answer since one of the factors to be considered to determine whether conduct is considered to be an 'abnormally dangerous' activity is whether the activity is appropriate to the area in which it is carried out. If the activity is 'abnormally dangerous', strict liability applies to hold the actor liable for any injuries caused by the activity. B is incorrect since this is contradicted by the facts. C is wrong since even with a history of no harm caused by the activity, it would still be considered 'abnormally dangerous' since it was not of common usage or appropriate to the area. D is wrong since intent is irrelevant.

Question 3 15 / 15 pts Adam's car sustained moderate damage in a collision with a car driven by Bob. The accident was caused solely by Bob's negligence. Adam's car was still drivable after the accident. No gas leakage was noticeable at the accident scene. Examining the car the next morning, Adam could see that a rear fender had to be replaced. He also noticed that large amounts of gasoline had dripped onto the garage floor. The collision had caused a sizable leak in the gasoline tank. Adam then took the car to Mechanic, who owns and operates a body shop, and arranged with Mechanic to repair the damage. During their discussion Adam neglected to mention the gasoline leakage. Thereafter, while Mechanic was loosening some of the damaged material with a hammer, he caused a spark, igniting vapor and gasoline that had leaked from the fuel tank. Mechanic was severely burned. Mechanic has brought an action to recover damages against Adam and Bob. The jurisdiction has adopted a pure comparative negligence rule. In this action, will Mechanic likely obtain a judgment against Adam? Yes, if a reasonable person in Adam's position would have warned Mechanic about the gasoline leak. Yes, because the car was unreasonably dangerous when Adam delivered it to Mechanic. No, because it was Mechanic's job to inspect the vehicle and repair whatever needed repair. No, unless Adam was aware of the risk that the gasoline leak represented.

A is the correct answer. Mechanic will prevail if Adam breached the duty of due care to the Mechanic. B is wrong since there are no facts to indicate the car was unreasonably dangerous. C is wrong since even if the mechanic was negligent, it does not negate Adam's negligence. D is wrong since if a reasonable person would have warned the mechanic, Adam is negligent for failing to warn him.

Question 15 2 / 2 pts Patrick ordered some merchandise from Store but decided he did not want it and returned it for credit. Store refused to credit Patrick's account, continued to bill him, and, after 90 days, turned the account over to Dexter, a bill collector, for collection. One evening, during a block party on Patrick's street, Dexter showed up with a bullhorn and began shouting "Patrick is in debt up to his eyeballs! He's so poor he can't even pay off a charge that he made at Store! What a loser!" Patrick was so traumatized by Dexter's conduct that he had a nervous breakdown and required psychiatric treatment. If Patrick asserts a claim based on intentional infliction of emotional distress against Dexter, will Patrick prevail? Yes, because Dexter was intruding on Patrick's privacy. Yes, because Dexter's conduct was extreme and outrageous. No, because Dexter did not intend to cause emotional distress, he intended to force Patrick to pay his bill. No, if Patrick still owed Store for the merchandise.

B is correct. Extreme and outrageous conduct is an element of the prima facie case for intentional infliction of emotional distress. Because Peter's conduct was extreme and outrageous, intentional, and caused Patrick severe emotional distress (a nervous breakdown), Patrick will probably prevail. (B) is incorrect because intrusion on the plaintiff's privacy does not necessarily give rise to a claim for intentional infliction of emotional distress. (C) is incorrect because Dexter's conduct was at least reckless. (D) is incorrect because, even if Patrick still owed Store for the merchandise, this would not justify extreme and outrageous methods of bill collection. Quiz Score: 24 out of 30

Question 9 2 / 2 pts Petra and Tammy were both patients in Hospital. Petra was scheduled for a kidney removal with Dr. Pham and Tammy was scheduled for an appendectomy with Dr. Timmons. Somehow, hospital personnel showed Dr. Timmons to the operating room with Petra who was already under anesthesia, and he conducted the appendectomy operation on Petra instead of Tammy. If Petra brings a claim for negligence against the Hospital, who will prevail? Petra will prevail based on the doctrine of negligence per se. Petra will prevail based on the doctrine of res ipsa loquitur. The Hospital will prevail, because Petra assumed the risk when she agreed to have surgery at the Hospital. The Hospital will prevail because it had no duty to Petra.

B is correct. This is a classic example of the res ipsa loquitur doctrine - which requires 1) the harm causing event was probably due to negligence; 2) exclusive control of the instrumentality; and 3) absence of voluntary action by the plaintiff. A is incorrect because the doctrine of negligence per se is not an issue in this question, as there is no statute mentioned. This is a classic example of the res ipsa loquitur doctrine - which requires 1) the harm causing event was probably due to negligence; 2) exclusive control of the instrumentality; and 3) absence of voluntary action by the plaintiff. C is wrong, because Petra did not assume the risk of the harm that occurred knowingly and voluntarily. D is incorrect - the Hospital did have a duty to Petra.

Question 8 2 / 2 pts Petra sued Damian for personal injuries resulting from Damian's negligence in leaving a jack on the road after repairing his tire. Damian had gotten a flat and fixed it but drove off leaving the jack in the roadway. Petra was entering onto the roadway when she hit the jack, which caused a tire to blow out and caused an accident with another vehicle. Petra received $30,000 in payments for her medical expenses from her insurance company. Petra recovered $100,000 from Damian at trial, based on the jury's verdict. Petra's total recovery will be: $70,000 from Damian, because Petra's recovery must be offset by what she received from the insurance company, otherwise she will be compensated in excess of her actual damages. $100,000 from Damian, based on the collateral source rule, because Damian should not benefit from the Petra's diligence in obtaining insurance. $130,000 from Damian, because the insurance company should also be able to recover from Damian. None of the above.

B is the best answer based on the collateral source rule, which provides that a tortfeasor should not benefit from payments made to the plaintiff for injuries by sources independent from the tortfeasor. A is incorrect because Damian's liability will not be reduced based on the collateral source rule. C is incorrect because the insurance company is not a party to the action, and will not be able to recover from Damian. D is incorrect because B is correct.

Question 3 2 / 2 pts Don runs a halfway house for people just released from inpatient drug rehabilitation. The home is located in a residential area where many families live. Residents and their friends often gather outside the halfway house in the evening, making noise late into the night. This has caused disturbance to Ned and his family, who are neighbors, because their young children have awakened due to the noise. Ned and his family may have a claim against Don: Only if the residents of the halfway house physically intrude upon the property of Ned. Only if the noise generated by the residents of the halfway house substantially interferes with the use of Ned's property. Only if the harm to Ned differs from the harm to the public generally. Only if the property is zoned so that Don's use of the property as a halfway house is not permitted.

B is the best answer based on the elements for nuisance. A is incorrect because physical intrusion is caused for trespass, but is not required for nuisance. C is not applicable to private nuisance, and not required for private nuisance. D is incorrect because there may be a claim for nuisance even if there is no zoning on the property, as a zoning violation is not required.

Question 20 0 / 15 pts Owen owns a lot in City with an abandoned house on it surrounded with a fence and marked with no trespassing signs. Owen inherited the lot and lives in another state and has never visited City. Owen has put the lot up for sale, and plans to demolish the old house. Tom, an eight year old boy, cuts across the lot daily, hopping the fence, on his way to and from the nearby elementary school. In a heavy rainstorm one afternoon, Tom took shelter in the old house. As he stepped inside, an old rotted floor board gave out, injuring Tom. Is Owen liable for Tom's injury? No, because landowners never have a duty to take action to protect trespassers. No, because Owen has no knowledge of trespassers on the property. Yes, because Tom is a child. Yes, because landowners have a general duty to inspect the property and warn trespassers.

B is the best answer because an owner of property to a trespasser if he has no reason to believe that his property is frequently used by trespassers, and there is no indication Owen has such knowledge here. A is incorrect because landowners have duties child trespassers and other trespassers under certain circumstances. C is incorrect because there is no duty triggered solely by Tom being a child - Tom must still have some knowledge that children are trespassing, which is absent here. D is incorrect, because generally there is no general duty to inspect and warn trespassers.

Question 2 2 / 2 pts Jill was angry with her supervisor Sam because he would not promote her. She put a sleeping pill in his coffee one morning and caused him to miss a major meeting. When Sam's supervisor Sara learned that Bob was asleep in his office during the important meeting, she fired Sam, with no questions asked. Is Jill liable for battery? Yes, if Sam had apprehension of Jill's conduct. Yes, because Jill's contact with Sam was harmful and offensive. No, because Jill had no contact with Sam. No, if she had no intent to cause Sam to be fired.

B is the best answer because battery requires intentional harmful or offensive contact with a person. A is incorrect because apprehension is not relevant to battery. C is incorrect because Jill had indirect contact with Sam, which suffices for battery. D is incorrect because intent to cause harmful or offensive contact is required, not intent to cause Sam to be fired, which is irrelevant to the tort of battery.

Question 2 2 / 2 pts Peter, a law student, was seriously injured when working out as a result of a defective exercise device. Peter sued Defendant Super Flex, the manufacturer of the equipment, and also, Silver's Gym, a nationwide chain, where he had been working out at the time of the injury. At trial, the jury awarded Peter $10 million in damages. In a jurisdiction following comparative fault, the jury determines that Super Flex was 75% at fault and Silver's Gym was 25% at fault. Peter recovers the full $10 million from Silver's Gym. If Silver's Gym sues Super Flex to resolve their respective liability, the most likely result is: Super Flex and Silver's Gym will share liability equally. Silver's Gym is entitled to contribution from Super Flex Peter can choose to collect from either defendant. Super Flex is entitled to indemnity from Silver's Gym.

B is the best answer because contribution will allow one party who pays more than his pro rata share based on proportional fault to recover from a co-defendant based on the proportional fault. Here, Silver's Gym paid 100% of the judgment but is only 25% liable, so it may recover 75% of the judgment from Super Flex. A is incorrect because this is a jurisdiction following comparative fault, so the parties will bear liability according to their proportional fault. C is a correct statement, but it does not resolve the liability as between Super Flex and Silver's Gym. D is incorrect because a manufacturer is not entitled to shift 100% liability to the retailer through indemnity.

Question 8 2 / 2 pts Priscilla was injured in an automobile accident with Dillon after Dillon ran a red light. She broke her arm and because she is a paralegal, Priscilla cannot work since she cannot type. She is projected to be out of work for six weeks. She has incurred expenses for treatment of her arm. If Priscilla sues Dillon for negligence, she can recover: I. Compensatory damages for lost earnings II. Medical expenses III. Punitive damages against Dillon All of the above. I and II only. II and III only. I and III only.

B is the best answer because damages for medical expenses and lost earnings are recoverable as compensatory damages, but punitive damages are not recoverable for negligence. Therefore A, C and D are incorrect.

Question 5 2 / 2 pts Patty sued for injuries and damage to her car that occurred when she was injured in an automobile accident with Arnold, who was driving a truck manufactured by Freight Haulers, Inc. The accident occurred due to a problem with the steering, when the truck's steering malfunctioned and locked up, forcing Arnold to swerve into oncoming traffic. If Patty sues Freight Haulers, Inc. on a theory of strict products liability, she can recover: I. Lost earnings, as special damages II. Pain and suffering, as general damages III. Past and future medical expenses, as general damages. All of the above. I and II only. I and III only. II and III only.

B is the best answer because special damages include lost earnings and general damages include pain and suffering; however, medical expenses are classified as special damages, not general damages. Therefore A, C and D are incorrect.

Question 9 2 / 2 pts Devin's dog Trixie has always been very friendly with children. When Devin's sister brought her 10 year old son Sam to visit, Sam and Trixie played in the yard for an hour. After Devin and his sister went inside, Sam came running into the house screaming that Trixie attacked him. Sam's arm was bleeding badly, and they rushed him to the hospital emergency room for treatment. Devin was shocked at Trixie's behavior. If Devin's sister sued Devin on behalf of Sam based on strict liability, Sam's best argument would be that: Devin knew that Trixie's breed of dog was potentially dangerous. Trixie had bitten another person on a prior occasion while unprovoked. Devin failed to use reasonable care in training Trixie. Sam failed to use reasonable care in playing with Trixie.

B is the best answer because strict liability applies where a domestic animal has previously exhibited a dangerous propensity. A is a possibility, since knowledge that a breed is potentially dangerous may give some indication of the individual animal which would be reason to know of dangerous characteristics. However, B is a stronger argument. C is incorrect because a failure to use reasonable care is the standard for negligence, not strict liability. D is incorrect because Sam's failure to use reasonable care would not benefit Sam; rather, this would be an argument for contributory negligence, which would benefit Devin, so it would not be Sam's best argument.

Question 14 0 / 2 pts Nurse Nancy worked in a hospital in Los Angeles where celebrities were sometimes treated. One day, she saw Lady Baba, a well-known singer, was admitted to the hospital. Nancy went to the floor where Lady Baba was being treated and used her cell phone to take a photo of Lady Baba sleeping in her hospital bed. She sold it to the tabloid Snitch for $500. Snitch published it with the caption, "Drug Overdose!" when in fact Lady Baba had been admitted for exhaustion. Based on the story, her recording company cancelled a lucrative contract with her. Lady Baba has a viable claim against Nancy for: I. Intrusion upon seclusion II. Public disclosure of private facts III. False light All of the above. I and II II and III I and III

B is the best answer because there is a viable claim for intrusion where defendant unreasonably violates the plaintiff's reasonable expectation of privacy, such as by spying or photographing a person. There is also a viable claim for public disclosure of private facts, based on the disclosure of the photo. There is no claim for false light against Nancy because she did not make any statements about Lady Baba, so III is not applicable here.

Question 7 2 / 2 pts Devlin is a cab driver. One night a thief jumps into his cab and points a gun to Devlin's head shouting "drive me to the airport - fast." Devlin is petrified but drives through heavy traffic as fast as he can while the gunman continues screaming at him. Pedestrian steps into the street and is hit by Devlin. What is Devlin's duty to Pedestrian? To act as a reasonable person under ordinary circumstances. To act as a reasonable person in a similar emergency situation. To act as a reasonable person of like age, intelligence and knowledge. To act as a reasonable person of like physical characteristics.

B is the best answer because this is an emergency situation so Devlin will not be held to the standard of care of an ordinary person. A is incorrect because this is an emergency situation so Devlin will not be held to the standard of care of an ordinary person. C is incorrect because the answer states the child standard of care. D is incorrect because it states the standard of care for a disabled person, which is not applicable here.

Question 15 2 / 2 pts The human resources manager for Acme Co. asked Brad if he knew why Carla had fallen asleep at her desk during the middle of the work day. Brad said that he thought she had been drinking alcohol, and indicated that Carla had a very serious drinking problem. In fact, Brad had placed a sleeping pill in Carla's coffee, in an attempt to get her fired. When Carla found out what Brad had said about her, she was extremely upset, embarrassed and humiliated. Acme fired Carla the following day. If Carla sued Brad for defamation, who will prevail? Carla, because Brad made the statement hoping to get Carla fired. Carla, because Brad knew his statement to the human resources manager was false. Brad, because he had a qualified privilege to make the statement. Brad, because he did not act with malice.

B is the best answer here because Brad acted with knowledge that his statement was false, making it slanderous per se. A is incorrect because the motivation for the statement does not go to an element of the offense of defamation. C is incorrect as there is no qualified privilege to make a false statement knowing it was false. D is incorrect because Brad's conduct in making the false statement knowing it was false would qualify as malice, however it is also irrelevant here, because Carla is not a public figure. Quiz Score: 26 out of 30

Question 6 2 / 2 pts Dex owns a lion that he has raised since it was a young cub. On one occasion, when his neighbor Paul came over to watch a football game, the lion, weighing over 500 pounds, tackled Paul, causing him to pull a muscle in his back. Dex also owns a dog that has never bitten anyone. However, on another occasion when Paul visited, the dog bit Paul in the neck, almost killing him. Paul sues Dex for both injuries, asserting strict liability. Will Paul prevail? Yes, Paul will prevail on his strict liability claims for injuries caused by the lion and the dog. Yes, but Paul will prevail only on his strict liability claim for injury caused by the lion. Yes, but Paul will prevail only on his strict liability claim for injury caused by the dog. No, Paul will not prevail on either claim for strict liability.

B is the correct answer since a person who keeps a 'wild' animal, such as a lion, is strictly liable for all damage caused by the animal, as long as damage results from the 'dangerous propensity' that is typical of that species in question. But injuries caused by domestic animals (cats, dogs, pigs, etc) do not give rise to strict liability unless the owner knows or has reason to know of the animal's dangerous characteristics. A is incorrect since Paul will not prevail on a strict liability claim for injuries caused by the dog since the dog is a domestic animal and there are no facts that Dex was aware of the dog's dangerous characteristics since the dog had never bitten anyone before.t C and D are incorrect for the same reasons.

Question 15 0 / 15 pts Phillip was walking around downtown in a city where there was a tremendous amount of construction on-going on various skyscrapers by several different construction companies. As he was walking down the sidewalk, a load of cargo fell on Phillip's shoulder, severely injuring him. Phillip sued all of the construction companies who were working on building projects in that area of the city at the time that Phillip was injured. Can Phillip proceed on the theory of res ipsa loquitur? Yes, because all of the companies were the proximate cause of Phillip's harm. Yes, because Phillip suffered an injury caused by one of the construction companies, which normally would not occur without negligence, through no fault of his own. No, because Phillip assumed the risk of harm by walking through a construction zone. No, because the companies did not have a duty of due care to Phillip, who was not a foreseeable plaintiff.

B is the correct answer since it includes all the required elements for res ipsa loquitur. D is incorrect because Phillip is a foreseeable plaintiff in the zone of danger, and the construction companies owed him a duty of care. C is incorrect because Phillip did not assume that risk that a load of construction materials would fall on him knowingly and consciously simply by walking through the construction zone. A is incorrect because not all of the companies caused his harm, only one of the companies did so

Question 14 0 / 15 pts Pam decided to take a vacation by herself. She booked a four-night stay at a resort. On the third day of her trip, she had dinner at a local restaurant and that night, she woke up with severe abdominal pains. Pam called the resort's front desk and asked the clerk to call a doctor. The clerk mistakenly took down the wrong room number and sent the doctor to the wrong room. As a result, the doctor never came to Pam's room. The next morning, a maid found Pam in her room, unconscious and Pam was taken to a hospital where she was treated for severe food poisoning. Pam had a permanent injury from the food poisoning. Pam asserted an action against the resort for negligence. At trial, the medical expert testified that Pam's injury would have been exactly the same, whether she was diagnosed at night or the next morning. What is the likely result? The resort will win because it did not breach any duty to Pam. The resort will win because Pam cannot establish proximate cause. The resort will win because it had no duty to summon a doctor. The resort is liable because it failed to send the doctor to Pam's room.

B is the correct answer since the resort will win since the outcome for the plaintiff would have been the same whether or not her illness had been diagnosed the night she first complained or the next morning. A is wrong since the resort assumed a duty to Pam when it promised to send a doctor to her room and breached that duty when it failed to do so. C is incorrect because the resort assumed a duty to summon a doctor and did not. D is wrong because the resort's failure to send the doctor to the correct room did not cause harm to Pam.

Question 14 2 / 2 pts Preston was driving to school with his 5-year old son Sam, when he was hit by a car driven by Deb when she ran a red light. Walker was walking his dog at the time and saw the accident and saw that Sam was injured, but Walker failed to assist Sam. Deb walked away from the accident uninjured. Deb also failed to assist Sam. Preston was not in a position to assist his son, as Preston was also severely injured. The delay in Sam receiving assistance after the car accident caused him to suffer from severe injuries. If Preston sues Deb and Walker on Sam's behalf for their failure to assist Sam, what is the most likely result? Both Deb and Walker had a duty to assist Sam. Deb had a duty to assist Sam, but Walker did not have a duty to assist Sam. Walker had a duty to assist Sam, but Deb did not have a duty to assist Sam. Neither Deb nor Walker had a duty to assist Sam.

B is the correct answer. Generally there is no duty to rescue another person. However, there is an exception where a person creates the peril involved, in which case a duty to rescue arises. Here, since Deb created the peril to Sam by running the red light, she would be required to use reasonable means to assist him. Walker, on the other hand, would have no duty to rescue Sam, a stranger. Consequently B is correct and all of the other answer choices are wrong.

Question 4 2 / 2 pts Polly purchases a chair from a department store. Subsequently, Polly stands on the chair to hang a picture. Due to a defect, the chair collapses, seriously injuring her. Polly brings a product liability action against the department store, claiming her injury was caused by a defect in the chair. The store defends the products liability action by claiming that Polly misused the chair by standing, rather than sitting on it. Who is likely to prevail? Polly, because the chair is defective. Polly, because her misuse of the chair was foreseeable. The store, since chairs are designed for sitting, not standing. The store, since Polly misused the chair by standing on it to hang a picture.

B is the correct answer. It is well known that chairs frequently are utilized as 'ladders'. Although chairs are not constructed for the purpose of having adults stand upon them, it is well known that these items are frequently utilized as makeshift foot-stools. Thus, using a chair to hang paintings would likely have been anticipated by the store. A is not the best answer since it is incomplete and does not address the store's defense to Polly's claim. C is wrong since although chairs are not constructed for the purpose of having adults stand upon them, it is well known that these items are frequently utilized as makeshift foot-stools. Thus, using a chair to hang paintings would likely have been anticipated by the store. D is wrong for the same reason.

Question 6 0 / 2 pts Polly purchases a chair from a department store. Subsequently, Polly stands on the chair to hang a picture. Due to a defect, the chair collapses, seriously injuring her. Polly brings a product liability action against the department store, claiming her injury was caused by a defect in the chair. The store defends the products liability action by claiming that Polly misused the chair by standing, rather than sitting on it. Who is likely to prevail? Polly, because the chair is defective. Polly, because her misuse of the chair was foreseeable. The store, since chairs are designed for sitting, not standing. The store, since Polly misused the chair by standing on it to hang a picture.

B is the correct answer. It is well known that chairs frequently are utilized as 'ladders'. Although chairs are not constructed for the purpose of having adults stand upon them, it is well known that these items are frequently utilized as makeshift foot-stools. Thus, using a chair to hang paintings would likely have been anticipated by the store. A is not the best answer since it is incomplete and does not address the store's defense to Polly's claim. C is wrong since although chairs are not constructed for the purpose of having adults stand upon them, it is well known that these items are frequently utilized as makeshift foot-stools. Thus, using a chair to hang paintings would likely have been anticipated by the store. D is wrong for the same reason.

Question 5 2 / 2 pts Golfo manufactures golf clubs. The clubs have rubberized grips that golfers hold onto to swing them. Bruce bought a set of clubs and took them when he went golfing with his golfing partner, Pat. While swinging a club, the grip came loose and the club sailed through the air, striking Pat in the forehead. On Bruce's particular golf club, Golfo had used a faulty glue that loses its adhesion after just a few months. If Pat asserts a strict products liability action against Golfo, will she likely prevail? Yes, since the golf club was defectively designed. Yes, since under strict liability theory, Pat, as a bystander, is a proper plaintiff. No, since Pat did not purchase the golf club. No, since Pat was not a user of the golf club.

B is the correct answer. Under strict liability, any individual who is foreseeably harmed by a defective product as a bystander is a proper plaintiff, whether or not they purchased or were the user of the product. A is incorrect since the golf club had a manufacturing, not design defect. The fact that a product has a defect, standing alone, does not necessarily make the manufacturer liable to Pat. C and D are wrong since under strict liability, any individual who is foreseeably harmed by a defective product as a bystander is a proper plaintiff, whether or not they purchased or were the user of the product.

Question 11 2 / 2 pts In 2006, Silo Cement Company constructed a plant for manufacturing ready-mix concrete in Lakeville. At that time Silo was using bagged cement, which caused little or no dust. In 2010, Peter bought a home approximately 1,800 feet from the Silo plant. In 2015, Silo stopped using bagged cement and began to receive cement in bulk shipments. Since then, at least five truckloads of cement have passed Peter's house daily. Cement blows off the trucks and into Peter's house. When the cement arrives at the Silo plant, it is blown by forced air from the trucks into the storage bin. As a consequence, cement dust fills the air surrounding the plant to a distance of 2,000 feet. Peter's house is the only residence within 2,000 feet of the plant. If Peter asserts a claim against Silo based on nuisance, will Peter likely prevail? Yes, unless using bagged cement would substantially increase Silo's costs. Yes, if the cement dust interfered unreasonably with the use and enjoyment of Peter's property. No, because Silo is not required to change its industrial methods to accommodate the needs of one individual. No, if Silo's methods are in conformity with those in general use in the industry.

B is the correct answer. Peter will prevail if the cement dust constituted an unreasonable interference. For a private nuisance to be actionable, defendant's conduct must amount to a substantial, unreasonable interference with plaintiff's use and enjoyment of property. A is incorrect because, even though a balancing test is sometimes used in nuisance cases, mere increased costs is not a sufficient justification to allow continued unreasonable interference with plaintiff's rights. C is incorrect because the number of individuals affected is not determinative of whether a nuisance exists. D is similarly incorrect; conformity to general methods in the industry will not preclude Silo from being liable for nuisance.

Question 10 2 / 2 pts Dennis lives in a remote cabin in the woods where he keeps a pet bear. Dennis tries to keep the bear inside the cabin in the summer when the area is frequented by campers. One night, Dennis' bear escapes from the cabin and wanders over to a campsite where Pat and her family are sitting around, singing camping songs. The bear lets out a huge roar, and although the bear did not attack Pat, it terrified her so much that she suffered a heart attack. If Pat asserts an action against Dennis in strict liability for her injury, she will likely: Prevail, since Dennis did not act reasonably in allowing the bear to escape the cabin. Prevail, since Dennis is strictly liable for injuries Pat suffered that were caused by the bear frightening her. Not prevail, since the bear did not physically attack Pat. Not prevail, because Dennis' bear was not a dangerous animal.

B is the correct answer. Since a bear is a wild animal, Dennis is strictly liable for any injuries caused by the animal. Arguably, Pat's heart attack was caused by the bear's dangerous propensity, its ability to frighten people and other animals by its loud roar. A is wrong since whether or not Dennis acted reasonably is not relevant to a strict liability claim. C is wrong since the nature of the injury caused to Pat is not relevant to her strict liability claim. D is wrong since a bear is a dangerous (wild) animal.

Question 5 2 / 2 pts Drink Inc. has just discovered a way to make a new artificial sweetener that they hope will become the next big product to be labeled as "safe sweetener" used in various diet sodas. However, the manufacturing process requires the use of Polyexlyte, a highly poisonous chemical. Because it is so poisonous, Drink stores it off site in a large tank. One morning, a security guard and Drink employee who was monitoring the storage facility of Polyexlyte, dropped one of the storage tanks, causing a small leak in the tank. The leak allowed the Polyexlyte to seep into the water supply and poisoned Phil, a downstream neighbor, who drank some of the water. If Phil sues Drink, will Phil likely succeed in arguing that strict liability applies? Yes, because Phil drank the water. Yes, because storing a poisonous substance is an abnormally dangerous activity that cannot be carried out safely, even with the exercise of great care. No, because it was unlikely that anyone would drop one of the tanks, causing the poison to seep into the water supply. No, because storing a poisonous substance is not an abnormally dangerous activity.

B is the correct answer. Strict liability applies to abnormally dangerous activities the risks of which cannot be eliminated by the exercise of reasonable care. Storing a poisonous substance would be considered subject to strict liability since it carries a high degree of risk of harm to others and it is not a common activity in a community. A is incorrect since the fact that Phil drank the water is not the only issue that is considered in determining whether strict liability applies. Strict liability applies to abnormally dangerous activities the risks of which cannot be eliminated by the exercise of reasonable care. Storing a poisonous substance would be considered subject to strict liability since it carries a high degree of risk of harm to others and it is not a common activity in a community. D is wrong for that reason. C is wrong since in strict liability, the actor carrying out the activity is liable for any harm caused by the activity, no matter how unusual.

Question 2 2 / 2 pts Patrick was using a ladder to put up his holiday lights on a wet December afternoon. He was almost done, but he could not reach the highest point of his roof from his current position on the ladder. He decided to stand on the top step of the ladder, and the ladder tipped over as a result. There was no warning on the ladder, so Patrick sues the manufacturer for negligent products liability- a failure to warn. The ladder manufacturer defends, claiming that the injury was a result of Patrick's misuse of the ladder. Who is likely to prevail? Patrick, since the ladder tipped over. Patrick, although his recovery may be reduced by his own negligence. The ladder manufacturer, since there is no duty to warn of the danger of stepping on the top step of a ladder. The ladder manufacturer, since Patrick misused the ladder

B is the correct answer. This is not the intended use of the ladder, but it is a foreseeable one. However, a court may also consider Patrick's own negligence in standing on the top step, which could limit the amount of his recovery on a comparative fault basis. A is wrong since this does not fully answer the question since it only describes how the injury occurred. C is wrong since it is likely that the manufacturer did have a duty to warn of the danger of standing on the top step of the ladder. D is wrong since foreseeable misuse by Patrick would not bar his recovery, although it may lessen it.

Question 2 2 / 2 pts Bob works for the LAPD as a detective having received numerous awards and TV interviews. He decides to retire and leaves the force to play golf. One day he is watching television and he comes across "LA Law" a docu-drama about the detective trade in LA. After the initial credits, Bob sees the following: "based on the trials and tribulations of detective Bob's life while serving on the LAPD police force. Please be aware that this is a stylized documentary and things may appear in a manner slightly different than they actually occurred." Bob watches the show and becomes angry, as the plot includes a sordid inter-office affair with a dispatcher that never occurred in his real life. Assume that Bob brings an action against the broadcasting company for invasion of privacy. Who will likely prevail? Bob, since they used his name for a commercial purpose. Bob, since his retirement and the associated right to seclusion has been upset. Bob, as he was portrayed in a false light. The broadcasting company, as the show when taken as a whole, was complimentary to Bob's life.

C is the best answer. Invasion of privacy based on a publication of facts that place the plaintiff is a false light, requires a showing of the following elements: 1) publication of facts about the plaintiff by the defendant placing the plaintiff in a false light in the public eye; 2) the "false light" must be something objectionable to a reasonable person under the circumstances; and 3) malice on the defendant's part where the published matter is in the public interest. Facts will be found to place a plaintiff in a false light if they showcase: 1) views that the he does not hold, or 2) actions that he did not take. The alleged sexual affair with a dispatcher did not take place and would be objectionable to a reasonable person under the circumstances. This meets the requirement for false light invasion of privacy (A) is incorrect. The case for invasion of privacy based on the use of a person's name, requires a showing of unauthorized use by the defendant of their picture or name for commercial advantage. Typically, this is the use of a name or image in the sale of a product by the defendant. Here, Bob's name is only in the credits and no facts indicate that it was used to boost sales etc. (B) is incorrect. The case for invasion of privacy based on an intrusion upon the plaintiffs affairs or seclusion needs proof of one the following: 1) the act of prying or intruding upon the affairs or seclusion of the plaintiff by the defendant; 2) the intrusion must be something that would be objectionable to a reasonable person; or 3) the thing to which there is an intrusion or prying must be "private." Here, there are no facts indicating that the broadcasting channel invaded Bob's private affairs by creating sex scenes that he objected to. (D) is incorrect because if Bob's privacy has been invaded, then the defense that the program as a whole was complimentary is not valid.

Question 5 2 / 2 pts Del lived next door to Pam in a Senior Citizens' apartment complex. Pam sometimes left her stove on, and Del could smell things burning from time to time. One day, from the balcony of his apartment, Del saw a fire in the area of Pam's kitchen stove. Del banged on the door, but she did not answer. Del got a hammer and banged to doorknob off, and entered Pam's apartment, to put out the fire. Finally Pam awoke to find Del in her apartment. She shouted at him to get out of her house. If Pam sues Del for trespass, the most likely result is that: Del is liable, because he entered into Pam's apartment without her permission. Del is liable, because he should have called the fire department. Del is not liable, due to the necessity of protecting against the spread of the fire. Del is not liable, because he acted in self-defense.

C is the best answer because the defense of public necessity would allow him to act protect against the spread of the fire. A is incorrect because, although Del did commit trespass, he has the defense of necessity. B is incorrect because, Del has the defense of necessity and the fact that he could have called for the fire department is irrelevant to liability. D is incorrect because Del is not acting in self-defense as he is not personally threatened, and is not protecting himself personally.

Question 6 2 / 2 pts Paul wanted to ship his cattle to a remote location up the coast for grazing. A statute requires that when animals are transported, each breed must be kept in a separate pen to avoid disease. Darwin, a ship operator, transported all of the animals by ship. During a storm several of the animals were washed overboard. If Paul brings a claim for negligence based on negligence per se, Paul can establish a duty based on the statute due to the loss of the sheep. Paul can establish a duty based on the statute because the accident would not have occurred in the absence of negligence by Darwin. Paul cannot establish a duty based on the statute because the statute was not intended to prevent the type of harm that occurred. Paul cannot establish a duty based on the statute because the statute creates the standard of care, but compliance was excused.

C is the best answer because the harm that occurred is not the type of harm that was intended to be prevented by the statute since it was aimed at avoiding disease. A is incorrect for the same reason: the harm that occurred is not the type of harm that was intended to be prevented by the statute since it was aimed at avoiding disease. B is incorrect because the test stated is for res ipsa loquitur, not negligence per se. D is incorrect because the statute did not create the standard of care in this situation because the harm that occurred is not the type of harm that was intended to be prevented by the statute since it was aimed at avoiding disease.

Score for this quiz: 20 out of 20 Submitted Oct 25 at 9:40pm This attempt took 19 minutes. Question 1 2 / 2 pts Walt published a book about the employees of Diamond's Department Store, a high-end, exclusive boutique with twenty world-wide locations, known for in-store modeling of designer evening wear. In his book, Walt stated that all of Diamond's models are "high end call girls," and some of the saleswomen are "cheap whores". Walt repeated these statements on his book tour on radio and television. Diamond employed nine models, and 300 saleswomen, all of whom filed suit against Walt for defamation. In fact none of the models or sales women were prostitutes. Which of the following are true? The statements are statements of opinion. All of the statements are actionable. The statements regarding the saleswomen are not actionable. The statements regarding the models are not actionable.

C is the best answer because the statements about the saleswomen refer to a group and the group is large so that the defamatory implication would not arise as to every member of the large group. A is incorrect because the statements are statements of fact, to wit, that the women are prostitutes. B is incorrect because the statements regarding the saleswomen are not actionable. D is incorrect because the statements regarding the relatively small group of models that all of the models are call girls would be actionable. Reference: Neiman-Marcus v. Lait 13 FRD 311 (SDNY 1952).

Question 16 15 / 15 pts Construction Company contracted to build a laundry for Wash Company on the latter's vacant lot in a residential area. Wash Company knew children often played on the lot. As a part of its work, Construction Company dug a trench from the partially completed laundry to the edge of a public sidewalk; waterlines were to be installed in the trench. Because of the contour of the land, the trench was dug to a depth ranging from seven to nine feet. Construction Company and Wash Company failed to place any barriers around the trench and permitted it to lie open for almost a week while waiting for the delivery of water pipes. During the time the trench was open, a series of heavy rains fell, causing five feet of surface water to gather in the bottom of the trench. While this condition existed, five-year-old Tommy, who was playing on the vacant lot with friends, stumbled and fell into the trench. Robert, an adult passerby, saw this and immediately lowered himself into the trench to rescue Tommy. However, his doing so caused the rain-soaked walls of the trench to collapse, injuring both Robert and Tommy. If Robert and Tommy sue Construction Company and Wash Company for negligence, the most likely result is that: Tommy will recover because he was a known trespasser, but Robert will not because his injury was not foreseeable. Tommy will recover because he was an invitee, but Robert's injury was not foreseeable. Both will recover, because the open trench was an attractive nuisance, and Robert's rescue was foreseeable. Robert will recover because his rescue was foreseeable, but Tommy will not because he was a trespasser.

C is the best answer because the trench was an attractive nuisance and the conditions for recovery are met in that Tommy is a known trespasser, the condition is dangerous, a five year old would not understand the danger created, precautions such as a fence were not put in place, and the risk to children was great and not outweighed by the utility of leaving the open trench unfenced. A is incorrect because more is required for Tommy to recover than just being a known trespasser, and Robert will be able to recover because injury to rescuers is foreseeable. B is incorrect because Tommy is not an invitee or business guest on the property and Robert's injury was foreseeable. D is incorrect because Tommy will be able to recover under the attractive nuisance theory.

Question 7 2 / 2 pts Diane is riding her bicycle in the park when she sees that a child has fallen into the fish pond and is struggling to get out, flailing and screaming. Diane could have pulled the child to safety without any risk to herself, and there is no one else around. Instead, Diane keeps pedaling. Under the majority view, what is the likely result should the child sue Diane? Diane had a duty to give assistance because the park was open to the public. Diane had a duty to give assistance because she had a superior ability to protect the child. Diane had no duty to give assistance because she had no special relationship with the child. Diane had a duty to give assistance because she must act as a reasonable person.

C is the best answer because there is generally no duty to take affirmative action, absent exceptions that are not applicable here which involve a special relationship. A is incorrect because the fact that the park was open does not create a duty on the part of Diane. B is incorrect because the test referenced applies only to a custodian, such as a jailer. D is incorrect because even a reasonable person does not have a duty to take affirmative action in the absence of a special relationship.

Question 3 2 / 2 pts Aldon was speaking at a gathering of research scientists in Nova Scotia, presenting information about his latest scientific findings. Bart, a scientist in the same discipline, and a practical joker, moved Aldon's chair slightly so one of the back legs was no longer on the stage platform. When Aldon sat down after speaking he fell backwards off of the stage. The audience gasped in surprise. Thankfully, Aldon was not injured. Is Bart liable to Aldon for battery? No, because Aldon was not injured as a result of the fall. No, because Bart did not touch Aldon. Yes, because Bart caused offensive contact to Aldon. Yes, because the incident shocked the audience.

C is the best answer. A is incorrect because battery is an offense to dignity, and nominal damages can be awarded. B is incorrect because Bart is still liable even if he did not touch Aldon himself, if Bart caused harmful or offensive contact to Aldon, as Bart did here. D is incorrect because the reaction of the audience is not related to the elements required for battery.

Question 13 2 / 2 pts Fritz owned a 30-acre parcel of undeveloped land in a suburb. The land was located between a new high school and a housing subdivision. Many students took a short cut through Fritz's property to reach the school. One Saturday morning, Fritz had a construction crew dig a 10-foot-deep hole for the foundation of a new building. Because Fritz knew that students sometimes walked across his land, he posted a warning sign near the hole. Elmer, a high school senior, walked across Fritz's property to get to school the next day. He almost ran into the sign, which was posted directly in front of the hole. Still, after walking past the sign, Elmer was not paying attention and fell into the hole, breaking both of his legs. If Elmer sues Fritz for negligence, the likely result will be: Elmer will prevail because Fritz failed to satisfy his duty to warn. Elmer will prevail because Fritz did not own him a duty to warn. Fritz will prevail because he posted a warning. Fritz will prevail because he owes no duty to Elmer.

C is the best answer. A landowner owes a duty to known trespassers to make safe or warn regarding artificial hidden highly dangerous conditions, such as the hole. Fritz performed this duty by posting the sign. A is incorrect because Fritz satisfied the duty to warn by posting the sign. B is incorrect because Fritz owed a duty to warn of the particular hazard. D is incorrect because Fritz owed a duty to Elmer.

Question 12 2 / 2 pts Bob works for as a police detective. He has received numerous awards and given TV interviews. He decides to retire and leaves the force to play golf. One day he is watching television and he comes across "LA Law" a docu-drama about the detective trade in LA. After the initial credits, Bob sees the following: "Based on the trials and tribulations of detective Bob's life while serving on the LAPD police force. Please be aware that this is a stylized documentary and things may appear in a manner slightly different than they actually occurred." Bob watches the show and becomes angry, as the plot includes a sordid inter-office affair with a dispatcher that never occurred in his real life. If Bob brings an action against the broadcasting company for invasion of privacy, who will likely prevail? Bob, since they used his name for a commercial purpose. Bob, since his retirement and the associated right to seclusion has been upset. Bob, as he was portrayed in a false light. The broadcasting company, as the show when taken as a whole, was complimentary to Bob's life.

C is the best answer. Invasion of privacy based on a publication of facts that place the plaintiff is a false light, requires a showing of the following elements: 1) publication of facts about the plaintiff by the defendant placing the plaintiff in a false light in the public eye; 2) the "false light" must be something objectionable to a reasonable person under the circumstances; and 3) malice on the defendant's part where the published matter is in the public interest. Facts will be found to place a plaintiff in a false light if they showcase: 1) views that the he does not hold, or 2) actions that he did not take. The alleged sexual affair with a dispatcher did not take place and would be objectionable to a reasonable person under the circumstances. This meets the requirement for false light invasion of privacy (A) is incorrect. The case for invasion of privacy based on the use of a person's name, requires a showing of unauthorized use by the defendant of their picture or name for commercial advantage. Typically, this is the use of a name or image in the sale of a product by the defendant. Here, Bob's name is only in the credits and no facts indicate that it was used to boost sales etc. (B) is incorrect. The case for invasion of privacy based on an intrusion upon the plaintiffs affairs or seclusion needs proof of one the following: 1) the act of prying or intruding upon the affairs or seclusion of the plaintiff by the defendant; 2) the intrusion must be something that would be objectionable to a reasonable person; or 3) the thing to which there is an intrusion or prying must be "private." Here, there are no facts indicating that the broadcasting channel invaded Bob's private affairs by creating sex scenes that he objected to. (D) is incorrect because if Bob's privacy has been invaded, then the defense that the program as a whole was complimentary is not valid.

Question 7 2 / 2 pts A man is at the airport riding up an escalator to reach the gates. He is holding on to the handrail when the escalator malfunctions. The handrail stops moving, but the stairs keep moving, causing him to lose his balance and fall, breaking his leg. The man sues the airport for negligence, but presents no proof of why the handrail malfunctioned. The airport performed all of its own maintenance. The airport argues that the negligence case cannot go to the jury on the basis of res ipsa loquitur. How should the court rule on the airport's argument? The airport should prevail since the man must show how the airport failed to use ordinary care to avoid a reasonably foreseeable risk to him. The airport should prevail the man was at fault for falling. The man will prevail if the jury could determine that this kind of escalator malfunction ordinarily does not occur in the absence of the negligence of the person in charge of keeping escalators in good repair. The man will prevail if he can put on evidence that he has never fallen on an escalator before.

C is the correct answer in stating that the issue should go to a jury under the doctrine of res ipsa loquitur if the jury might determine that these kinds of incidents ordinarily do not occur in the absence of negligence. A is incorrect because it says that a plaintiff must always show how an accident happened and what precautions could be taken to have avoided injury to the plaintiff. The doctrine of res ipsa loquitur shows that this is an incorrect statement of the law. B is incorrect since it is irrelevant if the fall was the result of the airport's negligence. D is incorrect since whether or not the man had previously fallen is not relevant to the issue of the airport's negligence.

Question 6 0 / 2 pts Paula became seriously ill after eating some canned squash that was contaminated with bacteria. Paula asserted an action for negligence against the company that canned the squash. At trial, Paula argued for the application of res ipsa loquitur. The only evidence that Paula presented was that she became ill after eating the contaminated squash. The company moved for a directed verdict on the grounds that Paula failed to meet her burden of proof under res ipsa loquitur. How should the court rule on the company's motion? Deny the motion, because Paula's illness would not happen in the absence of negligence by the company. Deny the motion, because Paula suffered an injury as a result of eating the squash. Grant the motion, because there are other ways that the squash could become contaminated with bacteria other than by the company's negligence. Grant the motion, because Paula contributed to the harm by eating the squash.

C is the correct answer since Paula did not show that she only could have gotten ill by the company's actions. Even if the precise mechanism of the injury is not known, the plaintiff must show that the negligent act probably was caused by a particular defendant. There is insufficient evidence on that issue, so the company would be entitled to a directed verdict. A is incorrect because Paula did not show that she only could have gotten ill if the company was negligent. Even if the precise mechanism of the injury is not known, the plaintiff must show that there was a negligent act probably was caused by a particular defendant. There is insufficient evidence on that issue. B is incorrect since showing an injury is not enough to prove a case based on res ipsa loquitur. D is incorrect since there is no evidence that Paula's negligence caused her illness.

Score for this attempt: 180 out of 650 * Submitted Nov 15 at 10:20pm This attempt took 120 minutes. Question 1 0 / 15 pts Mom rushed her eight-year-old daughter, Child, to the emergency room at Hospital after Child fell off her bicycle and hit her head on a sharp rock. The wound caused by the fall was extensive and bloody. Mom was permitted to remain in the treatment room, and held Child's hand while the emergency room physician cleaned and sutured the wound. During the procedure, Mom said that she was feeling faint and stood up to leave the room. While leaving the room, Mom fainted and, in falling, hit her head on the floor. She sustained a serious injury as a consequence. If Mom sues Hospital to recover damages for her injury, will she prevail? Yes, because Mom was a public invitee of the Hospital. Yes, because Mom was a licensee of the hospital. No, unless Hospital's personnel failed to take reasonable steps to anticipate and prevent Mom's injury. No, because Hospital's personnel owed Mom no affirmative duty of care.

C is the correct answer. A landowner has a duty to exercise reasonable care to prevent harm from a dangerous condition on the property. Since Mom is an invitee, the duty would consist of inspection for any dangerous conditions, warning of the danger or remedying the condition. A is wrong since Mom will not prevail simply because of her status as an invitee. B is wrong since Mom is an invitee. D is wrong because the hospital did owe Mom a duty of reasonable care.

Question 3 2 / 2 pts Mexicano manufactures and sells Mexican style foods, including chicken enchiladas. Mexicano has been producing Mexican style foods for a year and no consumers have been injured or complained about their products. Penny purchased Mexicano's chicken enchiladas and chokes on a chicken bone in the enchilada, suffering severe injury. If Penny sues Mexicano on a strict products liability theory, will she likely prevail? Yes, if a reasonable consumer in Penny's position would not have expected to find a bone in a chicken enchilada. Yes, since Penny suffered injury from eating the chicken enchilada. No, since no other consumers have been injured from eating Mexicano's chicken enchiladas No, since no other consumers have complained about Mexicano's products.

Correct Answer: A is the correct answer. When the product involved in a strict liability action is a food product, most courts apply the consumer expectation test and will find the food product to be defective only if it contains an ingredient that a reasonable consumer would not expect it to contain. B is wrong since suffering an injury is not enough to prevail in a products liability action; the product must be defective. C and D are wrong because they are not relevant to the issue of whether the chicken enchiladas eaten by Penny were defective.

Question 4 2 / 2 pts Dennis buys 2 snow tires for his truck from Tia's Tires. The manufacturer of the tires is Treadco and the two tires are mounted and balanced by Tia. After driving around for 3 weeks, Dennis notices that the left snow tire is low on air. He is angry and decides to refill the tire and then head to Tia's Tires to complain. Dennis drives to the nearest gas station for air and starts pumping air into the tire. He notices the tire slowly starting to bubble along the sidewall and the more air that he puts in, the worse the bubble gets. As Dennis puts in 5 lbs more, the bulge gets bigger and then suddenly explodes. A steel belt from inside the tire flies out and hits Dennis in the eye, causing an injury. Subsequent investigation revealed that the snow tires had a manufacturing defect. Dennis sues both Tia's Tires and Treadco. Will Dennis prevail in his suit against the tire manufacturer, Treadco? No, if Tia's Tires failed to inspect the tire. No, if the tire was improperly installed. Yes, if the defect existed when the tire left Treadco's plant. Yes, unless the design of the tires was the 'state of the art'.

C is the correct answer. As long as the tire was defective upon leaving the plant, Dennis wins under a strict liability analysis since the defective condition made the tire unreasonably dangerous. Recall that the elements for a strict liability analysis in a products liability setting are: 1) strict duty owed by a commercial supplier; 2) breach of that duty; 3) actual and proximate cause; and 4) damages. Note that in order to hold Treadco liable, the product must have reached Dennis in essentially the same condition as when it left Treadco's plant without any substantial changes. A is incorrect since the failure of Tia's Tires (an intermediary in this transaction) to discover the damage does not remove liability for Treadco. B is incorrect for the same reasons as A—Tia Tires' negligence, if any, does not mean that Treadco is no longer liable for the defective tires. Negligent installation does not break the chain from supplier to dealer and does not serve as a superseding cause to break the causal connection between injury and the Treadco's faulty manufacturing. D is incorrect since the tire had a manufacturing defect, not a design defect.

Question 3 2 / 2 pts Dave is speeding down the freeway in his brand new Corvette. While following all applicable traffic laws, Paula pulls out in front of him, and suddenly slams on her brakes when she sees a skunk in the road. Dave does not have enough time to react, and slams into the back of Paula's car. Paula sues Dave, asserting a theory of strict liability, claiming that Dave was engaged in an "abnormally dangerous activity" while driving. Is Paula is likely to prevail? Yes, since all of the risks of driving on a freeway cannot be eliminated by the exercise of reasonable care. Yes, since Dave's driving created a high degree of risk of causing harm to other drivers. No, since the activity of driving on a freeway is quite common. No, since Paula pulled out in front of Dave' car.

C is the correct answer. Individuals who engage in activities that are common in daily life, such as driving, are not held to a strict liability standard. Rather, those individuals are held to a negligence standard, where they have a duty not to cause an unreasonable risk of harm to others. Further, driving is not an activity that cannot be carried out safely. A is wrong while perhaps all of the risks of freeway driving cannot be eliminated, driving is not an activity that cannot be carried out safely. B is wrong since it states the standard of conduct for negligence, not strict liability. D is wrong since the facts indicate that Paula was not negligent since she followed all applicable traffic laws.

Question 8 2 / 2 pts Dexter lived in a home adjacent to a large green field. One afternoon Dexter took his dog, on a leash, for a walk across the fields. Unknown to Dexter, Abe was engaging in target practice by shooting at a target affixed to one of the trees in the field. Abe was hidden from Dexter's view by a small group of trees. As Dexter and his dog passed the group of trees, Abe fired at the target. The sound of the gun shot spooked Dexter's dog, causing the dog to break off the leash and run. The dog then bit Patrick who was walking in the fields about 100 feet from Patrick. If Patrick asserts a claim for damages against Dexter, will Patrick prevail? Yes, because Dexter owned the dog. Yes, because the dog escaped from Dexter's control. No, unless the dog had previously bitten some other person without being provoked. No, because shooting a gun is not an abnormally dangerous activity.

C is the correct answer. Injuries caused by domestic animals (cats, dogs, pigs, etc) do not give rise to strict liability unless the owner knows or has reason to know of the animal's dangerous characteristics. Here, if the dog had not bitten another person, Dexter would not be strictly liable for Patrick's injury (although he may be liable in negligence). A is the wrong answer because an owner of a domestic animal is only liable for injuries caused by the animal if the owner knows or has reason to know of the animal's dangerous characteristics. B is wrong because Dexter would not be strictly liable for Patrick's injury (although he may be liable in negligence). D is wrong because it is irrelevant.

Question 7 2 / 2 pts Dexter is an accountant. He also is a boat-lover who, as a hobby, rehabs old boats in his spare time and sells them. Paul sees Dexter's ad for a rehabbed Z26 MusterCraft. Paul goes over to the local marina to look at Dexter's Mustercraft. He decides to buy the boat from Dexter and says "I'll take it!" Dexter draws up the paperwork and completes the sale. Later that day, Paul goes out on the lake with his newly purchased boat. While driving at a normal speed, the boat's engine starts to run faster and faster and Paul is unable to turn the engine off. Paul jumps off the boat, into the water and is severely injured. Paul sues Dexter for breach of implied warranty. Who is likely to prevail? Paul, since the boat was not fit for its ordinary purpose. Paul, since he relied on Dexter's expertise in rehabbing boats. Dexter, since he was not in the regular business of selling boats. Dexter, since he did not provide an express warranty that the boat's engine would work.

C is the correct answer. The implied warranty of merchantability arises only if the seller of the product is in the business and regularly sells the kind of goods in question. --Here, Dexter is an accountant and not a merchant of boats. A is wrong because the implied warranty of merchantability arises only if the seller of the product is in the business and regularly sells the kind of goods in question. --Here, Dexter is an accountant and not a merchant of boats. B is wrong because it is not supported by the facts and the facts do not indicate that Dexter made any representations about his expertise in rehabbing boats. D is wrong since Paul is bringing a claim for implied warranty, not express warranty.

Question 10 2 / 2 pts Dexter is an accountant. He also is a boat-lover who, as a hobby, rehabs old boats in his spare time and sells them. Paul sees Dexter's ad for a rehabbed Z26 MusterCraft. Paul goes over to the local marina to look at Dexter's Mustercraft. He decides to buy the boat from Dexter and says "I'll take it!" Dexter draws up the peller of the product is in the business and regularly sellsaperwork and completes the sale. Later that day, Paul goes out on the lake with his newly purchased boat. While driving at a normal speed, the boat's engine starts to run faster and faster and Paul is unable to turn the engine off. Paul jumps off the boat, into the water and is severely injured. Paul sues Dexter for breach of implied warranty. Who is likely to prevail? Paul, since the boat was not fit for its ordinary purpose. Paul, since he relied on Dexter's expertise in rehabbing boats. Dexter, since he was not in the regular business of selling boats. Dexter, since he did not provide an express warranty that the boat's engine would work.

C is the correct answer. The implied warranty of merchantability arises only if the seller of the product is in the business and regularly sells the kind of goods in question. Here, Dexter is an accountant and not a merchant of boats. A is wrong because the implied warranty of merchantability arises only if the seller of the product is in the business and regularly sells the kind of goods in question. Here, Dexter is an accountant and not a merchant of boats. B is wrong because it is not supported by the facts and the facts do not indicate that Dexter made any representations about his expertise in rehabbing boats. D is wrong since Paul is bringing a claim for implied warranty, not express warranty.

Question 9 2 / 2 pts Don is a marine biologist. To aid in his study of black piranha fish, he installed a large tank in the backyard of his property in which he placed a pair of black piranhas. A black piranha is known to be one of the deadliest fish in the world, due to its large, razor sharp teeth. Due to a problem with the tank's filtration system, it started to emit a strong and offensive odor. Pam, Don's next door neighbor, came over to Don's property to find out the source of the odor. Don took Pam into the backyard to show her the fish tank. While Pam was standing next to the tank, the two piranha fish splashed water out of the tank, drenching Pam. Pam, who had a cold, developed pneumonia. If Pam asserts a claim against Don based on strict liability, she will likely: Recover, because Pam suffered an injury. Recover, because the possessor of a wild animal is strictly liable for damages the animal causes to other people. Not recover, because the dangerous aspect of the black piranha did not cause the harm. Not recover, because Pam stood next to the tank.

Choice C is correct because a possessor of wild animals is strictly liable for injuries caused by the animal, only if the injury results from the animal's dangerous propensities. Here, the piranha's dangerous propensities would be bites from its large, razor strong teeth, not from splashing water. A is wrong because Pam still would be required to prove that her injury resulted from the animal's dangerous propensities, not the case here. B is wrong as well because the injury did not result from the pirahna's dangerous propensity. D is wrong because it is irrelevant to strict liability.

Question 9 2 / 2 pts Peg purchased a bottle of liquid soap from ABC Market. The liquid soap had been manufactured by Sudsy Company. Sudsy Company had supplied the liquid soap to ABC Market with a tag attached to the neck of the soap dispenser that contained a warning that the "Soap should not be used by anyone who is prone to skin allergies". When ABC Market employees placed the liquid soap on the store shelves, most of the labels fell off the dispensers and the employees failed to replace them. The soap purchased by Peg did not have a warning tag. After using the product for a few days, Peg developed a serious rash on her hands and face as a result of an allergic reaction to a chemical in the soap. If Peg asserts a claim for product liability against ABC Market, a jury will likely find for: Peg, since ABC Market was negligent for failing to replace the warning tags on the soap dispensers Peg, since the soap was not manufactured to be used by anyone with skin allergies. ABC Market, since it did not manufacture the soap. ABC Market, since the soap was not manufactured to be used by anyone with skin allergies.

Correct Answer: A. A retailer can be negligent for selling a product that the retailer substantially alters, causing the product to be defective. Here, ABC market employees failed to exercise due care by not replacing the warning tags that had been affixed to the soap dispensers by the manufacturer. B is wrong since --Peg would have been aware of the intended proper use of the soap had ABC market not been negligent in removing the warning tags. C and D are wrong because --a retailer can be liable for injuries caused by a product manufactured by another entity if --the retailer acts unreasonably in altering the product to cause it to be defective when sold to a consumer.

Question 7 2 / 2 pts Peg purchased a bottle of liquid soap from ABC Market. The liquid soap had been manufactured by Sudsy Company. Sudsy Company had supplied the liquid soap to ABC Market with a tag attached to the neck of the soap dispenser that contained a warning that the "Soap should not be used by anyone who is prone to skin allergies". When ABC Market employees placed the liquid soap on the store shelves, most of the labels fell off the dispensers and the employees failed to replace them. The soap purchased by Peg did not have a warning tag. After using the product for a few days, Peg developed a serious rash on her hands and face as a result of an allergic reaction to a chemical in the soap. If Peg asserts a claim for product liability against ABC Market, a jury will likely find for: Peg, since ABC Market was negligent for failing to replace the warning tags on the soap dispensers. Peg, since the soap was not manufactured to be used by anyone with skin allergies. ABC Market, since it did not manufacture the soap. ABC Market, since the soap was not manufactured to be used by anyone with skin allergies.

Correct Answer: A. A retailer can be negligent for selling a product that the retailer substantially alters, causing the product to be defective. Here, ABC market employees failed to exercise due care by not replacing the warning tags that had been affixed to the soap dispensers by the manufacturer. B is wrong since Peg would have been aware of the intended proper use of the soap had ABC market not been negligent in removing the warning tags. C and D are wrong because a retailer can be liable for injuries caused by a product manufactured by another entity if the retailer acts unreasonably in altering the product to cause it to be defective when sold to a consumer.

Question 2 2 / 2 pts Dave loves computers. In his free-time, he hunts for computer parts, and then builds computers to sell to friends and family. One of his friends, Phil, approaches Dave and asks if he can buy one of Dave's computers that he "made from scratch." Dave agrees and sells the computer to Phil. Phil then uses the computer for his small business. A few months after buying the computer, Phil uses the computer and is severely injured when it catches fire. Phil realizes that the accident was caused when one of wires came out of the computer, and came into contact with a metal bolt, causing a fire. If Phil sues the manufacturer of the wire used in the computer based on strict products liability, he will probably: Lose, because the computer had been rebuilt by Dave. Lose, because there was no privity between Phil and the manufacturer of the wire. Recover, as long as the wire was new when it was put into the computer. Recover, because the computer was put into the stream of commerce by the manufacturer.

Correct Answer: A. As a general rule, a product manufacturer is not strictly liable for any defects that were not present when the product left the manufacturer's control. When Dave rebuilt the computer, from a products liability standpoint, the computer was no longer in the control of the manufacturer, making the manufacturer not liable for harm caused by the product. B is wrong because privity between the person injured by the product and the manufacturer is no longer required. C is wrong since the computer had been altered by Dave before it was used by Phil. D is wrong for the same reason.

Question 7 0 / 2 pts Durabrand Power Tools makes a stamping machine for metal fabricators. The machine does not have a guard to protect the operator's hands from entering the stamping area of the machine. Whenever the hand of an operator makes contact with the stamping area of the machine, it is likely that the operator will suffer serious injury to his hand from the stamping mechanism. Such a guard could be included, but it might reduce the speed of production. On the bed of the machine, directly in front of the operator, is a warning label, in large red letters: WARNING: KEEP HANDS FROM STAMPING AREA TO AVOID SERIOUS INJURY OR AMPUTATION! Paul, an operator who was using the machine, is distracted by a coworker. Paul turns his head and his hand drifts into the stamping area. Paul suffers severe injury to his hand. If Paul asserts a products liability claim against Durabrand based on a failure to warn, will Paul succeed? Yes, if the warning was an insufficient response to the danger posed by the machine. Yes, Durabrand should have known that a stronger warning was required. No, the warning was adequate since placement of a guard might have slowed production. No, Durabrand provided a warning.

Correct Answer: A. Paul will argue that a warning was an insufficient response to the danger posed by the machine. Durabrand should have eliminated the risk altogether by designing a guard that would prevent the operator's hands from entering the stamping area. Although a guard might have slowed production, under a risk/utility analysis this loss in production speed is likely outweighed by the increased safety. It is foreseeable — indeed, almost inevitable — that an operator's hands will end up in the stamping area. Therefore, the machine is probably defective since it lacks a guard. B is incorrect since a stronger warning likely would not have made the stamping machine reasonably safe, since it is likely that operator's hands still would have drifted into the stamping area of the machine. C is incorrect since on a risk utility basis, the risk of hand injury is significantly greater than the cost of slowed production due to placement of a guard. D is incorrect since although a warning was provided, it was not adequate to avoid the risk of harm. Thus, the court will almost certainly reject the argument that the warning is a proper substitute for a safer design.

Question 4 2 / 2 pts Pat liked to drive her new convertible manufactured by Open Motors. On a bright summer day, she was taking a drive on a local mountain road. When rounding a curve in the road, Pat's car was sideswiped by a car coming in the opposite direction. As a result of the collision, Pat, who was not wearing a seat belt, swerved out of her lane and suffered severe injury when she was thrown out of the vehicle. At the time of Pat's accident, it was widely known in the automobile industry that there was economically feasible technology that existed to prevent a car's engine from operating if the seat belt is not attached. Pat would not have been injured in the accident, had she been wearing a seat belt. Pat brings a strict products liability action against Open Motors, claiming that the convertible is defective. Is she likely to prevail? Yes, since the convertible had a manufacturing defect. Yes, since the convertible had a design defect. No, because Pat's negligence will bar her recovery completely in majority jurisdictions. No, the driver of the car that sideswiped Pat's car is solely liable for her injury.

Correct Answer: B is the correct answer. Since a reasonable alternative design existed that would have prevented Pat starting the car without wearing a seat belt, the car has a design defect. Had Pat been wearing a seat belt at the time of the accident, she would not have suffered injury. A is incorrect since the facts do not indicate that Pat's car differed from the intended design, required for a manufacturing defect. C is incorrect since Pat's negligence will not bar her recovery totally in the majority of jurisdictions, but may reduce her recovery. D is wrong since even if the other driver was negligent, the car manufacturer would still be liable for producing a defective car.

Score for this quiz: 24 out of 30 Submitted Nov 29 at 9:48pm This attempt took 30 minutes. Question 1 2 / 2 pts Patrick buys a ride-on snow blower to clear his driveway from Blow Inc., a snow blower retailer. He reads the instructions which state: "Proceed slowly at all situations. Full throttle should only be used to load or unload the blower from the garage." Patrick is in a rush to get to work one snowy morning, so he cranks the speed all the way up and rides around his property blowing everything in sight. As he nears the end of his driveway, Patrick tries to turn but the steering locks up and the blower overturns. Patrick is injured when he hits his head on the engine block. If Patrick sues Blow lnc. based on strict products liability, will he prevail? Yes, because the steering locked while Patrick was riding the blower. Yes, assuming that the steering locked because of a defect present when the blower left the factory. No, if Patrick worsened his injury by driving at full speed. No, if Blow Inc. carefully inspected the blower before selling it to Patrick.

Correct Answer: B is the correct answer. When dealing with a defective retail product, under strict liability, the retailer is liable for any injuries caused by a defect in that product sold by the retailer. Consequently, if a defect in the blower caused the steering to lock and Patrick's injury, Blow Inc. is liable. Blow Inc is liable under strict liability theory, even if it carefully inspected the product before selling it, making D incorrect. A is incorrect. The fact that the steering wheel locked up is not enough evidence to create liability, unless there is additional evidence to show that this malfunction was due to a manufacturing defect. C is wrong because Blow Inc is strictly liable for Patrick's injuries, whether or not he made the injury worse.

Question 5 2 / 2 pts Jane purchased a new washing machine manufactured by Washco. After the washing machine was installed in Jane's home, it began to make a loud, rattling noise whenever it was in the 'spin' cycle. Thinking nothing about the noise, Jane put her favorite, very expensive silk dress in the washer and during the spin cycle, the washer ripped her dress to shreds. If Jane brings an action for strict liability against Washco, is she likely to prevail? Yes, because she was the purchaser of the washing machine. Yes, because the washing machine was defective. No, because Jane should have not used a washing machine after it started making the loud, rattling noise. No, because Jane did not suffer any personal injuries.

Correct Answer: B. The correct answer is B since the washing machine apparently has a manufacturing or design defect that caused Jane to suffer damages (a destroyed dress). A manufacturer is strictly liable for harm caused by a defective product so long as the defect existed when it left the defendant's hand. Here, it appears that as soon as Jane began using the washer, it made a noise during the spin cycle, likely due to a defect. A is wrong because anyone using a product, including a purchaser can bring a products liability claim. However, the claim will not be successful unless the plaintiff can show that they were harmed by a defective product. C is wrong because Jane has brought her products claim under strict liability, making the manufacturer of the product liable as long as the plaintiff's harm is caused by the defective product. D is wrong because Jane is entitled to be compensated for damage to personal property caused by the defective product, not just for personal bodily harm.

Question 2 2 / 2 pts Dexter takes his motorcycle to Bob to buy a new engine. Bob installs a new engine - not designed to go above 200 MPH. The manufacturer knew, however, that some riders like to go to the extreme and take their bikes above 200 MPH so it built in a safety margin. The next year, Dexter sells the motorcycle to Paul. After purchasing the motorcycle, Paul takes it up to 220 MPH, the engine explodes, and Paul is severely injured. He sues Dexter for his injuries based on strict products liability. Who is likely to prevail? Paul, assuming the engine was defective when Dexter sold the bike to Paul. Paul, because he suffered an injury caused by the motorcycle's engine. Dexter, because he is a private seller. Dexter, because Paul misused the engine by speeding above its limit.

Correct Answer: C is the right answer. A strict liability claim relating to a defective product requires that the plaintiff show: 1) a strict duty owed by a commercial supplier; 2) breach of that duty by the sale of a product in a defective condition unreasonably dangerous to users; 3) actual and proximate cause; and 4) damages. At best, Paul can establish elements 2 and 3, but with no facts to indicate that Dexter is a commercial supplier. Therefore, Dexter will not be liable. A is wrong because there are no facts to indicate that Dexter is a commercial supplier. Therefore, Dexter will not be liable. B is wrong because suffering an injury from a product is only one of the elements of a products liability claim and Paul cannot prove the other elements. D is wrong since even though a manufacturer is liable for injuries caused by foreseeable misuse of their products, Dexter is not the manufacturer (commercial seller) of the motorcycle.

Question 10 2 / 2 pts Pharmaceuticals developed a new drug for depression, called Perzac. Pharmaceuticals tested the drug thoroughly for three years before placing it on the market, and discovers several side effects, but little evidence of any psychiatric side effects. It marketed Perzac, warning of several side effects but not of a 3% risk that users of the drug may suffer psychiatric side effects. Phil takes Perzac, suffers psychiatric side effects and brings suit against Pharmaceuticals , alleging it is liable for selling Perzac without a warning of the danger of psychiatric side effects. Pharmaceuticals argues that it is not liable. Will Pharmaceuticals succeed in its defense? Yes, since Pharmaceuticals warned of several side effects. Yes, since a warning is not required because very few users will likely suffer psychiatric side effects. No, if Phil had been warned of the risk of psychiatric side effects, he could have made an informed decision whether to run that risk or avoid the product. No, since the risks of Perzac justifies taking the drug off the market.

Correct Answer: C. When a drug is widely beneficial, yet presents risks to some users that cannot be eliminated, warnings give users the information they need to make choices about the product. If Phil had been warned of the risk of psychiatric side effects, he could have made an informed decision whether to run that risk or avoid the product. Because the drug had no warning of the risk of psychiatric side effects, Phil was unable to make that choice. A is wrong since Pharmaceuticals failed to warn about a significant side effect with would be material to potential users in their decision whether to assume that risk and take the medication. B is wrong since a warning is still required because the risk is significant. D is wrong since there are no facts to suggest that Perzac poses such unreasonable danger that outweighs the benefits of the drug. A manufacturer must be reasonable both in choosing to market a drug that poses risks, and in informing users about the risks it cannot eliminate from that product. See Restatement (Third) of Products Liability §2 cmt. k.

Question 14 2 / 2 pts Pamela was out on the lake in a row boat when she was struck and injured by Dexter who driving his speed motorboat at a very high rate of speed. Dexter was a 16-year-old who did not have a license to operate a motor boat. The police charged him with the crime of operating a water motor vehicle without a license. If Pamela sues Dexter to recover for her injuries, is it likely that she will prevail? Yes, she will recover if she can prove that Dexter did not act as a reasonable 16-year-old of similar age, experience, education, and maturity would have acted. Yes, she will recover because Dexter did not have a license. Yes, she will recover, but only if Dexter was exceeding the posted speed limit. Yes, she will recover if Dexter did not act as a reasonable person would have in the circumstances.

D is correct since Dexter, in driving a motorboat, would be held to the standard of conduct for a reasonable person under the circumstances, despite his minority. For that reason, A is incorrect. B is incorrect since the lack of a license was not a cause of the injury. C is incorrect since Dexter would be liable if he was driving negligently, whether or not he violated a statute.

Question 11 15 / 15 pts Pamela was out on the lake in a row boat when she was struck and injured by Dexter who driving his speed motorboat at a very high rate of speed. Dexter was a 16-year-old who did not have a license to operate a motor boat. The police charged him with the crime of operating a water motor vehicle without a license. If Pamela sues Dexter in tort to recover for her injuries, is it likely that she will prevail? Yes, she will recover if she can prove that Dexter did not act as a reasonable 16-year-old of similar age, experience, education, and maturity would have acted. Yes, she will recover because Dexter did not have license. Yes, she will recover, but only if Dexter was exceeding the posted speed limit. Yes, she will recover if Dexter did not act as a reasonable person would have in the circumstances.

D is correct since Dexter, in driving a motorboat, would be held to the standard of conduct for a reasonable person under the circumstances, despite his minority. For that reason, B is incorrect. A is incorrect since the lack of a license was not a cause of the injury. C is incorrect since Dexter would be liable if he was driving negligently, whether or not he violated a statute.

Question 6 2 / 2 pts Diane is driving and her dog is in the back seat distracting her. Diane drives her car into Paul, a pedestrian crossing the street. Paul is taken to the hospital and the doctor treats him for some cuts and releases him without noticing that Paul is suffering from a serious concussion. As a result of the doctor's failure to treat Paul properly for the concussion, Paul suffers more than he otherwise would have suffered. Paul sues Diane for the harm caused including the prolonged concussion. Diane claims that she should not be responsible for the harm resulting from the doctor's negligent treatment. The most likely outcome is: Diane will not be liable because she had no control over the doctor's conduct, which broke the chain of causation. Diane will not be liable because the doctor's conduct was not a foreseeable cause of Paul's injury. Diane will be liable because the doctor's conduct was a superseding cause of Paul's injury. Diane will be liable because her conduct was the proximate cause of Paul's injury.

D is correct since a doctor who negligently treats a patient causing harm beyond the scope of the original harm caused by the original tortfeasor is generally regarded as foreseeable, and the doctor's conduct did not break the chain of causation. Therefore Diane was the proximate cause of Paul's injury. Although the doctor's conduct was an intervening cause, it was not a superseding cause. A doctor who negligent treats a patient causing harm beyond the scope of the original harm caused by the original tortfeasor is generally regarded as foreseeable, and the doctor's conduct did not break the chain of causation. Therefore Diane was the proximate cause of Paul's injury. Although the doctor's conduct was an intervening cause, it was not a superseding cause. A , B and C are incorrect for the same reason.

Question 4 2 / 2 pts Passengers on a ship arriving in the L US are told they cannot leave the ship unless they have been vaccinated for a specified virus. Lane, a passenger on the ship, indicates that he has been vaccinated before, but is told that he will need to be vaccinated again. Lane waits in line with the other passengers. When his turn comes, Lane holds out his arm, and the doctor vaccinates him. Lane sues the doctor and ship for battery. How should the court rule? Lane wins, because the vaccination is a harmful and offensive touching. Lane wins, because the vaccination was intentional. Lane loses, because the vaccination was justified. Lane loses, because he consented by holding out his arm.

D is the best answer because Lane's conduct shows his consent to the vaccination. A is incorrect because although there is a battery, Lane gave consent through his conduct. B is incorrect, because although the conduct was intentional, Lane gave consent. C is incorrect because the defense of justification is available when none of the conventional defenses are applicable, and here, the conventional defense of consent is applicable.

Question 3 0 / 2 pts Matthew walked into a bar with a baseball bat after he heard that Hal was dating Matthew's girlfriend. Matthew shouted, "I'm going to kill you Hal!" Matthew headed in Hal's direction. Hal was working behind the bar. Owen, the owner of the bar, yelled at Matthew, "Back off dude, or you're going down." Matthew continued in Hal's direction. Owen tackled Matthew and threw Matthew to the ground. Owen took the baseball bat out of Matthew's hands. If Matthew sues Owen, Owen is liable for: Battery of Matthew for tackling Matthew and throwing him to the ground. Assault of Matthew for threatening to harm him. Battery and assault of Matthew. None of the above.

D is the best answer because Owen has a privilege to defend others, as long as he is using reasonable force. Here Owen defended Hal from Matthew's threats, using reasonable force, and therefore will not be liable for either battery or assault.

Question 5 2 / 2 pts Dan tried to shoot Pat in the back of the head. Dan snuck up on Pat, and aimed the gun at Pat's head. Trish saw Dan, and pushed the gun away. Pat then saw what Dan had been trying to do, and became hysterical. Dan fled the scene, and Trish tried to calm Pat down, assuring Pat that he was okay. What tort claims can Pat assert against Dan? Assault, battery and intentional infliction of emotional distress. Battery and intentional infliction of emotional distress. Assault and battery only.! None of the above.

D is the best answer because Pat did not see the gun, since Dan was aiming to shoot Pat in the back of the head, so there is no assault. There is no contact, so there is no battery, and there was no intent that would support intentional infliction of emotional distress because the doctrine of transferred intent is not applicable.

Question 9 2 / 2 pts Daniel owns a bar. Walter, one of the regulars, arrived alone by car. Daniel served Walter six shots of tequila in an hour and Walter was obviously drunk when he left the bar. Walter ran a red light and caused an accident with Peter, resulting in serious injury to Peter, who sued Daniel for negligence. Walter's conduct in driving drunk is: An unforeseeable intervening cause of Peter's injury because driving drunk is a crime. A superseding cause of Peter's injury, because Walter acted negligently. The only cause in fact of Peter's injury. A foreseeable intervening cause of Peter's injury.

D is the best answer because generally negligence is foreseeable as an intervening cause, and even criminal conduct, her drunk driving can be foreseeable, and probably is foreseeable here. A is incorrect because Walter's conduct is foreseeable. B is incorrect because Walter's conduct is not a superseding cause. C is incorrect because both Daniel's conduct and Walter's conduct are causes in fact of Walter's harm.

Question 3 2 / 2 pts Husband is sometimes a distracted driver who frequently drives over the speed limit. While texting and driving at high speed, Husband looks up suddenly to see a pedestrian in his path and swerves to miss the pedestrian, hitting a pole. The pedestrian fell when jumping out of the way, but was not injured. Wife and Child, who were in the car with Husband at the time, both suffered injuries. Under common law: Wife and Child can bring suit against Husband to recover for their injuries. Only Wife can bring suit against Husband to recover for their injuries. Only Child can bring suit against Husband to recover for their injuries. Both Wife and Child are barred from bringing suit against Husband to recover for their injuries.

D is the best answer under common law, based on spousal immunity and parent/child immunity. Note that modernly most states have abolished both types of immunity. Accordingly, A, B and C are incorrect under common law.

Question 10 2 / 2 pts Barbara was cooking when she splashed oil onto her stove starting a fire. She dialed 911 and the fire department sent out a fire engine. Frank, a firefighter, arrived to put out the fire. In the process, Frank was injured when a ceiling fan nearby gave way hitting him in the head. Which of the following is correct under the majority view? Frank is an invitee and Barbara owed him a duty to inspect her property, make it safe, and warn him. Frank is an invitee and Barbara owed him a duty to inspect the property and make it safe, but had no duty to warn him. Frank is a licensee and Barbara owed him a duty to inspect her property and make it safe. Frank is a licensee and Barbara owed him a duty to warn of known dangers.

D is the best answer under the firefighter's rule, which provides that firefighters are precluded in some circumstances from recovery for injuries on the job, and generally treats them as licensees for whom owners have a duty to warn of known conditions. A is incorrect because a firefighter is not an invitee under the majority view. B is incorrect for the same reason. C is incorrect because an owner does not owe a duty to inspect to a licensee. Quiz Score: 20 out of 20

Question 5 0 / 2 pts Pam buys a set of oversized wheels for her pickup truck. The tires bear a label stating that they should not be installed on a truck larger than a certain size. Pam installs the tires on her truck, even though it is larger than the size specified on the tire label. Pam is injured when the truck rolls over in a sudden stop. If Pam brings a products liability action against the manufacturer of the tires, claiming that there was a failure to warn, will she likely succeed? No, because there was an adequate warning. No, because Pam installed the tires herself. Yes, since Pam was injured when the truck rolled over. Yes, because there was no adequate warning of the roll over danger.

D is the best answer. The notice on the tires is clearly inadequate, and arguably is not a warning at all -- it's an instruction about how to use the product. It is frequently insufficient to simply instruct consumers not to use a product in a particular way. A true warning should inform the consumer of the nature and extent of the risk posed by ignoring the instructions. To really affect consumers' behavior, a warning must convey the nature and gravity of the risk of ignoring instructions. ''Warning! Use of these tires on trucks heavier than 2,000 pounds may cause serious injury or death from rollover accidents.'' would be an obviously more effective warning. A is wrong since the warning was inadequate because it does not mention the roll over risk. B is wrong since the warning did not alert Pam of the risk of harm that she ultimately suffered. C is wrong since injury alone does not establish liability unless the harm can be causally related to the failure to warn of the harm.

Question 10 2 / 2 pts Phillip has recently taken up mountain climbing. He goes to his local sporting goods store and tells David that he needs a pair of shoes for a planned climbing trip to Browback Mountain. David recommends that Phillip buy Brand Z shoes because they have great traction and are designed for trips to mountains similar to Browback. Phillip, relying on David's recommendation, purchases Brand Z shoes. During his climb up Browback Mountain, Phillip's Brand Z shoes lose traction and he falls 50 feet down the mountain, suffering severe injuries. If Phillip sues David for his injuries, what is Phillip's strongest argument? The Brand Z shoes have a manufacturing defect. The Brand z shoes have a design defect. David breached the implied warranty of merchantability. David breached the implied warranty of fitness for a particular purpose.

D is the best answer. The warranty of fitness for a particular purpose arises when the seller knows that the buyer wants the goods for a particular (not customary purpose) and the buyer relies on the seller's judgment to recommend a suitable product. Here, David recommended the Brand Z shoes for climbing Browback Mountain and Phillip relied on his recommendation. The shoes, however, did not provide the traction represented by David. A and B are wrong since there are no facts to indicate that the shoes were defective, although they may not have been of good quality. C, while a feasible argument, is not the best answer. The facts indicate that Phillip provided specific information to the seller, David, about the purpose to which he intended to use the product.

Question 11 2 / 2 pts Ron put a hidden camera in the dormitory room of his next door neighbor, Ned, when Ned was out of town for the weekend. The camera captured images of Ned sleeping and snoring loudly. Ned's snoring was not only loud, but very strange sounding, due to a childhood injury to his nose. Because of this Ned was allowed to room alone. Ron posted some of the video of Ned snoring in bed on the internet with the caption "Freakish Loud Snoring," and the video went viral, generating millions of hits online. Ron ended making money off the video because he had pop up ads that came up in connection with the video. If Ned sues Ron for invasion of privacy, he can assert: False light, because the video is embarrassing to Ned. Misappropriation of Ned's image. Public disclosure of private facts. Intrusion upon seclusion, because Ned's snoring is a private matter. All of the above. I and III only. II and III only. III and IV only.

D is the best answer. There is no false light claim because the video is not false. Misappropriation of image is typically actionable where the image of another is used to promote a product, not where it is used to make money for a publication. Public disclosure of private facts is available here because making a person's snoring public would be highly offensive to a reasonable person. Intrusion upon seclusion is available because Ned has a private room due to his snoring, and the camera intruded on this set up.

Question 2 2 / 2 pts Donna needed to remove a tree stump from the back yard of her home in a suburban neighborhood. She bought some dynamite, set it in holes under the tree stump and blasted away. Donna had been trained to use dynamite in the Marine Corps and was an expert in its use. When she was engaged in blasting the tree stump out, she was very careful to follow all of the proper guidelines in the use of dynamite. Even with all of that care, however, her neighbor, Pam, complained. The shock waves from the blasting had cracked the foundation of Pam's house. If Pam asserts an action against Donna, which of the following is true? Since Donna exercised due care in removing the stump, she will not be liable. Since the use of dynamite is common for blasting, strict liability will not apply. Since blasting is common for stump removal, strict liability will not apply. Since blasting is uncommon and creates a high risk that cannot be mitigated, strict liability will apply.

D is the correct answer since blasting is considered to be an abnormally dangerous activity, since it cannot be conducted with guaranteed safety. For that reason and because blasting a tree with dynamite in a suburban neighborhood would not reasonably be a common activity, strict liability would apply to any personal injury or property damage resulting from the activity. A is incorrect since strict liability applies to abnormally dangerous activities the risks of which cannot be eliminated by the exercise of reasonable care. B and C are wrong since blasting a tree with dynamite in a suburban neighborhood would not reasonably be a common activity.

Question 6 2 / 2 pts Peter purchased a Tetra car, advertised as a safe car. Peter was driving his Tetra car down the highway when traffic suddenly stopped ahead of him. Denny, who was driving behind Peter, was unable to stop before hitting Peter's car in the rear. When Denny hit Peter's car, it was pushed forward and the back of Peter's head hit the head rest. The headrest was built of padding around a central metal core. As Peter's head hit the rest, the padding compressed, allowing the front portion of the metal core to concentrate pressure on the back of Peter's head to a small area, causing a fracture to his skull and first vertebra. Had the metal core been broader and flat, it would have spread the pressure over a wider area and caused no damage to Peter. Peter is paralyzed from the neck down as a result. Peter sues the car maker, Tetra, on a strict products liability theory. On the facts set forth above, the head rest has which of the following defects? A manufacturing defect A warning defect A marketing defect, since the Tetra was advertised as a safe car A design defect

D is the correct answer. A design defect is found when the product is made unreasonably dangerous to the user and there is a reasonable alternative design that could have made the product safe. Here, the metal core of the headrest could have been broader and flat, spreading the pressure over a wider area and causing no damage to Peter. A is incorrect because a manufacturing defect is found when the product is not made as intended, not the case here. B is incorrect because a there are no facts stating that Tetra knew or should have known about the defective head rest and failed to warn about it. C is wrong because unless Tetra knew or should have known about the defective head rest and failed to warn about it, they would not be liable for marketing the car as 'safe'.

Question 2 0 / 15 pts Adam's car sustained moderate damage in a collision with a car driven by Bob. The accident was caused solely by Bob's negligence. Adam's car was still drivable after the accident. Examining the car the next morning, Adam could see that a rear fender had to be replaced. He also noticed that massive amounts of gasoline had dripped onto the garage floor. The collision had caused a large leak in the gasoline tank. The gas leakage was not apparent at the accident scene the prior day. Adam then took the car to Mechanic, who owns and operates a body shop, and arranged with Mechanic to repair the damage. During their discussion Adam neglected to mention the gasoline leakage. Thereafter, while Mechanic was loosening some of the damaged material with a hammer, he caused a spark, igniting vapor and gasoline that had leaked from the fuel tank. Mechanic was severely burned. Mechanic has brought an action to recover damages against Adam and Bob. The jurisdiction has adopted a pure comparative negligence rule. In this action, will Mechanic likely obtain a judgment against Bob? Yes, unless Mechanic was negligent in not discovering the gasoline leak himself. Yes, since the damage to Adam's car was caused by Bob's negligence. No,since Bob did not violate a motor vehicle statute. No, if Mechanic would not have been harmed had Adam warned him about the gasoline leak.

D is the correct answer. Adam's failure to warn Mechanic of the leak was the actual cause of Mechanic's injury. It was not foreseeable that Bob's negligence would have caused burns to the Mechanic. A is wrong since the mechanic's negligence, if any, would not bar his claim in a pure comparative negligence jurisdiction. B is incorrect since Mechanic's injury was not a foreseeable result of Bob's negligence. C is wrong since violation of a statute by Bob is irrelevant.

Question 9 2 / 2 pts Pat was walking home from a late night in a bar, holding a bottle of Foamy beer. Pat threw the bottle against a telephone pole, and the bottle shattered, injuring Pat's eye. Pat brings a strict products liability action against the beer bottle manufacturer, claiming that the bottle was defective. Is he likely to prevail? Yes, because his eye injury was caused by the beer bottle. Yes, because it is foreseeable that someone would throw a beer bottle. Yes, because there was no warning on the bottle stating that it should not be thrown. No, since Pat cannot establish that the bottle was defective.

D is the correct answer. He cannot establish that the product was defective, since he cannot show that the bottle was used for a purpose the manufacturer intended or should have foreseen. Such a use is ''so unusual that the average consumer could not reasonably expect the product to be designed and manufactured to withstand it.'' A is wrong since Pat still cannot prove that the injury was caused by a defect in the bottle. B is wrong since he cannot show that the bottle was used for a purpose the manufacturer intended or should have foreseen. Such a use is ''so unusual that the average consumer could not reasonably expect the product to be designed and manufactured to withstand it.'' C is wrong since manufacturers are not required to warn about open and obvious dangers.

Question 13 2 / 2 pts A twelve-year old boy suddenly ran out into the street chasing a basketball, in the path of a car being driven by the Dan. Since Dan was speeding in violation of a city vehicle code, he was unable to stop in time to avoid hitting the boy. The boy suffered an injured leg in the accident. The jurisdiction applies the all-or-nothing rule of contributory negligence. If the boy asserts a claim against the Dan for the injuries to his leg, which of the following would be Dan's most effective argument in defense? Dan had a clean driving record, with no infractions. Dan's violation of the city's vehicle code was not the proximate cause of the boy's injury. Dan was only driving 5 miles above the speed limit. The boy was negligent for running out into the street to chase a basketball.

D is the correct answer. In a jurisdiction applying the contributory negligence doctrine, any negligence on the part of the plaintiff would bar his action. Consequently, choice D is the only answer that would provide a successful defense.

Question 11 2 / 2 pts Debbie is jogging, when a chartered bus swerves onto the sidewalk where she is jogging and heads straight for her. The bus driver is so preoccupied with the passengers in the bus that he did not realize that the bus has gone off the road. Debbie tried her best to get out of the way in time and at the last minute she jumped into her neighbor Pam's yard. Once she landed on the ground, Debbie realized that she was lying in a patch of Pam's prize-winning pumpkins, and had essentially killed an entire section of the pumpkin patch. Pam asserts an action against Debbie for damages to her pumpkins. How will the court likely rule? Debbie is liable as she had no privilege to enter Pam's yard. Debbie is liable because she failed to act with due care. Debbie was privileged to enter Pam's land, therefore she is not liable. Debbie is liable for the damaged pumpkins.

D is the correct answer. The privilege of necessity states that a person can interfere with the property of another to reasonably avoid threatened serious injury from a natural act or the act of another if the threatened act is substantially more serious than the invasion of the party. When dealing with private necessity (i.e., an act that only benefits one person) the actor remains liable for any damage they cause, even though they may be privileged to enter. A is partly right, as it does correctly state that Debbie is liable for the damage. It incorrectly states, however, that she was not privileged to enter Pam's yard. B is incorrect. Exercise of due care is meaningless since the claim against Debbie will be one of intentional tort, such as a trespass or conversion. C is wrong. The fact that Debbie has privilege does not mean that she is not liable, as set forth in the correct answer.

Question 13 2 / 2 pts Donna and Phyllis were standing next to each other on the subway. Without notice, the subway car comes to a screeching stop due to a dog wandering on the tracks and they both lost their balance. Phyllis is wearing 8-inch platform heels and is about to fall when Donna grabs Phyllis by the waist to stop her fall. Phyllis sues Donna for battery. Is she likely to recover? Yes, if Donna intended to put her arm around Phyllis' waist. Yes, this was a touching without consent or permission. No, assuming Donna put her arm around Phyllis' waist by accident. No, this was socially acceptable conduct by Donna.

D is the correct answer. a battery requires a harmful or offensive touching. This was not the case here. Battery requires the following elements: 1) an act by the defendant that brings about harmful or offensive contact to the plaintiffs person; 2) intent on the part of the defendant to bring about harmful or offensive contact to the plaintiff's person; and 3) causation. Donna trying to keep Phyllis from falling is neither offensive nor harmful. Helping someone stay upright is done with an implied consent based on custom and is not offensive, making D correct. A and B are both wrong. Even if Donna intended to put her arm around Phyllis' waist, there was no harmful or offensive contact and, therefore, her conduct does not meet the elements of battery. C is wrong. Even if the placing of her arm around Phyllis' waist was not by accident, the harmful or offensive touching required by a battery is still lacking.

Question 15 2 / 2 pts Raul owns an apartment building in an urban area. A city ordinance requires that all common stairways in apartment buildings be illuminated by artificial lighting after 5:00 p.m. One evening, at 5:30 p.m., Clark, an employee of a local pizza parlor, was delivering a pizza to the apartment of one of Raul's tenants. As Clark climbed a staircase on the outside of the building, he tripped, fell, and broke his ankle. The staircase contained a light fixture but the bulb was broken. Clark has sued Raul for his injuries. Which of the following facts would be most helpful to Raul in defense of his case? Raul inspected the light two weeks prior to the accident and it was working. No one else had ever fallen on the stairs before. The sun set at 8:00 P.M. on the day of the accident. Raul's apartment building had a sign at the entrance prohibiting solicitors from entering.

The best answer is C because it would remove the broken light as a cause of the injury. A landowner owes a duty to an invitee to --inspect, --make safe or warn of any dangerous conditions and --to control third persons on the property. B is irrelevant to the duty of a landowner. A would be a helpful fact, but would still leave room for argument as to how frequently inspections should occur. D would make Raul a trespasser, but most likely an anticipated trespasser, so that there would still be a duty to warn and make safe as to any artificial, hidden highly dangerous conditions, though it may be debated whether this condition is "hidden." Quiz Score: 30 out of 30

Question 8 2 / 2 pts Pat is walking down the street when he is hit on the head with a barrel falling out of David's warehouse window. Pat brings an action against David for negligence. Pat does not put on any evidence showing how the incident occurred or what precaution David could have taken to prevent it. Will Pat likely prevail on the basis of res ipsa loquitur? Pat likely will prevail if David does not put on evidence blaming someone else for the accident. Pat likely will prevail if the jury determines these kinds of incidents ordinarily do not occur in the absence of negligence of a person like David. Pat likely will not prevail absent direct proof of how the barrel fell out of the window. Pat likely will not prevail because he did not put on evidence that he was not at fault.

The facts are based upon the classic Byrne v. Boadle case. B is the correct answer since res ipsa loquitur allows an inference of negligence based upon circumstantial, rather than direct, proof of negligence. A and C are wrong since Res ipsa loquitur allows an inference of negligence based upon circumstantial, rather than direct, proof of negligence. D is wrong since juries can use common sense to infer that the plaintiff asserting res ipsa loquitur was not negligent, causing their own injuries.

Question 14 0 pts Bob, a professional photographer, went to Department Store to purchase some equipment. As he was leaving the store, he noticed that Patty, a well-known Hollywood starlet, was browsing through the women's clothing department. Bob took a picture of Patty without her knowledge. Several days later, Bob took the photograph to the manager of Department Store and sold him the picture, explaining that Patty had agreed that Department Store could use the photograph in an advertising campaign. The manager enlarged the photograph and hung it above the main entrance to Department Store with a caption that read, "Patty Shops Here." One month earlier, Patty had entered into a contract with Hardware Store, the terms of which provided that Hardware Store had the exclusive right to use Patty's name and likeness for advertising purposes. As a result of the photograph's appearing at Department Store, Hardware Store canceled its contract with Patty. If Patty asserts a claim based on invasion of privacy against Department Store, will Patty likely prevail? Group of answer choices Yes, because Department Store, without Patty's permission, used Patty's picture for profit. Yes, because Bob had no right to take Patty's picture. No, because Department Store believed it had permission to display Patty's picture. No, because Patty is a public figure.

(A.) Where plaintiff is well-known, so that her name and likeness have commercial value, using plaintiff's name or likeness for commercial purposes without permission is an invasion of privacy. (B) is wrong. There is generally no violation of one's right of privacy when her picture is taken in a public place. (C) is wrong. In the absence of actual consent or some other recognized privilege, the appropriation is actionable. Department Store's good faith belief that consent was obtained is no defense. (D) is wrong because Patty's status as a public figure bolsters this type of invasion of privacy charge (commercial appropriation) rather than diminishes it.

Question 10 0 pts Pippin and Mary were going through a divorce. Mary hired Dino, a retired detective, to spy on Pippin. Dino followed Pippin to the Hotel, where he saw Pippin meet a woman and go into her hotel room. Dino checked into the adjoining room, placed an electronic listening device on the wall, and listened to the activities of Pippin and the woman in the next room. A state statute makes adultery a crime. If Pippin sues Dino for invasion of privacy: Group of answer choices Pippin will win, unless Dino's electronic eavesdropping was legal. Pippin will win, because he had an expectation of privacy in the woman's hotel room. Pippin will lose, because adultery is illegal. Pippin will lose if Dino did not publish anything about Pippin's activities.

(B.) Dino is liable for the type of invasion of privacy of intrusion upon plaintiff's private affairs or seclusion. Pippin could reasonably expect that what he did in the woman's hotel room would be free from intrusion. Dino violated Pippin's seclusion by listening to the activities in the room. This intrusion would certainly be objectionable to a reasonable person. (D) is wrong because the viability of this cause of action does not depend upon publication by the defendant; the interest protected by this type of invasion of privacy is the plaintiff's right to be let alone, rather than his interest in not having the information disseminated. (A) is wrong because illegality of the defendant's actions is not an element of the prima facie case for this form of invasion of privacy. Even if the eavesdropping is legal, Dino's conduct would still be an objectionable intrusion upon Pippin's private domain. Finally, the fact that Pippin may have been engaging in illegal adulterous conduct does not justify a private individual's intrusion on his solitude. Thus, (C) is incorrect.

Question 2 0 pts Paul is a partner at an accounting firm. One night, the police call Paul to inform him that his teenage daughter, Amy, was busted with 5 grams of cocaine on her possession. Paul heads to the jail to bail Amy out. Once he arrives at the station, a reporter from the local tabloid snaps a picture of Paul standing with his hands behind his back next to two police officers. In actuality, Paul was simply reaching for his wallet, but the photo looks like he was under arrest. The tabloid prints the photo with a caption implying that Paul was under arrest. Paul's accounting firm sees the photo and, because they are fearful of losing some of their conservative clients, they vote Paul out of the partnership in an emergency partner meeting. If Paul sues the tabloid, will he likely recover? Group of answer choices Yes, for intrusion upon seclusion. Yes, for false light publicity. No, because the caption was true. No, because printing the picture was in the public interest.

(B.) False light requires the following elements: 1) publication of facts about the plaintiff by the defendant that places the plaintiff in a false light in the public eye; and 2) the "false light" must be something that would be objectionable to a reasonable person under the circumstances. The large picture on the front page, tends to imply that Paul is actually the one being taken into custody. This "false light" would clearly be objectionable to a reasonable person under the circumstances. (A) is wrong as the elements of this tort are not met. Intrusion upon a plaintiffs affairs or seclusion, requires; 1) an act of prying or intruding into the affairs or seclusion of the plaintiff by the defendant; 2) the intrusion must be something that would be objectionable to a reasonable person; and 3) the thing to which there is an intrusion or prying must be "private." Here, the photo was snapped at the police station, a public place, making the intrusion into Paul's private domain inapplicable (C) is wrong as the implied fact that Paul actually committed a crime was not adequately addressed by including this small caption in the paper. The caption fails to serve as valid defense from suit for the tabloid. (D) is incorrect. Printing a picture such as this is not likely to be construed as being in the public interest. Paul is simply an accountant (not a public official or a public figure of any sort) and a father posting bond for his daughter's cocaine possession charge is generally not newsworthy public interest reporting.

Question 9 0 pts Polly works for an oil and gas company. She is world renowned for finding gas in Alaska. One day after being on a rig for 10 years, Polly returns to her hometown to find a huge billboard with a photo of her that reads: "Smoke Sucker Cigarettes. They won't even kill you." Polly is outraged, as she is against smoking and would never want to be associated with the industry. She is certain that the picture is of her, even though she never went to a photo shoot, as her ex-boyfriend happens to be the owner of Sucker Cigarettes. Looking closely, she realizes that the photo has been "photoshopped" from her college graduation picture. It's her head, but the body belongs to someone else. If Polly sues Sucker Cigarettes, her most effective best cause of action would be: Group of answer choices Defamation Invasion of privacy . Intentional infliction of emotional distress Negligent infliction of emotional distress

(B.) The most applicable cause of action is invasion of privacy, as related to commercial appropriation. Note: There is no defamation at issue, and we have no facts indicating that Polly suffered any sort of emotional distress or harm. Therefore, Answers (A), (C), and (D) are all incorrect.

Question 8 0 pts Pat Smith works for the LAPD as a detective and had received numerous awards and given several TV interviews about some of his most interesting cases. He decides to retire and leaves the force to play golf. One day his golf game is rained out and as he is flipping channels, he comes across "LA Law" a docu-drama about the detective trade in LA. After the initial credits, Pat sees the following: "based on the trials and tribulations of detective Pat Smith's life while serving on the LAPD police force. Please be aware that this is a stylized documentary and things may appear in a manner slightly different than they actually occurred." Pat watches the show and becomes angry, as the plot includes a sordid inter-office affair with a dispatcher that never occurred in Pat's real life. Assume that Pat sues the TV broadcasting company for invasion of privacy. Who will prevail? Group of answer choices Pat, since the broadcasting company used his name for a commercial purpose. Pat, since his retirement and the associated right to seclusion has been upset. Pat, as he was portrayed in a false light. The broadcasting company, as the show when taken as a whole, was complimentary to Pat's reputation.

(C.) Invasion of privacy based on a publication of facts that place the plaintiff is a false light, requires a showing of the following elements: 1) publication of facts about the plaintiff by the defendant placing the plaintiff in a false light in the public eye; 2) the "false light" must be something objectionable to a reasonable person under the circumstances; and 3) malice on the defendant's part where the published matter is in the public interest. Facts will be found to place a plaintiff in a false light if they showcase: 1) views that the he does not hold, or 2) actions that he did not take. The alleged sexual affair with a dispatcher did not take place and would be objectionable to a reasonable person under the circumstances. This meets the requirement for false light invasion of privacy. Note: Malice is not required, as the TV show does not exist to benefit the public's best interest. (A) is incorrect. The case for invasion of privacy based on the use of a person's name, requires a showing of unauthorized use by the defendant of their picture or name for commercial advantage. Typically, this is the use of a name or image in the sale of a product by the defendant. Here, Pat's name is only in the credits and no facts indicate that it was used to boost sales etc. Therefore, the element of invasion of privacy is not met. (B) is incorrect. The case for invasion of privacy based on an intrusion upon the plaintiffs affairs or seclusion needs proof of one the following: 1) the act of prying or intruding upon the affairs or seclusion of the plaintiff by the defendant; 2) the intrusion must be something that would be objectionable to a reasonable person; or 3) the thing to which there is an intrusion or prying must be "private." Here, there are no facts indicating that the broadcasting channel invaded Pat's private affairs by creating sex scenes that he objected to. Also note that there is no exception to the rule in regard to retirement. The facts form the show were pulled from the private domain, namely, facts from Pat's stellar detective career. (D) is incorrect because if Pat's privacy has been invaded, then the defense that the program as a whole was complimentary is not valid. The broadcasting company can advance it, but it is clearly a losing argument. Invasion of privacy torts do not have a balancing test such as this attached to them.

Score for this quiz: 18 out of 30 Submitted Sep 6 at 9:15pm This attempt took 25 minutes. Question 1 3 / 3 pts Charlotte and Pam are at a shooting range. Charlotte insults Pam for "shooting like a girl" and both Pam and Charlotte begin fighting. The range marshal breaks up the fight. While in the parking lot, Charlotte grabs a knife and throws it at Pam's face, striking her in the eye. While at trial for battery, Charlotte testifies that Pam approached her in the parking lot and said "this is going to end now," and grabbed what Charlotte thought was a pistol. Charlotte claims that she then threw the knife at Pam in self-defense. The marshal happened to walk out to the parking lot immediately after this incident. He testifies that Pam was holding a glasses case about the size of a gun in her hand while she lay in a pool of blood and he thinks a reasonable person might have thought the case was a gun, since it was about the same size as a small pistol. Pam's guns actually were in the trunk of her car. If Pam brings an action against Charlotte for her injuries, who will likely prevail? Charlotte, if a reasonable person in a similar situation would believe that she was about to be shot. Charlotte, assuming she believed Pam was about to shoot her. Pam since she was not armed. Pam, since she was not the aggressor.

A is correct because an act committed is self-defense is permissible when there is either a real danger or a reasonable expectation of a danger, and a reasonable person would have acted the same under the circumstances. B is wrong. Note: The test here is an objective test. Charlotte honestly believing that she was about to be shot is not enough, and we will evaluate what a reasonable person would have believed. C is wrong in light of the correct answer since either a real threat, or a reasonably perceived threat, is enough to allow action in self-defense. As the glasses case may have been mistaken for a gun, and a reasonable person could have viewed it that way, Charlotte is justified in acting as she did. D is wrong because the facts presented here are not clear in identifying who was the true aggressor (Charlotte or Pam). Therefore, D might be a misstatement of the actual facts at issue. Additionally, D fails to address Charlotte's right (or lack thereof) to use deadly force.

Score for this quiz: 24 out of 30 Submitted Aug 30 at 8:11pm This attempt took 22 minutes. Question 1 3 / 3 pts Peter gives his Homeowner's Association permission to build a snow fence on his property parallel to a nearby road. The Homeowner's Association agrees that the fence will be removed at the end of the winter. When winter ended, the Homeowner's Association removed the fence, but left a stake that had secured the fence sticking up in the dirt. Peter walked to his mailbox next to the road and cut his foot on the stake, suffering a serious injury. If Peter asserts a claim for trespass against the Homeowner's Association, what result? Peter will prevail because the Homeowner's Association failed to remove the stake. Peter will prevail because the fence was placed on his land. The Homeowner's Association will prevail because it had Peter's consent to place the fence on his land. The Homeowner's Association will prevail because there was no damage to Peter's property.

A is correct because trespass occurs where the defendant is authorized to put on object on plaintiff's land, but then refuses or neglects to remove it when it is supposed to be removed. This is the case of Rogers v. Bd of Road Commissioners. B is wrong because Peter consented to the placement of the fence. C is wrong since the object remained on Peter's land beyond the time period of the consent. D is wrong because the trespasser can be liable for physical damage to the landowner.

Question 5 15 / 15 pts Paul and Dan were both avid football fans. One day, they were having a heated argument about which football team was going to win the Super Bowl, when Paul said to Dan, "Let me show you how tough my team can be. Do you want to fight?" Dan replied, "You bet I do. I'll show you who's tougher!" As Paul and Dan began fighting each other, their anger escalated and Dan pulled out a knife and stabbed Paul. If Paul asserts a claim against Dan for his injuries, who should prevail? Paul, since Dan's use of a knife exceeded consent under the circumstances Paul, since he suffered an injury Dan, since he was only acting in self-defense Dan, since Paul was the instigator of the fight

A is correct. B is wrong since Paul consented to possibly being injured by a fistfight, but not being stabbed by a knife. C is wrong since Dan is only entitled to defend himself with a level of force equivalent to the force used by Abe. D is wrong since it is irrelevant.

Question 2 15 / 15 pts Dave is mentally ill and has a pathological hatred of women with curly hair. Pamela has curly hair. One day, Dave ambushed Pamela while she was walking across the university's campus. Dave held a knife against Pamela's neck to instill fear and proceeded to cut Pamela's hair with a pair of scissors. Pamela was so traumatized by the incident that she had to quit her job at the university and has been diagnosed with post traumatic stress disorder. Pamela now has recurring nightmares about scissors. If Pamela asserts a claim against Dave for intentional infliction of emotional distress, Pamela should: prevail, because Dave's actions were extreme and outrageous prevail, because Dave caused reasonable apprehension to Pamela not prevail, because his actions did not cause severe emotional distress not prevail, because Pamela did not suffer a physical injury

A is correct—this conduct would be extreme and outrageous; B is incorrect because reasonable apprehension is the standard for assault, not intentional infliction of emotional distress; C is incorrect because Pamela did suffer severe emotional distress as a result of Dave's conduct. D is incorrect because physical injury is not required for IIED.

Question 7 2 / 2 pts Roger, a college senior, was moving out of his second floor apartment. He wanted to throw away some of the junk he had collected rather than moving it. There was a trash dumpster on the floor below, so he tossed an old stereo speaker that no longer worked over the balcony toward the dumpster, shouting "bombs away." The speaker hit Bryan, a 5 year old boy riding by on is bicycle, and seriously injured him. Did Roger meet the standard of care? No, because the risk and magnitude of harm was very great compared with the utility of Roger's conduct. No, because Roger failed to act like a reasonable person of like age and experience. Yes, if Roger was drunk at the time of his actions. Yes, if Roger's conduct was customary in his neighborhood.

A is the best answer based on the application of the risk utility analysis, as Roger could easily have acted much more safely without great expense. B is incorrect because the child standard of care (a reasonable person of like age, intelligence and experience) is not applicable to Roger, who is an adult. C is incorrect because even if Roger was drunk, the reasonable person standard applies and his intoxication would not excuse his conduct. D is incorrect because custom is a factor in the analysis but is not dispositive, and a jury may still conclude that Roger failed to meet the standard of care.

Question 4 3 / 3 pts Brenda's lawnmower broke and she borrowed her neighbor Ned's lawnmower. Ned wasn't home and she intended to bring it right back. When she finished, she was in a hurry, and put it into her garage, took a shower and went to a movie. Ned came home later that night to find his mower missing. He went next door and looked in the window to Brenda's garage and saw the mower sitting there. Brenda was not home. Ned called her cell phone but she did not answer. Ned opened the door, and took the mower home. Which of the following statements are accurate? Ned had a right to retake his property. Ned had no right to trespass on Brenda's property. Brenda had lawful possession of the mower because she intended to return it. Brenda is liable for conversion of the mower.

A is the best answer because Ned can assert the defense of recapture of his chattel, as long as he does so in a reasonable manner and at a reasonable time and place, which appears to be the case here. B is incorrect because Ned can trespass in order to reclaim his chattel. C is incorrect because she committed trespass to chattel when she took the mower, a volitional act, without Ned's permission. D is incorrect because the deprivation was brief and she did not cause any damage to the mower, so it is not likely that this would constitute conversion.

Score for this quiz: 27 out of 30 Submitted Sep 13 at 11:32pm This attempt took 21 minutes. Question 1 3 / 3 pts Al and Bob were patients in Hospital. Dr. Arthur was scheduled to operate on Al and Dr. Barton was scheduled to operate on Bob. Both Bob and Al consented to the surgery for which each was scheduled. By mistake Al was taken to Dr. Barton's operating room and Dr. Barton commenced operating on Al. If Al asserts a claim against Dr. Barton, will Al prevail? Yes, for battery. Yes, because Al was confined to the wrong room. No, because Al consented to the surgery. No, because Dr. Barton's conduct was privileged.

A is the best answer because battery is an intentional harmful or offensive touching of another. B is incorrect because confinement is not established on the facts, which do not indicate Al was precluded from leaving the room. C is incorrect because consent to the surgery by Dr. Arthur would not be valid consent to surgery by another doctor. D is incorrect because there is no basis for a privilege by Dr. Barton under the facts given.

Score for this quiz: 225 out of 600 * Submitted Oct 4 at 9:17pm This attempt took 118 minutes. Question 1 15 / 15 pts Phoebe was employed by Dexter as an auto mechanic. When she received her pay check, Phoebe noticed that she had not been paid for overtime that she had worked the previous month. When she complained to Dexter about it, Dexter said that all company employees were expected to work over-time when necessary, and that he had no intention of compensating Phoebe for the excess hours. Phoebe resigned immediately and advised Dexter that she would hold the tools that Dexter issued to her until she received payment. After Phoebe's resignation, Dexter was angry and wanted to get back at her, so Dexter called Manny, who managed the only other garage in the small town where Phoebe had any hope of getting a job and said, "Just a warning in case Phoebe contacts you. She was a terrible mechanic. Her knowledge of cars is badly outdated and she made so many mistakes I finally had to let her go." As a result, Manny said, "Thanks, I will not be hiring her." All the statements were false, and Dexter knew it. In an action by Phoebe against Dexter for defamation, who will likely prevail? Phoebe, because Dexter's statements were slanderous per se. Phoebe, but only if Dexter made the statements with malice. Dexter, because his statements were absolutely privileged. Dexter, because all of his statements to Manny were opinions.

A is the best answer because defamation is a false statement of material fact of or about the plaintiff, published to another party causing damage to plaintiff's reputation. These elements are met here, and the statement is slanderous per se because it addresses her job performance. B is incorrect because since Phoebe is not a public figure, Dexter may be liable for defamation even if she cannot show malice. C is incorrect because his statements do not qualify for the absolute privilege, since that applies only in legislative proceedings, juridical proceedings, government proceedings, or between husband and wife, none of which are applicable here. D is incorrect because the statements concerning her knowledge and her mistakes are factual, and would not qualify as opinions.

Question 2 0 / 3 pts Pam and Diane graduated from the same university and both interviewed for the same job. Pam had more extra-curricular activities and authored a widely published article. Diane, however, had slightly higher grades although she did no such extra-curricular work. Diane is interviewed by Michael. During the interview, Michael casually asks Diane what she thinks about Pam's article. Diane states, "Oh, the article is brilliant. But I heard some grumblings from professors that Pam actually got help from Tayler Bayler for that article. Bayler practically wrote the whole thing." Based in part on this information, Diane gets the job. One year later, Pam learns that Diane had made this comment during their interviews and sues Diane for slander. Assume that there never were such rumors from professors, and that Pam did not receive help from Bayler. Is it likely that Pam will win the slander suit? Yes, because Diane's statements were false. Yes, because Pam is a public figure. No, because Diane's statements are privileged. No, because Diane's statements can be retracted.

A is the best answer because defamation requires: 1) defamatory language on the part of the defendant; 2) the defamatory language must be "of or concerning" the plaintiff (i.e., it must identify the plaintiff to a reasonable reader, listener, or viewer); 3) publication of the defamatory language by the defendant to a third person; and 4) damage to the reputation of the plaintiff. Diane made false statements harmful to Pam's reputation to the employer, causing harm to Pam's reputation. B is incorrect because there is no indication in the facts that Pam is a public figure. C is incorrect because false statements made with knowledge of falsity or reckless disregard are not privileged. D is incorrect because retraction is a remedy by a news organization barring recovery if the organization publishes a retraction.

Question 5 2 / 2 pts Todd often practiced tennis on Saturdays by hitting the tennis ball in a parking lot against a commercial office building that was closed for business. This did not cause any harm to the building, and no one had ever asked Todd not to practice there. On weekends, no one parked in the parking lot because the building was closed, so it was the perfect location to practice. There was no other place in Todd's neighborhood to practice. Which of the following statements are correct? Todd is liable for trespass to the property if he does not have permission. Todd liable not for trespass based on the defense of necessity. Todd is not liable for trespass, because he caused no damage. Todd is not liable if he had no intent to trespass.

A is the best answer because trespass is any physical invasion of real property with intent to bring about the physical invasion of the property. B is incorrect because there is no necessity for Todd to perform this recreational activity on private property, as it is not performed for the public good or intended to benefit anyone. C is incorrect because damage is not required for trespass and nominal damages may be awarded. D is incorrect because intent to trespass is not required; only intent to enter onto the land is necessary.

Question 10 3 / 3 pts Danico Industries operates a chemical manufacturing plant in the town of River Springs. Some of the equipment at the plant malfunctions. A Danico supervisor realizes that the malfunction will cause a substantial amount of chemical ash to be released into the air if the plant continues its operations, but decides that he does not want to shut down the plant. A substantial release of ash indeed occurs, and Fred, a neighboring farmer, finds that his fruit crops are covered with chemical ash, so that he cannot sell the fruit. Under the majority view, Fred can sue Danico Industries for: I. Trespass to land II. Nuisance All of the above. I only. II only. None of the above.

A is the best answer because where there is a knowing release of particles or gasses onto a plaintiff's property, most courts will consider this a trespass. A nuisance claim is also available because there is a substantial interference with Fred's use and enjoyment of his land. Therefore B, C and D are incorrect. Quiz Score: 24 out of 30

Question 6 3 / 3 pts Mark was jogging when a large pit bull running loose began chasing him. Frightened, Mark ran into the front yard of Pat. The dog continued to pursue Mark, as he banged on the front door but Pat was not home. Just as the dog approached him, Mark hopped the fence into Pat's back yard landing on a small newly planted avocado tree, snapping it into several pieces and essentially destroying the tree. If Pat sues Mark for trespass, what is the most likely result? Mark will have to pay for the tree. Mark will not have to pay for the tree. Pat will recover nominal damages. Pat will lose because Mark's entry onto the property was not intentional.

A is the best answer here, because Mark did commit a trespass to Pat's property, which is an intentional volitional entry onto the land without permission, and although the defense of private necessity will apply he will still be liable for any harm done to the property. B is incorrect because Mark is liable to pay for harm to the property. C is incorrect because nominal damages are recoverable in the absence of actual damages when no other damages; here there are actual damages to the tree. D is incorrect because Mark's entry into the property was volitional or voluntary, and therefore is intentional.

Score for this quiz: 14 out of 20 Submitted Oct 18 at 9:15pm This attempt took 20 minutes. Question 1 2 / 2 pts Dan kidnapped Anna, a 10 year old, when she was walking home from school. He put her in a shed behind his house with a padlock on it, opening the door only to bring in food. On one occasion Anna would have been able to escape, but Dan showed her his gun, and told her if she tried to get away, he would shoot her, and find her parents and kill them too. Finally, Anna was rescued by police. Her parents sued Dan on her behalf, for false imprisonment, seeking damages. While Anna was not physically harmed, the experienced scarred her emotionally. Her parents sought psychological treatment for her. Dan is Liable because Anna was confined to a bounded area. Liable because Dan's conduct was extreme and outrageous. Not liable because Anna could have escaped. Not liable because Anna was not physically harmed.

A is the best answer, and Anna was confined directly by the padlock and indirectly by the threats, in a bounded area. B is incorrect because extreme and outrageous conduct is not a required element for false imprisonment, which is the subject of the question. C is incorrect because Dan's present threat of physical harm kept her confined. D is incorrect because harm is not required if she is conscious of confinement, which was the case here, and at any rate she suffered mental harm, which would suffice.

Question 7 3 / 3 pts Pam was participating in a game of touch football with a group of her friends, male and female. Throughout the game Pam noticed that the intensity of the game was increasing. Toward the end of the game, Deb tackled Pam as Pam was running toward the goal, throwing Pam to the ground and causing Pam to break her leg. If Pam sues Deb for battery, and Deb asserts the defense of consent, Pam will: Prevail if the scope of Deb's actions exceeded Pam's implied consent. Prevail only if Deb misled Pam about the level of contact in order to get Pam to play. Lose, because Pam consented to the contact by agreeing to play. Lose, because the contact was within the expectations of a reasonable person.

A is the best answer. Consent is a defense to an intentional tort, and a participant in a contact sport is deemed to impliedly consent to the level of contact normally associated with the sport. Consent may be express or implied. By playing in the game, Pam impliedly consented to contact normally associated with touch football, but Deb's tackling would most likely exceed the scope of Pam's implied consent. B is incorrect because even if Deb did not mislead Pam about the level of contact, Pam will prevail if Deb exceeded the scope of Pam's consent. C is incorrect because Pam did not consent to the contact by continuing to play, although the game became more intense, because there is no indication of physical contact prior to Deb tackling Pam. D is incorrect because we look at the actual or implied consent of the person in question, not what a reasonable person would have done, in order to evaluate consent.

Question 3 0 / 3 pts Porter was very upset with his final grade, and he decided to get even with his teacher, Finkel, who was his history professor in the course. Finkel holds a Ph.D. from Harvard University and is considered an eminent authority. Porter went online to his Facebook page and posted a message describing Finkel as a "big bonehead," and pointed out that he felt that Finkel was "very strange looking" too. Finkel was tired of students posting negative comments about him online and sued for defamation. Porter can argue: I. The statements are opinions. II. The statements are not libelous per se. All of the above. I only. II only. None of the above.

A is the best answer. I is an argument that Porter could make, and there is a good argument that the statements are statements of opinion rather than fact, and are likely to be understood as hyperbole. In addition, Porter could argue that the statements are not libelous per se, because they do not attribute professional misconduct, crime, sexual misconduct or a loathsome disease to Finkel. Accordingly, B, C and D are incorrect.

Question 5 3 / 3 pts Hal wanted to have sex with Jan, a woman he worked with at an accounting firm. At the firm's holiday party, Hal, told Jan, "Come here, I've got to show you something," and then pulled Jan, into a small closet where the company kept office supplies. Hal pushed up against Jan and tried to kiss her but she resisted him as much as possible. Jan tried to escape but Hal blocked the door. "You are going to stay here until I get what I want," Hal said. He blocked the door. Jan started screaming and fainted. Another employee came to the rescue and let Jan out of the storage room. Hal is liable for: I. Assault II. Battery III. Intentional infliction of emotional distress IV. False imprisonment All of the above. I, II and IV only. II and IV only. II, III and IV only.

A is the best answer; assault is the intentional causing of apprehension of harmful or offensive contact, which occurred when Hal told Jan he would hold her until he got what he wanted, after she had resisted him. Battery is the intentional infliction of harmful or offensive contact, which occurred when Hal grabbed Jan and when he pushed himself against her. Intentional infliction of emotional distress is the intentional or reckless infliction of severe emotional distress through extreme and outrageous conduct, which occurred here when Hal tricked Jan into going into the room and held her there, threatening her, causing her severe distress so the she screamed and fainted. Hal committed false imprisonment when he intentionally confined Jan to the closet by means of threat. B, C, and D are correct but incomplete.

Question 8 3 / 3 pts Debbie owns a tract of undeveloped land. Every spring, she has had a problem with trespassers who enter her land without permission to use her lake. In an effort to discourage these trespassers, she posted large "no trespassing" signs in conspicuous places along the property's boundary. She also set several traps with explosives covered with dirt to catch and trap any intruders. Pat, a gardener, accidentally entered Debbie's land as he went to clear weeds in the lot adjacent to Debbie's property. He stepped on one of the explosives and fell into one of the traps on Debbie's property and was very seriously injured. If Pat sues to recover for his injuries and Debbie claims defense of property, who will prevail? Pat will prevail because Debbie used excessive force in defense of her property. Pat will prevail because his trespass on Debbie's property was not intentional. Debbie will prevail because her use of force was reasonable. Debbie will prevail because the signs alone were not effective.

A is the correct answer because Debbie's use of the explosives and trap to protect her property goes beyond what is reasonable force. B is incorrect because Pat's intent is irrelevant to whether Debbie's use of force was reasonable. C is incorrect because use of traps and spring guns is not reasonable to prevent trespass. C is incorrect because Debbie can use reasonable force to protect her property beyond signage, but cannot use traps and explosives.

Question 8 3 / 3 pts Debbie owns a tract of undeveloped land. Every spring, she has had a problem with trespassers who enter her land without permission to use her lake. In an effort to discourage these trespassers, she posted large "no trespassing" signs in conspicuous places along the property's boundary. She also set several traps with explosives covered with dirt to catch and trap any intruders. Pat, a gardener, accidentally entered Debbie's land as he went to clear weeds in the lot adjacent to Debbie's property. He stepped on one of the explosives and fell into one of the traps on Debbie's property and was very seriously injured. If Pat sues to recover for his injuries and Debbie claims defense of property, who will prevail? Pat will prevail because Debbie used excessive force in defense of her property. Pat will prevail because his trespass on Debbie's property was not intentional. Debbie will prevail because her use of force was reasonable. Debbie will prevail because the signs alone were not effective.

A is the correct answer because Debbie's use of the explosives and trap to protect her property goes beyond what is reasonable force. B is incorrect because Pat's intent is irrelevant to whether Debbie's use of force was reasonable. C is incorrect because use of traps and spring guns is not reasonable to prevent trespass. C is incorrect because Debbie can use reasonable forcew to protect her property beyond signage, but cannot use traps and explosives.

Question 7 0 / 3 pts Maureen works in a book store. She believes that Molly is shoplifting a book from the self-help section and informs her supervisor, Ned, via walkie-talkie. She describes Molly as a brunette woman in a light blue overcoat heading for the exit. Ned believes that Maureen is referring to Pam, another brunette woman in a light blue overcoat, who happens to reach the exit before Molly. Ned follows Pam out of the store and across the parking garage, where he pulls a gun and says, "Ma'am, you'll have to come with me." Pam says "Whoa, take it easy there with that gun, I've done nothing wrong." Ned keeps Pam in the store office waiting for 2 hours until Maureen comes in and informs Ned that he is holding the wrong person. On their way to the security office, Ned had paraded Pam past several of her co-workers who were shopping on their lunch hour and she is mortified by the situation. If Pam sues Ned for assault, Pam is likely to... Prevail, if Pam saw the gun pointed at her. Prevail, because Maureen's description caused Ned's conduct. Lose, if Pam suffered no physical injury. Lose, if there was no intent to injure.

A is the correct answer. An assault is an apprehension of an immediate battery. This is exactly the case when Ned spoke in a threatening manner and pulled out a gun. Therefore, Pam will prevail. B is incorrect because it does not go to an element of assault. C is wrong since a recovery for assault does not require an actual injury. Even without actual injuries, Pam is free to recover nominal damages and, if the defendant's conduct was extreme or outrageous, possibly punitive damages. D is wrong because intent to injure is not an element of the tort of assault.

Question 5 0 / 3 pts Pam is an eighty-two-year-old woman who is a diabetic. As part of her daily medical routine, she drinks a liter of Dr. Pippen. Late one night, Pam realized she had not had her soda. She went to a convenience store to buy some Dr. Pippen. While waiting, she began to feel faint. The line was very long, and fearing that she would faint, Pam put down $5 and left the store. The list price for the soda was $3. The store's security guard, saw Pam walk out the store with the soda. Believing that she had stolen the soda, he followed her out of the store. When he saw her get into her car, the guard immediately jumped in front of Pam's car, preventing her from leaving. Unable to exit the store parking lot, Pam became frightened and suffered a diabetic seizure. If Pam asserts a false imprisonment action against the store, she should: Not prevail, if the guard acted reasonably in believing a theft had occurred Not prevail, because it was unforeseeable that a temporary detention would result in the detainee suffering a diabetic seizure Prevail, because she didn't steal anything Prevail, because the guard blocked her exit

According to the "shopkeeper's privilege", a businessman who reasonably suspects a customer of theft, or of failure to pay, may detain the suspected individual for a short time in order to investigate. The privilege is, however, a very restricted one, confined to what is reasonably necessary for its limited purpose, of enabling the defendant to do what is possible on the spot to discover the facts. There will be liability if the detention is for a length of time beyond that which is reasonably necessary for such a short investigation. In the present case, the guard was privileged to make a temporary detention since he reasonably believed that a theft had occurred. As a result, choice (A) is correct. Note that choice (B) is less preferred because Pam is asserting an action for false imprisonment. Thus, it is irrelevant whether her seizure was foreseeable. The "key" issue here is whether the storekeeper acted reasonably under the circumstances.

Question 2 0 / 2 pts Patricia and Scarlet were roommates who both attended a prestigious fashion design school. When Patricia had received her acceptance letter from the school, she splurged and purchased a new sewing machine for $1,000 so that she would be more than adequately equipped for her design assignments. One day when Patricia was at class, Scarlet loaned Patricia's sewing machine to Violet, whose own machine lacked the fancy options of Patricia's machine. Scarlet had loaned Patricia's sewing machine to other design students on several prior occasions. Unfortunately, Violet, who was not familiar with Patricia's delicate machine, caused extensive damage to the machine. It would cost $400 to repair the sewing machine. If Patricia sues Scarlet for the damage Violet caused to the sewing machine, what will be the result? Patricia will recover $1,000. Patricia will recover the fair market value of the sewing machine. Patricia will recover $400. Patricia will recover nothing, because Scarlet did not damage the machine and Violet's conduct was not intentional.

Answer: B is the correct answer. Olivia will recover the fair market value of the machine because Scarlet is liable for conversion. Conversion is the intentional interference with the plaintiff's right of possession in the chattel that is serious enough to warrant that the defendant pay the full value of the chattel. Conversion will be found if the defendant was using the chattel without permission and it was accidentally damaged, as in this case. The remedy for conversion is the fair market value of the chattel at the time and place of conversion. Thus, (B) is correct and (A) is incorrect. (C) is incorrect because that would be the remedy for trespass to chattels, which is a less serious interference than conversion. Here, an unauthorized use that resulted in damages equaling almost half the chattel's original cost is too serious an interference in nature and consequences to be only trespass to chattel. (D) is incorrect because Scarlet's lending the sewing machine without permission satisfies the intent requirement. Even though the damage was accidental, Scarlet is liable for conversion.

Question 15 0 pts Dave, who was sent to prison, vowed to get even with Paul, the prosecutor at his trial. While in prison, Dave was told by another prisoner, Dirk, that when Paul was in private practice as a criminal defense attorney, he had represented Dirk in a drug charge. Dirk claimed that because he did not have the cash to pay Paul his fees, he offered to pay his fees with five ounces of cocaine, and Paul accepted. Although Dave had no reason to believe that what Dirk said was true, when he got out of prison he learned that Paul was running for District Attorney. Dave went to one of the local papers and sold them the story for $1,000. In the article that resulted, Dave was quoted as saying "I only hope that Paul suffers like I had to suffer in prison." Although the allegation was false, Paul withdrew from the race as a result of the article. In a suit by Paul against Dave for defamation, the probable result would be: Group of answer choices Paul will prevail because Dave acted with actual malice. Paul will prevail if Dave should have known that the story was false. Dave will prevail because the story was a matter of public concern. Dave will prevail if he honestly believed the truth of the assertion made by Dirk.

Answer: D is the correct answer. Dave will prevail if he believed the truth of the assertion. Paul is a candidate for public office and, as such, to recover he must establish 1) falsity, and 2) "actual malice"-i.e., knowledge of falsity or reckless disregard of truth or falsity. Thus, if Dave honestly believed the truth of his statement, Paul could not show actual malice, and Dave would prevail. (A) is wrong because the facts do not indicate that Dave acted with knowledge of falsity or reckless disregard of truth or falsity. (B) is wrong because a showing of negligence is not sufficient to recover for defamation of a public official or figure; knowledge of falsity or reckless disregard of truth or falsity is required. (C) is wrong because even if the story is a matter of public concern, Paul might still prevail if he can show fault on the part of Dave-such as Dave's reckless disregard for whether the statement was true.

Question 9 0 / 3 pts Stella works for Bullseye, a large department store. One day, she saw Dianne, a customer, and believed her to be stealing school supplies. She called up the store's security guard on duty, and informed him that Dianne was a slender, blonde female, wearing a purple coat and brown boots. The guard saw a woman that matched the description and grabbed the woman saying "Come with me or I'll shoot you in the face!" It turns out that the guard was mistaken, and accidentally detained Sally, who matched the description of Dianne. The guard forcefully moved Sally to the cleaning closet in front of numerous shoppers where he held her for two hours until he realized she had done nothing wrong. Assume that Sally sues Bullseye for damages relating to the humiliation she suffered when paraded past the shoppers as a shoplifter. Bullseye will likely: Prevail, unless the conduct of Bullseye's security guard was extremely outrageous. Prevail, because humiliation is not an actionable offense. Lose, if Bullseye's security guard acted negligently in identifying Sally as the shoplifter. Lose, if Sally was falsely imprisoned.

Answer: D is the correct answer. In the present fact pattern, Sally clearly has a cause of action for false imprisonment against Bullseye. False imprisonment requires that the defendant intentionally causes, by acts or threats, the plaintiff to be totally confined for an unreasonable amount of time within some boundary imposed by the defendant, with no reasonable means of escape. Additionally, the plaintiff must be aware of the confinement and must not consent to it. In light of this, recovery by Sally is clearly available, making Bullseye liable. A is a statement that is factually wrong. Bullseye's outrageous conduct may play a part in an alleged intentional infliction of emotional distress case, but this plaintiff is suing under the theory of false imprisonment. The elements of a false imprisonment case do not require a showing of outrageous actions by Bullseye in order for Sally to be allowed to recover. B is a wrong answer because humiliation, as related to Sally's false imprisonment, is clearly an actionable offense. C is wrong. While Bullseye may have acted negligently in detaining Sally, such that Sally was falsely imprisoned, the case that Sally is actually advancing simply arises from the fact that she was falsely imprisoned. How exactly that occurred, (i.e., by the store's negligence) is really meaningless to the analysis from Sally's perspective.

Question 12 0 pts Debbie is jogging, when a chartered bus swerves onto the sidewalk where she is jogging and heads straight for her. The bus driver is so preoccupied with the passengers in the bus that he did not realize that the bus has gone off the road. Debbie tried her best to get out of the way in time and at the last minute she jumped into her neighbor Pam's yard. Once she landed on the ground, Debbie realized that she was lying in a patch of Pam's prize-winning pumpkins, and had essentially killed an entire section of the pumpkin patch. Pam asserts an action against Debbie for damages to her pumpkins. How will the court likely rule? Group of answer choices Debbie is liable as she had no privilege to enter Pam's yard. Debbie is liable because she failed to act with due care. Debbie was privileged to enter Pam's land, therefore she is not liable. Debbie is liable for the damaged pumpkins.

Answer: D is the correct answer. The privilege of necessity states that a person can interfere with the property of another to reasonably avoid threatened serious injury from a natural act or the act of another if the threatened act is substantially more serious than the invasion of the party. When dealing with private necessity (i.e., an act that only benefits one person) the actor remains liable for any damage they cause, even though they may be privileged to enter. A is partly right, as it does correctly state that Debbie is liable for the damage. It incorrectly states, however, that she was not privileged to enter Pam's yard. B is incorrect. Exercise of due care is meaningless since the claim against Debbie will be one of intentional tort, such as a trespass or conversion. C is wrong. The fact that Debbie has privilege does not mean that she is not liable, as set forth in the correct answer.

Question 3 0 pts Ron played softball with a team from work. Bart was a player on one of the opposing teams in the same league, and somewhat of a poor sport. One evening after Ron's team had won the game in the final inning, Bart said to Ron, "If you weren't so much bigger than me, I'd throw this baseball glove right at your head." Ron was much bigger than Bart, so he laughed it off and called Bart a "shrimp," and punched Bart in the arm. Bart yelled, "Ouch! You big bully!" But Ron just laughed at Bart again, saying "You can't even take a joke." The most likely result is that: Group of answer choices Bart is liable for assault and Ron is liable for battery. Bart is not liable for assault, but Ron is liable for battery. Bart is liable for assault, but Ron is not liable for battery. Bart is not liable for assault, and Ron is not liable for battery.

B is the best answer because Bart's threat was conditional, and was not imminent, and Ron was not afraid of Bart, so he did not have the required apprehension for assault; Ron's punch to Bart was a harmful contact that was intentional, and therefore a battery. For these reasons, A, C and D are incorrect.

Question 16 15 / 15 pts Don, expecting to meet his wife, Mary, on a particular street corner, arrives and sees Patty, with her back turned toward him. Patty bears a close resemblance to Mary, even wearing a black dress similar to a dress that Mary often wears. Don walks behind Patty and grabs her by the neck, intending to scare her, believing Patty to be Mary. Patty screams and jerks herself away from Don, and falls off the curb, suffering an injury. If Patty brings an action for battery against Don, will she likely prevail? Yes, if Patty had a reasonable apprehension of Don's contact. Yes, because Don's contact was offensive and harmful. No, because Don intended to make contact with his wife, not Patty. No, because Don did not intend to cause the actual type of harm that occurred to Patty.

B is correct because battery is an intentional infliction of harmful or offensive bodily contact, and Don's conduct was intentional and the contact was intended to scare his wife, therefore, to be offensive; and Don's intent will transfer from his wife to Patty. A is wrong because reasonable apprehension of contact is not an element of battery; there must be an intent to bring about harmful or offensive contact; also, because Don approached Patty from behind, there would be no opportunity for her to apprehend harmful contact prior to the contact itself. C is wrong because this involves transferred intent, and Don's intent to make contact with his wife transferred to Patty. D is incorrect because an intent to cause harmful or offensive contact is enough for liability, even if a different type of harm results, under the eggshell skull plaintiff rule.

Question 20 0 / 15 pts The Devils are a professional football team with enthusiastic fans. During contentious games, rowdy fans of the team have thrown objects, including cans and bottles, from the stands onto the field, aimed at opposing team players. On one occasion, Doug, a rabid Devils fan, was able to smuggle a can of beer into the stadium. Doug threw the beer can at the opposing team from the stands. The can missed the field, sailed out of the stadium and hit a policeman who was patrolling the parking area. If the policeman brings a claim against Doug for battery, is the policeman likely to prevail? Yes, because the policeman was hit by the can Yes, because the doctrine of transferred intent would apply No, because the policeman was not under any apprehension or fear No, because Doug lacked the requisite intent

B is correct. A is wrong since although there was physical contact with the policeman, here the intent to commit a battery on one person transferred to another person. C is irrelevant since this is an element of assault, not battery. D is wrong because Doug had the intent to hit someone; the intent can transfer from Doug's chosen victim to whoever was actually harmed

Question 8 3 / 3 pts Bryce was driving his speedboat on the lake when a massive storm arose. Afraid that the boat might sink, Bryce drove to a boat dock on shore and tied the boat to the dock. The shore property and dock were the private property of Dane. While the boat was tied at the dock, Dane ordered Bryce to remove the boat because the waves were so large that he feared that the boat would slam into the dock and damage it. Bryce refused, and Dane untied the boat causing the boat to sink. Bryce lived a mile down the lake from Dane. However, the only land routes back were a steep, muddy trail (dangerous during the storm), and a 45-mile road around the lake. The storm continued, and Bryce asked Dane to take him home. Dane said, "Yeah right, you almost damaged my dock." After three hours, the storm stopped, and Bryce walked home over the trail. A necessary element in determining if Bryce is liable for a trespass is whether: Dane had clearly posted his property with a sign indicating that it was private property. Bryce had reasonable grounds to believe his boat might be swamped and might sink. Bryce had reasonable grounds to believe the property belonged to a private person. Bryce knew that the property belonged to a private person.

B is correct. Although Bryce intended to go onto Dane's property, his entry was privileged if Bryce believed it was necessary to protect himself and/or his boat. Note that this privilege is qualified in that Bryce must pay for any damage he caused to Dane's property. (A), (C), and (D) are incorrect because knowledge that the land is privately owned is irrelevant to the privilege of necessity.

Question 3 3 / 3 pts Ann and her brother Ben were college roommates, but they did not always get along very well. Ann was angry with her brother Ben after he left the place in a mess after a party with his friends. Ann walked into the living room and saw the mess and became irate. She found Ben sleeping in his room and started yelling at him, but he remained asleep because he was somewhat drunk. Ann threw an empty beer can at Ben, but missed him. Ann walked into Ben's room, picked up a sweatshirt and began hitting Ben with it while he slept. Which torts, if any, has Ann committed? Both assault and battery. Only battery. Only assault. Neither assault nor battery.

B is the best answer because Ann has committed an offensive touching of Ben with intent. C and A are incorrect because Ann did not create apprehension of contact when she threw the beer can, since Ben was asleep, and her words alone are not enough. D is incorrect because Ann committed battery.

Question 4 3 / 3 pts Patty and her boyfriend Dale were double-dating with Betty and her boyfriend Carl. Patty was arguing with her boyfriend Dale about where they should go out to dinner. As the four walked to the car together, Dale said, "No, not hamburgers again! I can't take another hamburger!" He turned and dramatically threw the jacket he was carrying at Patty. Patty ducked, and the jacket missed her. Betty was hit by the jacket and the zipper on the jacket hit Betty in the eye, causing a serious injury. Is Dale liable to Betty? Yes, if Dale intended to harm Betty. Yes, because Dale's conduct harmed Betty. No, because Dale did not intend to harm Betty. No, because Dale did not intend to harm anyone.

B is the best answer because Dale acted volitionally and desired to cause contact with Pam, and that intent is transferred to Betty. A is incorrect because intent to harm Betty is not required, and intent to cause contact is sufficient, even if Dale intended to cause contact with Pam. C is incorrect because intent to cause contact is sufficient for battery. D is incorrect because Dale can be liable for the harm caused if he was substantially certain that by throwing the jacket it would touch another..

Question 2 2 / 2 pts One hot summer day, Simon, a teenager, was hiding behind a bush, throwing rocks at cars stopping at a nearby stop sign launching the rocks high into the sky, hoping drivers would think the rocks just fell from the sky. Simon watched with amusement the reactions of the drivers as they saw the rocks coming at them. Simon launched a rock intended for Al's convertible. Al saw the rock coming at him and yelled at Simon, "That rock better not hit my car, buddy!" The rock missed Al's car. Simon launched a rock as Bob's jeep stopped at the stop sign, intending to hit Bob. Bob did not see the rock coming, and took off. The rock hit Carl, who was driving behind Bob on a motorcycle and did not see it coming. The rock shattered the shield of Carl's helmet. If Al, Bob and Carl sue Simon, which of the following is correct? Simon is liable to Al and Bob for assault, and to Carl for battery. Simon is liable to Al for assault, not liable to Bob and liable to Carl for battery. Simon is liable to Al for assault and battery, liable to Bob for assault, and to Carl for battery. Simon is liable to Al for assault, and liable to Carl for assault and battery.

B is the best answer because Simon is liable to Al for assault, since Al saw the rock coming at him in his convertible and felt apprehension as reflected in his statement to Simon. Simon is not liable to Bob because Bob was not touched so there is no battery, and he did not see the rock, and so there was no apprehension for assault. Simon is liable to Carl for battery given the intentional harmful contact with Carl's helmet which is associated with his person. For these reasons, A, C and D are incorrect.

Question 10 3 / 3 pts Ann, Betty and Cara were waiting for the bus. When the bus approached, it was very crowded. In order to be sure she would get on the bus, Ann pushed Betty against Cara, knocking Cara down to the ground, causing Cara to break her arm. Cara sued Ann and Betty for battery. How should the court rule? Both are liable for battery. Ann is liable but Betty is not liable. Betty is liable but Ann is not. Neither Ann nor Betty will have liability for battery.

B is the best answer because battery is an intentional harmful or offensive touching of another; here Ann used Betty to make contact with Cara. Even though Betty made contact, she is not liable for battery, as there is no indication she had intent to take any action causing the harmful of offensive contact with Cara. As a result, Ann C is incorrect and D is incorrect. Quiz Score: 27 out of 30

Question 4 2 / 2 pts Paul submitted a job application to WalCo, a large department store, indicating that Paul had previously worked for TradeSmart, another retail operation. Walt, the manager of WalCo's human resource department contacted TradeSmart for a reference, asking about Paul. Ted, who worked for TradeSmart, but had never worked with Paul, got mixed up and pulled up the wrong file on his computer during the phone call with Walt. Ted thought that Walt was referring to another person named Paul who had also worked at TradeSmart. Ted stated, "I am not sure I would trust Paul with cash." After he hung up the phone, Ted realized his mistake, but made no effort to correct the impression he had left with Walt. As a result, WalCo decided not to hire Paul. If Paul sues TradeSmart for defamation, which of the following is true? The statement is not actionable because it is an opinion. The statement is slanderous per se. The statement is not actionable because it is within the qualified privilege. The statement is not actionable because it was only published to one person.

B is the best answer because slander per se is a false statement regarding Paul's honesty and his business ethics, implying he is responsible for theft, which is criminal conduct. A is incorrect because the statement implies facts as previously indicated. C is incorrect because the qualified privilege is available only if the privilege is not abused, and here Ted acted with malice (reckless disregard) when he did not correct the statement he realized was false. D is incorrect because publication to just one person is sufficient for defamation.

Question 6 2 / 2 pts Charles, a 10 year old boy, love to go boating with his family in their 20 foot speed boat. From time to time, Charles' father Frank would let Charles drive the boat. Charles was involved in an accident on the lake when he hit a person who cut in front of him riding a jet ski. In determining whether or not Charles was negligent, what is the standard of care? Charles will be held to the standard of care of a reasonable child of like age and experience. Charles will be held to the standard of care of a reasonable adult. Charles will be held to the standard of care of his father Frank. Charles will be held to the standard of care of a person with similar physical characteristics.

B is the best answer because this is an adult activity, so the standard of care of a reasonable adult will apply. A is incorrect because the child standard of care does not apply to an adult activity. C is incorrect because the reasonable person is not measured in reference to a particular adult. D is incorrect because the standard of care of a person with similar characteristics is applied where a person has a physical disability, which is not the case here.

Question 5 3 / 3 pts Arthur is a news reporter and presented a story on the evening news for KXTV, indicating that Professor, an instructor at one of the local community colleges, had been fired for having an impermissible relationship with a student. The truth was that this occurrence involved Coach, not Professor. If Professor sues KXTV for defamation, he will prevail: If KXTV had no ill will toward Professor. If KXTV failed to investigate the story with reasonable care. Only if KXTV had knowledge of the falsity of the story. If KXTV honestly believed the story when it aired.

B is the best answer here because Professor would not be a public figure, so KXTV would be required to use reasonable care, and would be liable if it failed to do so. A is incorrect, because actual ill will toward Professor is not relevant. C is incorrect because actual knowledge that the story was false would not be required here, since Professor is not a public figure. D is incorrect because an honest belief in the accuracy of the story will not be enough to defeat a defamation claim.

Question 4 3 / 3 pts Clint's team lost a hard fought baseball game for the regional championships against Alvin and Bart's team. Angry about a bad call, after the game Clint sees Alvin walking out of the gym and yells "Loser"!" and throws a rock at Alvin, intending to scare him. Hearing Clint, Alvin ducks. The rock hits Bart, who was walking out of the gym right behind Alvin. Bart's eye is seriously injured. If Bart sues Clint, the most likely result is: Clint is only liable for assault because he intended to scare Alvin. Clint is liable for battery based on transferred intent. Clint is not liable because he intended only to scare Alvin. Clint is not liable because Alvin's conduct caused the injury to Bart.

B is the best answer. Battery is an intentional harmful or offensive touching of another. The doctrine of transferred intent applies so that Clint's intent to scare Alvin transfers from the tort of assault to the tort of battery, and also to transfer between victims, from Alvin to Bart. A is incorrect because Clint will also be liable for battery, based on the doctrine of transferred intent. C is incorrect because even though Clint only intended to scare Alvin, he will be liable here for the battery on Bart. D is incorrect, because Clint's conduct was a direct cause of the injury to Bart.

Question 12 15 / 15 pts Two teams were fierce competitors in a National Soccer Tournament Championship held in Las Vegas. Both teams were staying at the Sports Hotel in anticipation of the tournament. The Team A coach learned that the Team B coach was staying in Suite #102 and when the Team A coach checked into the hotel, he requested Suite #104, which he knew adjoined the Team B coach's suite. To obtain valuable information about Team B's strategy for the tournament, the Team A coach used an electronic listening device to listen to conversations between the Team B coach and his players. The Team A coach used this information and Team A won the tournament. If the Team B coach brings a claim against the Team A coach based on invasion of privacy, the Team B coach likely will: prevail, because the Team A coach was recording his conversations to gain an advantage in the tournament prevail, because the Team B coach had a reasonable expectation of privacy in his suite not prevail, unless there was publication of the recorded conversations not prevail, because the conversation was on a private, not public issue

B is the correct answer since the coach would expect that conversations held in a hotel room would be private and invasion of that privacy would be highly offensive to a reasonable person. A is wrong since the motivation of the Team A coach in invading the privacy of the Team B coach is irrelevant. C is wrong since publication is not required for intrusion on seclusion. D is irrelevant.

Question 8 15 / 15 pts Patricia headed out of a store after an afternoon of shopping. Patricia approached the front door of the store at the same time as a shoplifter, whose stolen goods set off the store's inventory control alarm. The security guard at the door requested that Patricia remain in the store so that her bags could be checked. When Patricia protested, the security guard told her that he would handcuff her if she would not cooperate in having her bags searched. Patricia begrudgingly agreed to the search and she was detained in the store for 15 minutes, missing her bus to go home. If Patricia is unsuccessful in a false imprisonment action against the store, it will be because: The security guard did not intend to handcuff her The security guard acted reasonably in dealing with a suspected shoplifter Patricia suffered no physical injury from being detained in the store Patricia was able to take the next bus home

B is the correct answer since the shopkeeper's privilege allows the security guard, acting on behalf of the store, to take reasonable measures to attempt to recapture stolen chattel. A is incorrect since whether or not the guard intended to handcuff Patricia, he intended to detain her in the store. C is wrong since a physical injury is not a required element of false imprisonment. D is wrong since it is irrelevant.

Question 4 0 / 3 pts Paula owns a tract of undeveloped land. Every autumn, she has had a problem with hunters who enter her land without permission to shoot geese. In an effort to discourage hunters, she posted large ""no trespassing"" signs in conspicuous places along the property's boundary. She also dug a number of deep holes, which she covered with brush to catch any intruders. Dave, a local land surveyor, was retained to survey the parcel of land adjoining Paula's property. While conducting the survey, Dave inadvertently walked onto Paula's property. He fell into one of the holes and was seriously injured. In an action by Paula against Dave for trespass to land, the likely outcome will be: Paula will recover only if her land was damaged. Paula will recover, even if the land was not damaged. Paula will not recover because Dave did not intend to trespass. Paula will not recover, unless she can prove some actual damage.

B is the correct answer, because damages are presumed in trespass cases. Even if Paula cannot show actual damage, the law will allow nominal damages. A and D are incorrect because Paula can prevail on a trespass to land claim whether or not she can show actual damages. C is incorrect because the only intent necessary for trespass is to walk on the land, an intent that Dave had.

Question 2 3 / 3 pts Alex was a sixteen-year old boy who loved camping. Alex planned to spend an entire week-end camping alone in the woods to test his survival skills. Bob, who was aware of Alex's plan, telephoned Patty, Alex's mother, the day after Alex left to go camping. Bob, disguising his voice, told Patty, "We have kidnapped your son. If you want to see him again, you must leave $10,000 in small bills in a brown paper bag at the entrance to the park at midnight." Horrified that her son might be injured or killed, Patty delivered the ransom money as instructed. The next day, she had a nervous break-down. Patty's emotional condition improved when her son Alex returned home uninjured two days later, but she continued to need anti-anxiety medication and therapy. If Patty asserts a claim against Bob for damages based on intentional infliction of emotional distress, who will likely prevail? Patty, because Bob's conduct caused reasonable apprehension in Patty. Patty, if Patty's nervous breakdown was caused by Bob's outrageous conduct. Bob, but only if he made the threat to Patty to obtain the ransom money, not to cause Patty harm. Bob, because Patty's nervous breakdown occurred after his telephone call to her.

B is the correct answer. A defendant is liable for intentional infliction of emotional distress he acted to intentionally or recklessly inflict emotional distress through extreme and outrageous conduct which causes severe mental suffering. A defendant intends to cause severe emotional distress if he desires or knows that it will result from his conduct. Because of the parent-child relationship, Bob probably knew that his threats to injure or kill Alex would cause Patty to suffer severe emotional distress. A is wrong because reasonable apprehension is the test for assault, not for emotional distress. C is wrong since Bob probably knew that his threats to injure or kill Alex would cause Patty to suffer severe emotional distress. D is wrong since the passage of time is not sufficient to prevent liability for IIED if causation is present.

Question 17 15 / 15 pts An insecticide factory was located at the edge of a City. When the wind blew, foul-smelling waste gases from the factory were blown over the City, causing most of its residents to experience burning of the eyes and throat. Patrick was a resident of the City. On several occasions, he attempted to persuade the City Attorney to seek an injunction against the factory. The City Attorney refused, however, because he was afraid that by doing so, the factory would be driven out of the area and it employed many city residents. If Patrick seeks an injunction by asserting a claim against the factory on a theory of public nuisance, which of the following would be the factory's most effective argument in defense? The City Attorney's decision is binding Patrick has not sustained harm different from that of the general public A private citizen may not seek an injunction against environmental polluters A private citizen may not sue on a theory of public nuisance

B is the correct answer. A private citizen can assert a claim for public nuisance only if the harm which the citizen sustained is different from that sustained by the general public. Since there are no facts indicating that is the case with Patrick, he may not assert the claim. A is incorrect since a private citizen in not bound by the City Attorney's decision. C and D are incorrect since private citizens can sue on a private nuisance claim.

Question 6 3 / 3 pts On a cold winter evening, Debbie and a friend went out to dinner at a fancy restaurant and they checked their coats at the entrance. The couple had a great meal and they stayed for hours, eating and drinking numerous glasses of wine. They were both very inebriated when they left the restaurant and took a cab home. Before they left, Debbie was given someone else's expensive leather coat instead of her own. Debbie did not realize the coat was not her own until six months later. After realizing the mistake, she immediately returned the coat to the restaurant. Debbie would likely be liable to the owner of the coat for which of the following: Trespass to chattels. Conversion. Neither trespass to chattels nor conversion because the coat was returned to its rightful owner. Neither trespass to chattels nor conversion because Debbie acted in good faith.

B is the correct answer. Although the coat was eventually returned to the owner, the period of time during which Debbie exercised dominion over the chattel (coat) was so long that the deprivation would be deemed to be substantial, therefore, conversion. A is incorrect since the period of time during which Debbie exercised dominion over the chattels (coat) was so long that the deprivation would be deemed to be substantial, therefore, conversion rather than trespass to chattels. C is incorrect because the deprivation was substantial. D is incorrect because the only intent required for conversion is the intent to perform the act (here, taking the coat) that brings about the interference with a plaintiff's right to possession.

Question 14 15 / 15 pts Donna went to a party at her friend's home and when she arrived, she placed her coat in the hall closet. When Donna grabbed her coat out of the closet to leave, she took her friend Paula's coat instead. When Donna got into her car, she realized her mistake and immediately went back into her friend's home and returned the coat to Paula, taking her own coat out of the hall closet. If Paula asserts an action against Donna, on which theory is she likely to succeed? Conversion Trespass to Chattels Both Conversion and Trespass to Chattels Battery

B is the correct answer. Trespass to Chattels is the appropriate claim here since Donna did exercise dominion over the personal property of another, albeit for a short period of time and without damaging the chattel. Nominal damages for loss of use would be available.

Question 4 0 pts Peter has liver disease that has been diagnosed as terminal. During his decline, Dr. Dan visits daily and snaps a photo, documenting things for a book that he is publishing. For weeks he does this without protest from Peter or his wife. One day Peter passes out and, after being revived, the nurses inform his wife that Peter only has minutes to live. As Peter's wife is stroking his hair and saying goodbye, Dr. Dan walks in ready to snap another photo. Peter's wife asks him to leave. Dr. Dan ignores the request and goes to rearrange some flowers near Peter's bedside to make the photo happier looking. After rearranging the flowers and propping up Peter's head with a pillow, the doctor snaps a picture as Peter dies. Peter's wife decides to sue and considers the following tort actions against Dr. Dan: I. Assault and Battery; II. False Light; III. Intrusion on Seclusion IV. Publication of Private Facts. Which of these tort claims will Peter's wife likely recover for: Group of answer choices I only I and III only II, III, and IV only I, II, III, and IV

B.) Peter's wife will likely recover on I. and III. Battery requires: 1) an act that brings about harmful or offensive contact; 2) the intent to bring about the harmful or offensive contact; and 3) causation. Whether or not contact is offensive, is judged by a reasonable person standard. Contact by a doctor is generally deemed NOT to be offensive by nature, but the propping up of Peter's head during his last minutes is clearly beyond what a reasonable person would expect. Intent is satisfied by the fact that Doctor Dan was aware that neither Frank, nor his wife, wanted the touching and went through with it anyway. Assault requires: 1) an act by the defendant to create a reasonable apprehension in the plaintiff of immediate, harmful or offensive contact to the plaintiff's person; 2) intent to create the apprehension or bring about the contact; and 3) causation. Dr. Dan's intent to commit this touching serves to meet the intent requirement of assault, making an assault-based recovery also possible. In both battery and assault, there is clearly causation and actual damages are not needed. Further note that a person like a wife can sue even after her husband's death occurs. Intrusion on the plaintiff's seclusion is also a viable recovery channel based on the wife's own claim, making (A) wrong. This requires a showing of some intruding on the seclusion of the plaintiff when dealing with personal private matters, and that the intrusion was objectionable to a reasonable person. Dr. Dan's actions were clearly intruding into the personal matters of a plaintiff and were also clearly offensive. Generally speaking, however, a doctor walking into the room of a dying patient would not suffice for commission of this tort. It is Dr. Dan's explicit behavior and the facts that allow this recovery. (C) and (D) are wrong because either or both of these torts require some form of publicity or publication, and the facts here only note that Doctor Dan planned to use the photos in a book. There are no facts to indicate that he has used them in any public manner.

Question 10 0 / 15 pts Dean wrote a weekly humorous column about daily life in a small town for the local newspaper. One of Dean's columns described the petty battles that neighbors in a small town engaged in at times. In that column, Dean recounted a battle that he got into with his neighbor, Peter, about the height of a hedge between their property, stating that Peter was a "stubborn mule." If Peter asserts a claim for defamation against Dean, what would be Dean's most effective argument in defense? That he did not have actual malice when he made the statement about Peter That Dean had an absolute privilege to write about Peter That reasonable people would not believe that the statement reflected Dean's opinion That the statement did not address Peter's business or professional activities

C is correct because one is liable for defamation only for statements of fact, not opinion. A is incorrect since there is no actual malice requirement for statements about private persons on private matters. B is wrong since the absolute privilege only applies to statements made in legislative, judicial or government proceedings, or between husband and wife, all of which are not applicable here. D is incorrect because Dean can be liable even if his statements do not address Peter's business or professional activities.

Question 3 15 / 15 pts Dennis was driving through a coastal community 200 miles from home when a hurricane began to hit the area. Because of the dangerous weather which was making the low-lying road impassable, Dennis decided it would be safe for him to park his car on the top of a hill and wait out the storm and then drive home later. The property on the hill was owned by Pat, who came out and demanded that Dennis remove his car. Dennis refused, explaining that the storm made it too dangerous for him to return down the hill to the road. In an action by Pat against Dennis for trespass, who is likely to prevail? Pat, because Dennis intended to enter his property Pat, because Dennis knew that someone other than himself owned the hill Dennis, if he had a reasonable fear that his life was in danger from the storm Dennis, but only if he can show consent by Pat

C is correct. A and B are wrong because Dennis can assert the privilege of private necessity to enter Pat's land to avoid serious injury. D is incorrect because Dennis can prevail based on the defense of necessity, even if he cannot show consent by Pat.

Question 6 3 / 3 pts Ben lived along the coast of Florida. There was an advisory warning that a hurricane was approaching. The warning advised residents to evacuate their homes and flee inland for their safety. Ben, who heard the announcement, ignored the warning and stayed at home. The hurricane destroyed Ben's home. To save his life, Ben took his neighbor Phillip's jet ski without permission. The jet ski was worth $15,000. While Ben was operating the jet ski, he ran over a piece of wood in the water, causing $4,000 in damages to the jet ski. Phillip has asserted a tort action against Ben. Phillip should be entitled to recover: Nothing, because Ben took the jet ski in an emergency situation Nothing, based on the defense of necessity $4,000 $15,000

C is correct. As a general rule, private necessity will excuse tortious conduct aimed at property. However, according to Restatement of Torts, 2d, Section 263, which specifically addresses the privilege of private necessity as applied to trespass to chattels or conversion, "Where the act is for the benefit of the actor or a third person, he is subject to liability for any harm caused by the exercise of the privilege." Therefore, Ben would be liable for the damage caused to Phillip's jet ski. Choice (D) is incorrect because Ben is not liable for conversion because he only caused partial damage to the jet ski. A and B are incorrect because even if he took the jet ski in an emergency situation based on necessity, he is liable for the damages caused to the chattel.

Question 2 3 / 3 pts Percy goes twice a week to Denise, a psychiatrist, for treatment of his Tourette syndrome. Unknown to Percy, Denise has a hidden video camera in her office and tapes each session as part of her thesis on the body language of Tourette sufferers. Denise does not record any audio, so no one would be able to figure out what Percy is saying from the video. Percy learns of the videotaping and fears that it may jeopardize his public image as husband to a US Senator. He sues Denise for battery. If Percy does not win, it is likely because: No injury was suffered by Percy due to Denise's actions. Denise did not act with intent. There was no harmful or offensive touching. Denise's conduct caused Percy reasonable apprehension.

C is correct. Here, there is no harmful or offensive touching, therefore, no claim for battery. A prima facie case of battery requires the showing of an intentional act by the defendant that caused harmful or offensive contact to the plaintiffs person.A is wrong. For battery, you do not have to show an injury to the person. Battery can occur even without a claimed physical injury. B is incorrect because Denise did act with intent in taping Percy. D is incorrect because a reasonable apprehension is an element of assault, not battery.

Question 6 0 / 15 pts Phillip was jogging through his neighborhood late at night. A policeman who saw him running down the street pulled his patrol car over the curb and ordered Phillip to stop and identify himself. Phillip showed the policeman his driver's license and told the officer that he was just jogging. When the policeman told Phillip to get in the back of the patrol car, Phillip asked if he was under arrest. The policeman said , "No, but you better get in the back of this car and shut up while I check you out and decide what to do with you!" Phillip got into the backseat of the patrol car and remained there around 20 minutes with the car door open while the policeman called Phillip's information from the driver's license into police headquarters over the radio. Finally satisfied that Phillip was not breaking any laws, the policeman let him go. If Phillip asserts a claim against the policeman for false imprisonment, will he likely prevail? No, because Phillip did not object to sitting in the patrol car. No, because the door of the patrol car remained open. Yes, if the language used by the policeman induced Phillip to remain in the car. Yes, as long as Phillip sustained physical harm as a result of remaining in the patrol car.

C is correct. Since the reasonable person usually obeys the directions of a policeman, the police officer's words would have been sufficient to overcome the reasonable person's will to leave. A is wrong because the policeman's assertion of legal authority and language would have made any protest by Phillip futile. B is wrong since Phillip was still confined if he believed that any attempt to leave the car would result in harm. D is incorrect because physical harm is not an element of false imprisonment.

Question 10 0 / 3 pts StarWalkers is under contract with the government to build a missile defense system. StarWalkers tests their prototype on a deserted stretch of desert. They fire up the satellite rocket system and huge clouds of smoke and particles are generated and these deposits are spread all over Pete's ranch located several acres west of StarWalkers testing property. The debris caused damage to delicate plantings on Pete's land. Pete files an action for trespass against StarWalkers. Which of the following would provide the best argument for StarWalkers? Starwalkers did not know that Pete had delicate plantings. Neither StarWalkers, nor its employees, ever set foot on Pete's property. StarWalkers had no reason to believe that the testing would result in deposits on Pete's property. StarWalkers did not intend to harm Pete's property

C is correct. Trespass requires intent. If StarWalkers had no reason to believe that its tests would result in this type of damage, then they lack the requisite intent. By definition, a prima facie case for trespass to land consists of: 1) an act of physical invasion of the plaintiff's real property by the defendant; 2) intent on the defendant's part to bring about a physical invasion of the plaintiffs real property; and 3) causation. StarWalkers' testing did result in debris falling on Pete's property, which is deemed a sufficient physical invasion for purposes of a trespass action. However, there are no facts indicating that StarWalkers intended to send their material onto Pete's land, making C the correct answer. A is incorrect. StarWalkers' knowledge of the plantings is irrelevant to a trespass claim. B is incorrect. Physical invasion does not require that the defendant enter onto the land of the plaintiff; proven intent, in a situation such as this, is sufficient to constitute a trespass. D is incorrect. Intent to harm the property is not the intent required for trespass; the intent required is an intent to physically invade the property of another without consent. Quiz Score: 18 out of 30

Question 3 0 / 2 pts Andrew loved to scare people, especially children. One day he dressed up like a clown and went to a nearby preschool. He ran into one of the classrooms before anyone could stop him and started laughing in a hysterical manner. He shouted, "I hate little children." As a result of this experience, Ron, a four year old in the class, began experiencing serious sleeping problems and heightened separation anxiety, which continued even after he left the preschool. Ron's parents sought medical treatment for Ron, and also took him to a child psychologist. Andrew is liable for intentional infliction of emotional distress: Only if he acted with the intent to cause distress to Ron. Because his behavior was harmful and offensive to a reasonable child. Only if Andrew caused physical injury to Ron. Because Ron suffered severe emotional distress.

C is incorrect because physical injury is not required, although Ron must prove actual damages. B is incorrect because harmful and offensive behavior is not a requirement for IIED, which requires intentional or reckless extreme and outrageous conduct causing severe emotional distress. A is incorrect because reckless conduct would also suffice.

Question 3 3 / 3 pts Art and Bill were involved in an argument over Gemma, a woman in their college class that both Art and Bill liked. Art said, "If you don't stay away from her, I'm going to kick you in the behind." Bill picked up a baseball near where he was standing and threw it at Art, shouting, "No one tells me what to do! I'll show you!" Art ducked and the baseball did not hit him, and instead hit Carla who happened to be walking by. Carla fell to the ground and screamed and suffered a bruise on the side of her head. What intentional torts have Art and Bill committed? Both Art and Bill committed assault. Bill committed assault only. Bill committed assault and battery. None.

C is the best answer because Bill committed assault when he threw the ball at Art while threatening Art, but Art's words alone will not constitute assault without any further action on his part; Bill committed battery as well although against Carla, who was hit by the ball, based on transferred intent, which provides that the intent to hit Art with the baseball transfers to Carla. A is incorrect because Art did not commit assault. B is incorrect because Bill also committed battery. D is incorrect because Bill committed both assault and battery.

Question 9 3 / 3 pts Alice was sleeping in the park when Dan, a delinquent teenager, threw a rock at her, aiming at her head. Unfortunately, Pat was jogging past and Pat was hit by the rock, seriously injuring Pat's eye. Alice was fine and never woke up. Dan is liable for: Assault on Alice and battery on Pat. Assault on Alice. Battery on Pat. None of the above.

C is the best answer because Dan's intent to commit battery on Alice will transfer to his actual battery of Pat. A is incorrect because Dan is not liable for the assault of Alice, since she was asleep and his conduct caused no actual apprehension of contact. B is incorrect for the same reason. D is incorrect because Dan will be liable for battery against Pat.

Question 13 0 / 15 pts Northern State has long had a reputation for growing the biggest and tastiest potatoes in the country. Growers of potatoes in Northern State recently began spraying Buggy onto their crops to prevent the spread of the Potato Maggot, an insect that can destroy young potato plants. Buggy is manufactured exclusively by Grow, Inc., at its plant in Southern State. When the plant is producing Buggy, it emits a fine, sticky, harmless mist as a byproduct. The mist drifts over Patrick's property which is adjacent to Grow's plant. Although Grow uses the best technology available, it is unable to prevent the release of the mist. Patrick brings suit against Grow on the theory of private nuisance to enjoin the production of Buggy at the Southern State plant. The mist has not caused any harm to Patrick's property. Which of the following facts, if established, will be helpful to Grow's defense? I. Grow began the manufacture of Buggy at the Southern State plant three years before Patrick acquired the land adjacent to the plant. II. Buggy is the only pesticide that can safely and effectively kill the Potato Maggot, which, if not controlled, would destroy Northern State's potato crop, its principal product. All of the above. I only. II only. None of the above.

C is the best answer because I would not be persuasive, since coming to the nuisance is generally not a good defense; but II is a persuasive defense because a private nuisance action requires a showing that defendant's interference with the use and enjoyment of plaintiff's property was unreasonable. To be characterized as unreasonable, the severity of the inflicted injury must outweigh the utility of defendant's conduct. Here, the fact that the pesticide is the only means of preventing destruction of the state's principal agricultural product would be very helpful to Grow's defense. Therefore A, B and D are incorrect.

Question 6 3 / 3 pts Morton owns a beach front home in Harbor Beach, California. After Axxon's oil rig failed, causing oil to spill onto the beach, Morton's property declined in value because the spill contaminated the beach area outside of his home and left a terrible smell in the entire area. The Harbor Beach City Council closed the beach for several months while clean-up efforts were made. Morton sued Axxon for nuisance. The most likely result is: Morton has no standing to bring a nuisance claim. Morton can sue only for private nuisance. Morton can sue for public and private nuisance. Morton can only sue for trespass to his land.

C is the best answer because an action for public nuisance is available given that the impact on his property is greater than that of the general public; he also has a claim for private nuisance based on the substantial interference with his use and enjoyment of his land. A is incorrect because he will have standing to bring a nuisance claim since his use and enjoyment of his land is substantially impacted. B is incorrect because he can also sue for public nuisance, given the harm to the public, and the fact that his harm is greater than the harm to members of the general public. D is incorrect because while trespass to land may also be a viable claim, it is not the only viable claim.

Question 7 3 / 3 pts Paula signed a consent form authorizing Dr. X to remove her gallbladder. After she went under anesthesia, Dr. X removed her appendix as well as her gallbladder. Which of the following is correct? Dr. X had no liability to Paula because of her consent. Dr. X is liable to Paula for assault and battery. Dr. X is liable to Paula for battery but not for assault. Dr. X is liable to Paula for assault, but not battery.

C is the best answer because battery is the intentional infliction of harmful or offensive bodily contact. B and D are incorrect because assault is an intentional act causing a reasonable apprehension of harmful or offensive contact and here, Paula was unconscious at the time of the surgery. A is incorrect because consent is a defense to intentional torts where it is informed, voluntary and given with legal capacity, but the surgery was outside the scope of Paula's consent.

Score for this quiz: 16 out of 20 Submitted Oct 11 at 9:21pm This attempt took 16 minutes. Question 1 2 / 2 pts Matt Pitt, a famous movie actor, enjoys Bad Bull Beer. Bad Bull Beer took a photo of Matt Pitt drinking a Bad Bull Beer on Malibu State Beach and decided to use it in an advertisement in a sports magazine and on the related website. If Matt Pitt did not authorize the advertisement, he can sue Bad Bull Beer for: Defamation, if the advertisement harmed Matt Pitt's reputation. False light, if Matt Pitt is not really a big drinker. Misappropriation, even if the photo was an accurate depiction. Intrusion upon seclusion, if Matt Pitt was with his family at the beach.

C is the best answer because misappropriation of likeness for financial benefit is an invasion of privacy. A is incorrect, because there no indication that the advertisement was false, which is required for a claim of defamation of a public figure. B is incorrect because the image is not false. D is incorrect because Matt was on a public state beach so his private space was not invaded as required to establish intrusion upon seclusion.

Question 5 3 / 3 pts Singer throws loud parties every weekend, with music blaring past midnight, in violation of local regulations. Singer lives in a residential neighborhood. People attending his parties cram the streets and loiter on the neighbor's front yards, occasionally strewing trash around the area, and causing noise late into the night. Paul, Singer's immediate next door neighbor, spoke to Singer several times about the trash and noise, to no avail, and Paul is tired of losing sleep every weekend. If Paul brings a claim against Singer for nuisance, Paul will most likely: Lose, due to a lack of standing. Lose, because Singer's conduct does not rise to the level of a public nuisance. Win, because Singer's conduct substantially interfered with Paul's use of his property. Win, only if Singer physically entered Paul's property.

C is the best answer here because a claim for private nuisance requires conduct that unreasonably and substantially interferes with Paul's use of his property, and Singer's conduct rises to this level. A is incorrect because Paul will have standing for a private nuisance claim. B is incorrect because Paul can bring a private nuisance action even if he cannot bring a public nuisance action. D is incorrect because Singer's entry onto Paul's property would not be required for nuisance which can be based on noise alone without physical entry.

Score for this quiz: 24 out of 30 Submitted Sep 20 at 9:55pm This attempt took 28 minutes. Question 1 3 / 3 pts Patty and Dan attended a realtor's conference. On the last night of the conference, after several drinks, Patty sees Dan talking with Alice, another realtor, and approaches him. Patty asked Dan if he would like to work with her, and Dan declined by saying, "Your business is failing, you're the worst, I heard that you haven't sold a house in 15 years." As she heard this comment, Alice sputtered her drink and widened her eyes in amusement. Assume that Patty sues Dan for defamation, and that the comments made by Dan were false, because Patty had sold several houses in the past year, which Dan would have known if he'd bothered to look. Who will prevail? Dan, because he did not intend to harm Patty's reputation. Dan, because his statements were only opinions. Patty, because Dan's statements were heard by Alice. Patty, because the statements were libelous.

C is the best answer here because defamation requires: 1) defamatory language on the part of the defendant; 2) the defamatory language must be "of or concerning" the plaintiff (i.e., it must identify the plaintiff to a reasonable reader, listener, or viewer); 3) publication of the defamatory language by the defendant to a third person; and 4) damage to the reputation of the plaintiff. Here Dan's statement has a clear adverse impact on Patty's reputation, and a reasonable listener would understand that her language applied directly to Patty. Clearly this meets the publication requirement, as Alice clearly heard and understood the comment to be adverse. This is slander per se which adversely affects her business pursuits. A is incorrect because Dan was negligent in making the statements about Patty not selling a house. B is incorrect because the statement about not selling the house is a factual statement, not a matter of opinion. D is incorrect because there is no writing so this is slander, not libel.

Question 9 3 / 3 pts Homer was thinking of hiring Ed as a professor for his new law school. Ed had published several scholarly articles on the subject of the death penalty, so Homer contacted Dean who had also published articles on the subject of the death penalty. Homer asked if Dean was familiar with Ed's work and Dean said that one of Ed's law review articles contained plagiarized content. This in fact was not true, and Dean was aware that it was not true. Dean's statement to Homer is: Libel per se. Libel per quod. Slander per se. Slander per quod.

C is the best answer, because the statement is oral, and therefore, slander rather than libel, and per se, because it concerns Ed's professional conduct. A is incorrect because the statement is not written or recorded in a manner that provides longevity so therefore it is not libel. B and D are both incorrect because a per quod statement requires external proof because the statement's meaning is not apparent on its face.

Question 6 3 / 3 pts Paul was having an argument with his co-worker Danielle in his office. Danielle yelled at Paul, "You don't know what you are doing!" Danielle then turned around and walked out of Paul's office slamming the door loudly on her way out. The lock jammed and Paul was locked into his own office. Paul had to call the maintenance man to un-jam the lock before he could get out of the office. Does Paul have a valid claim against Danielle? Yes, for false imprisonment, because Paul could not get out of the office. Yes, for assault, because Danielle yelled at Paul. No, because Danielle had no intent to cause Paul to be confined. No, even if Danielle intended to confine Paul.

C is the best answer, because there is no indication that Danielle intended to confine Paul to his office. A is incorrect because there is no valid claim for false imprisonment due to lack of intent. B is incorrect because there is no claim for assault because there is no indication of imminent harmful or offensive contact. D is incorrect because if Danielle intended to confine Paul, and did in fact do so, given that Paul had knowledge of his confinement, the elements of false imprisonment would be satisfied.

Question 7 3 / 3 pts Don was a practical joker. As a prank, he knocked on Pam's door one night, wearing a police officer's costume that he had rented. When Pam answered the door, Don told her that her husband had just been killed in an automobile accident and that she would have to come with Don to identify her husband's body. Pam recognized Don and was very outraged and distraught that he played such a cruel joke on her. Pam slammed the door in Don's face. If Pam asserts a claim for intentional infliction of emotional distress against Don, who is likely to prevail? Pam, because Don's act was calculated to cause emotional distress. Pam, because Don's conduct was extreme and outrageous. Don, because Pam did not experience severe emotional distress. Don, because he intended only to play a joke on Pam.

C is the correct answer. IIED is the intentional or reckless infliction of severe emotional or mental distress by extreme and outrageous conduct. A and B are wrong since Pam did not suffer severe emotional distress and therefore will not prevail. D is wrong since intending to play a joke can be sufficient for IIED, if the conduct is reckless and causes severe emotional distress through extreme and outrageous conduct.

Question 18 15 / 15 pts A State Senator was attending a football game when a member of the home team made a touchdown. The Senator jumped to his feet and cheered loudly, along with the rest of the crowd. Daniel, a freelance photographer, took the Senator's picture when he was cheering. When the photograph was developed, Daniel had it printed on targets that he packaged with plastic darts. Daniel marketed the targets and darts under the name of "The Cheering Senator's Dart Board Game," selling several thousand games. The Senator sues Daniel for invasion of privacy. On which of the following theories is the Senator most likely to prevail in a case against Daniel? Public disclosure of private facts Intrusion on seclusion Misappropriation False Light

C is the correct answer. Since Daniel sold games with the Senator's likeness imprinted on the targets, he would be liable for using the Senator's identity for a commercial purpose. None of the other theories for invasion of privacy are applicable to these facts.

Question 8 0 / 3 pts One morning, Peter brought his new car to the dealership for servicing. When he came to pick his car up at the end of the day, it was no longer at the dealership. Peter contacted the police and it was later discovered that the dealer had sold Peter's car to Hank. Hank did not know that the car sold to him actually belonged to someone other than the dealership. If Peter asserts a claim based on conversion against the dealership and Hank, Peter likely will prevail against: The dealership, but not Hank, since Hank was a good faith purchaser; Hank, but not the dealership, because the dealership no longer has possession of the car; Hank, but not the dealership, because the dealership had lawful possession of the car. Both the dealership and Hank, since they both exercised dominion over the car.

D is correct because conversion is an act by a defendant that interferes with a plaintiff's right of possession of chattel that is substantial and the intent required is the intent to perform the act that brings about the interference with plaintiff's right to possession. A is wrong since a bona fide purchaser of the (stolen) car is a converter, even if there is no way that he could have known that the car was stolen. B and C are wrong since the dealership interfered with Peter's rights of ownership of the car.

Question 11 15 / 15 pts A newspaper was publishing a series of articles about rampant exam cheating at a local high school. One of the photographs appearing in the article showed Pearl, a high school student, sitting on the lawn outside the high school, talking to her friends. The caption for the photograph said "Exam Cheating Rampant at High School." Pearl had never cheated on an exam. When the article and photograph was published, black squares were superimposed over the faces of Pearl and her friends. Although the article did not name Pearl, because she had a distinctive tattoo, several of her high school friends recognized her in the photograph. If Pearl asserts a claim against the newspaper for invasion of privacy, under which of the following theories is she most likely to succeed? Intrusion, because the photograph appearing in the newspaper was taken without her permission Appropriation of identity, because the photograph was a publication of her likeness without her permission Public disclosure, because her identity was revealed by publishing her photograph while she had a distinctive tattoo False light, because the photograph and the newspaper suggested that she cheated on exams

D is correct since the article inferred erroneously that Pearl cheated on exams. A is wrong since there would be no expectation of privacy for persons out in public, sitting outside a school. B is wrong for the same reason as A. C is wrong since that tort is inapplicable in that there were no private facts disclosed about Pearl.

Question 3 3 / 3 pts Late one night, Cindy hears someone outside her window. She throws open the door and finds Pete, the neighborhood troublemaker, swimming in her pool. Cindy yells at Pete, who immediately jumps over the fence into the adjoining property, owned by Dustin. Unfortunately for Pete, Dustin had just purchased a vicious and deadly pet python. When Pete landed on the other side of the fence, the python started strangling him. Pete is soon rescued, but not before suffering serious injury. If Pete sues Dustin for his injuries, he will likely: Lose, as long as Dustin has posted signs stating, "Beware of Attack Python." Lose, since Pete was a trespasser on Dustin's property. Win, because Dustin is liable for any damages to a trespasser on his property. Win, since Dustin cannot use a dangerous animal like a python to protect only his property.

D is correct. When dealing with the intentional use of a dangerous animal to protect property, Dustin loses and Pete wins, as such use is not permissible. Only reasonable force can be used in defense of property, not force that would cause death or serious injury. Here the python is deemed excessive deadly force. Also recall that indirect deadly force (like spring guns) is not permissible if this force cannot be lawfully used appropriately. In other words, deadly force is not permitted as a deterrent against mere trespassers. B is wrong. Generally the court will not apply strict liability in cases involving trespass onto land. Strict liability typically does not attach to injured trespassers as used against a landowner, but rather courts look to principles of negligence to determine if the landowner acted unreasonably by harboring an unreasonably dangerous animal etc. C is wrong. A landowner, who keeps a dangerous animal on his property that does damage to a trespasser, may be held liable for these damages under intentional tort principles. Since a landowner cannot use deadly force to protect his property, he cannot use deadly force indirectly to protect this same property by using a dangerous animal. A is wrong regardless of how many signs Dustin posts. Under intentional tort principles, Dustin's use of a python to protect his property knowing that the snake is likely to cause serious injury, may make him liable. Deadly force, either direct or indirect, can never be used to protect property.

Question 7 0 pts As City continued to grow in population, City decided to build a new fire station on the edge of a very large public park, across from a residential neighborhood. The fire station had a public warning system and was equipped with loudspeakers and an alarm. The speakers and alarm were intended to be used to warn people in the park of any public safety emergency situations, or any weather conditions, including tornadoes, requiring that people in the park take shelter or leave the park. The fire station tested the alarm every six months, at random times of the day or night. The sound of the alarm woke the surrounding neighbors, Paula, and her young infant son. Which of the following are correct? I. Paula has a claim for public nuisance. II. Paula has a claim for private nuisance. III. The neighbors have a claim for public nuisance. Group of answer choices All of the above. I and II only. II and III only. None of the above.

D is the best answer because (I) is incorrect because the harm to Paula is the same as the harm to the neighbors in terms of being woken up, so she cannot bring a claim for public nuisance. (II) is incorrect because a nuisance must be substantial, and the alarm is tested only once every six months, which is not substantial; a nuisance must also be unreasonable, and the public safety considerations in this scenario are significant, while the interference with use and enjoyment of property is relatively minimal. (III) is incorrect because the harm to the neighbors is outweighed by the safety considerations the alarm is designed to serve.

Score for this quiz: 18 out of 30 Submitted Aug 23 at 8:45pm This attempt took 27 minutes. Question 1 0 / 3 pts Don wanted to get his co-worker Pam into trouble with the boss Bob. Don hoped that Bob would fire Pam. Shortly before the weekly Monday morning meeting Don went to Pam's office and told her to stay in her office otherwise Don and his friends would go to Pam's house later that evening and beat her up. Pam realized that she could have left her office anytime, and could have gone to the meeting. But, Pam was frightened and stayed in her office while Don went to the meeting. When Bob saw Pam was not at the meeting, Bob decided to fire Pam. Pam has a viable claim against Don for: False imprisonment, because Don told Pam to stay in her office. Battery, because Don's conduct was offensive. Assault, because Don caused apprehension in Pam. None of the above.

D is the best answer because Pam does not have a claim for battery, since the element of touching is absent, Pam does not have a claim for assault, since any harmful of offensive touching was not imminent, and she does not have a claim for false imprisonment since she was not confined to her office.

Question 10 0 / 3 pts Pat was shopping in a hardware store looking for some screws. Dave worked undercover in the hardware store as a store detective trying to stop shoplifting. Dave watched Pat shopping via a video camera. Dave's view of Pat was obscured by a beam and when Dave again gained sight of Pat, Dave saw Pat taking his hand out of his pocket. Dave thought maybe Pat had put something into his pocket. In fact Pat had not done so, but had reached into his pocket to switch off his phone when it vibrated. Dave confronted Pat and brought Pat to a back room of the store, telling Pat that he was suspected of shoplifting and could not leave the store. Dave told Pat he had called the police and that Pat had to wait in the room until the police arrived. The police determined Pat had not stolen anything. Watching the video, police later determined that Dave had no basis for suspecting Pat. Is Dave liable to Pat for false imprisonment? No, because Pat was not harmed. No, because Dave had a reasonable basis for his conduct. Yes, if Pat was detained for at least 30 minutes. Yes, because Pat was detained without any reasonable basis.

D is the best answer because detention without any reasonable basis constitutes false imprisonment even if no harm is done, as long as the victim is aware of the detention. A is incorrect because harm is not required. B is incorrect because Dave did not have a reasonable basis for his conduct according to the police on these facts. C is incorrect because a detention for a minimal period of time will suffice for false imprisonment. Quiz Score: 18 out of 30

REVIEW TORTS 1 Question 1 0 pts Devin owns a bed and breakfast which he runs. In each room, Devin has installed hidden cameras to allow him to detect any theft by guests. Sometimes Devin watches the goings on within the bedrooms. On one occasion he observes a woman with a man who is not her husband. Devin told several people about the woman's conduct and also notified the husband. Is Devin liable to the woman? Group of answer choices a. No, assuming that the information Devin disclosed was accurate. a. No, because Devin has a privilege to monitor his property. a. Yes, because a reasonable person would find the woman immoral. a. Yes, because the woman had a reasonable expectation of privacy.

D is the best answer because the tort of intrusion upon seclusion depends on whether the woman had a reasonable expectation of privacy, as would probably be the case with a woman engaging in adultery. The fact that the information disclosed was accurate would be a defense to a claim for defamation but not to a claim for invasion of privacy based on intrusion upon seclusion, therefore A is incorrect. The fact that Devin owns the property does not authorize the intrusion under the circumstances, so B is incorrect. The fact that the woman 's conduct may be immoral does not justify or permit the disclosure so C is incorrect.

Question 3 3 / 3 pts Don is in love with Ann, who had recently announced her engagement to Peter. Angered by her engagement, Don sent Ann the following letter: "I hope you know what you are doing. The man you think you love is actually gay. A Friend." This caused Ann great emotional distress. She hysterically telephoned Peter, read him the letter, and told him that she was breaking off their engagement. The contents of the letter were not revealed to others. Peter, who was a young attorney in the state attorney's office, suffered serious humiliation and emotional distress as a result of the broken engagement. If Peter asserts a claim against Don based on defamation Don's best argument will be: Peter is a public figure. The issue is a matter of public interest. Peter acted only negligently, but not recklessly, in making the statement. The statement is true.

D is the best answer because truth is a complete or absolute defense to a claim of defamation. A is incorrect because if Peter is a public figure, it may be harder for him to make the required showing of malice, but this argument would not defeat the claim. B is incorrect for the same reasons discussed in reference to A. C is incorrect because, acting negligently would still potentially result in liability, while the truth would be a complete defense.

Question 5 2 / 2 pts Carol and Linda live next door to each other in the University dorm and visited each other's rooms all the time. Linda's parents sent her a beautiful gold bracelet for her birthday. Carol saw the bracelet and wanted to wear it to class the following day to impress another woman in the class. Carol asked to borrow it, but Linda said, "No." When Linda wasn't looking, Carol took the bracelet into Carol's dorm room. Carol wore it to class the next day, and meant to return it but left it on her desk. Linda did not even notice that the bracelet was missing. When Linda was in Carol's room the next day, while Carol was in the restroom, Linda saw the bracelet sitting on Carol's desk. Linda grabbed the bracelet and grew very angry. She left to go back to her own room. Which of the following is true? i. Carol is liable for trespass to chattel based on taking the bracelet ii. Carol is liable for conversion of the bracelet iii. Linda has the right to take possession of the bracelet All of the above. I and II only. II and III only. I and III only.

D is the best answer here because a trespass to chattel is intentional dispossession or use of chattel by another, but probably not an exercise of dominion and control over the chattel that is a serious interference with the rights of another, since Linda didn't even notice it was missing, so Carol's conduct does not rise to the level of a conversion. III is correct because Linda has the right to recapture her chattel since it was tortiously taken.

Question 13 0 pts Tattler, a popular tabloid newspaper, printed the sensational details about the extramarital sexual activities of Paul, a wealthy philanthropist who is accused of killing his spouse. The information was true. The disclosure of the information was highly embarrassing to Paul. Paul was unaware of the publication, because he was in custody in solitary confinement. Paul has a valid claim for: Group of answer choices Defamation, if the publication harms his reputation with the public. Invasion of privacy, because Paul's private life was exposed. Intentional infliction of emotional distress. None of the above.

D is the best answer. A is incorrect because truth is an absolute defense to defamation. B is incorrect because there is a legitimate public interest in Paul's life, given he is charged with murder, so he cannot claim public disclosure of private facts. Intentional infliction of emotional distress will not be viable if he does know of the publication, and is in solitary confinement, because there can be no actual severe emotional distress caused by the publication, so C is incorrect.

Question 10 3 / 3 pts Troy decided to play a prank on another college student, Evan, right before finals. When Evan was in the shower, Troy went to Evan's room. Evan's roommate Ron let Troy in. Troy loaded a fake virus program onto Evan's computer. The next time Evan booted up his computer, Troy's fake virus popped up, making it look like everything on the computer had been deleted. Evan freaked out for a few seconds, then remembered he had a back-up of everything. Later that evening Troy called Evan and told him how to turn it off. Luckily, the fake virus had not harmed the computer. Troy is liable for: I. Trespass to chattel. II. Intentional infliction of emotional distress. III. Conversion. IV. Trespass to land. I, II and IV. I and II only. II and III only. None of the above.

D is the best answer. Because there is no damage, there is no trespass to chattel. Because Evan did not suffer extreme distress, there is no IIED. Because there was no dispossession of the property, there is no conversion. Because the roommate gave consent to enter the room, there is no trespass to land. Quiz Score: 24 out of 30

Question 2 0 / 3 pts Fred and Barney were neighbors who had been in a long-running feud. Both of them had told each other not to set foot onto each other's property. One day when Barney was away on vacation, Fred went into Barney's garage to borrow a shovel. He thought he could return it and Barney would never know about it. While Fred was using the shovel, it broke, so Fred decided to put it into his trash because it was useless. When Barney returned from his vacation, Barney left Fred a note in his mailbox asking if Fred had seen the shovel and Fred did not reply. Fred is liable to Barney for: I. Trespass to Land II. Trespass to Chattel III. Conversion All of the above. I and II only. II and III only. I and III only.

D is the best answer. Fred is liable for his entry onto Barney's property, which is a trespass to land for which there is no consent. Fred is liable for interference with possession of the shovel, resulting in the destruction and disposal of the shovel, which permanently deprived Barney of the shovel (conversion). Since the shovel is no longer available for Barney, there is no trespass to chattel.

Question 3 2 / 2 pts Due to an oil rig explosion set off by a disgruntled employee off the coast of Santa Barbara, California, oil spilled into the area and washed up on the local beaches. Frank, a local fishermen has lost his livelihood, because the pollution has ruined the catch. Bill, a local businessman with a hamburger restaurant right on the beachfront, lost his customers, after oil washed ashore onto his property. The best claim(s) are: I. Public nuisance because the harm extends to the public's use of the beach. II. Private nuisance for Frank. III. Private nuisance for Bill. All of the above. I and II. II and III. I and III.

D is the best answer. There is a claim for public nuisance because the oil has interfered with a right common to the general public in terms of their use of the beach. There is no private nuisance claim for Frank because there is no interference with an interest in his own land that is substantial and unreasonable, since he does not own the beach or the ocean. Bill does have a private nuisance claim, since his business on the beachfront has been adversely impacted by the oil, and the impact is a result of wrongful conduct, creating a substantial and unreasonable interference with his business.

Question 4 0 / 2 pts Ron put a hidden camera in the dormitory room of his next door neighbor, Ned, when Ned was out of town for the weekend. The camera captured images of Ned sleeping and snoring loudly. Ned's snoring was not only loud, but very strange sounding, due to a childhood injury to his nose. Because of this Ned was allowed to room alone. Ron posted some of the video of Ned snoring in bed on the internet with the caption "Freakish Loud Snoring," and the video went viral, generating millions of hits online. Ron ended making money off the video because he had pop up ads that came up in connection with the video. If Ned sues Ron for invasion of privacy, he can assert: I. False light, because the video is embarrassing to Ned. II. Misappropriation of Ned's image. III. Public disclosure of private facts. IV. Intrusion upon seclusion, because Ned's snoring is a private matter. All of the above. I and III only. II and III only. III and IV only.

D is the best answer. There is no false light claim because the video is not false. Misappropriation of image is typically actionable where the image of another is used to promote a product, not where it is used to make money for a publication. Public disclosure of private facts is available here because making a person's snoring public would be highly offensive to a reasonable person. Intrusion upon seclusion is available because Ned has a private room due to his snoring, and the camera intruded on this set up.

Question 8 0 / 3 pts Don and Patty are riding the metro train to work. They do not know each other, but just happened to get on at the same stop. Both of them are standing, because the car is crowded with commuters. The car comes to a screeching stop due to a dog on the tracks and they both lose their balance. Patty is wearing 4 inch platform heels and is about to fall when Don grabs Patty by the waist. Patty sues for Don for battery. Will she likely recover? Yes, if Don intended to put his arm around Patty's waist. Yes, this was a a harmful or offensive touching. No, assuming Don put his arm around Patty's waist by accident. No, this was socially acceptable conduct by Don.

D is the best answer. To constitute a battery, there must be harmful or offensive touching. This was not the case here. Battery requires the following elements: 1) an act by the defendant that brings about harmful or offensive contact to the plaintiffs person; 2) intent on the part of the defendant to bring about harmful or offensive contact to the plaintiffs person; and 3) causation. Based on this, trying to keep Patty from falling is neither offensive nor harmful. Note that although "offensive" conduct is conduct that the plaintiff has not expressly or implicitly consented to, consent can be implied from custom, conduct or law. Here, helping someone stay upright is done with an implied consent based on custom and is not offensive. A is incorrect because even if Don intended to put his arm around Patty's waist, there was no harmful of offensive touching. B is incorrect for the same reason. C is incorrect because even if placing his arm around Patty's waist was not by accident, there is no harmful or offensive touching as required for battery.

Question 2 3 / 3 pts Dr. David and Dr. Paul were having an argument at their medical offices one evening. Dr. David became very angry and left Dr. Paul's office, slamming the door behind him. Dr. Paul was unable to get the door open and had to call security in order to get out of his office. If Dr. Paul sues Dr. David for false imprisonment, will Dr. Paul prevail? Yes, because Dr. Paul was confined. Yes, because Dr. David caused the door to jam. No, because Dr. Paul was unharmed. No, because Dr. David did not intend to confine Dr. Paul.

The best answer is D because intent is required for false imprisonment, which is an intentional tort. A is incorrect because confinement without intent is not enough for false imprisonment. B is incorrect because causation without intent is not enough for false imprisonment. C is incorrect because harm is not required for false imprisonment where there is awareness of confinement.


Set pelajaran terkait

Wilkins Chapter 2 Evidence-Based DHP

View Set

Ch.9 Personal Finance Study Guide

View Set

Strategic Management Practice Exam 2

View Set

Chapter 11: Performance and Learning

View Set

Politics in the Middle East Final

View Set

historia examen #4 lol te vas a colgar bye

View Set

Components of Master Treatment Plan

View Set